Download as pdf or txt
Download as pdf or txt
You are on page 1of 116

SSEN 수학 (하) 12.

집합의 뜻과 표현

[ !!") ∼ !!"- ] 다음 집합을 벤다이어그램으로 나타내시오.


A단계 기본 다잡기 15. 1 5 . "!보다 작은 홀수인 자연수의 집합 *

12-1 집합과 원소

16. 1 6 . 6 7 01 2 1 &1 8 " '&1 9 ( ' 7 !3

[ !!!" ∼ !!!# ] 다음 중 집합인 것은 ○ 를 집합이 아닌 것은 × 를


& ' 안에 써 넣으시오.
1. .
1
공부를 잘하는 학생들의 모임 & '

2. .
2 ‘sinsago’에 들어 있는 알파벳의 모임 & '
[ !!": ∼ !!"/ ] . 의 양의 약수의 집합을 * 라 할 때, 집합 * 를 다음
방법으로 나태시오.

3. .
3
우리 반에서 키가 큰 학생들의 모임 & ' 17. 원소나열법
1 7 .

4. .
4 ( 보다 작은 소수의 모임 & '
18. 조건제시법
1 8 .

[ !!!) ∼ !!"! ] "( 의 양의 약수의 집합을 * 라 할 때, 다음 안에


기호 ∈, ∉ 중 알맞은 것을 써 놓으시오
19. 벤다이어그램
1 9 .

5. .
5 " *

6. , *
12-3 집합의 원소의 개수

.
6

7. .
7 - *
[ !!(! ∼ !!(, ] 다음 집합이 유한집합이면 ‘유’를 무한집합이면 ‘무’를
& ' 안에 써넣으시오. 또 공집합이면 ‘공’을 함께 적으시오.
8. .
8 . *
20. 2 0 . 0"4 ,4 )4 :4 /4 ⋯3 & '

9. .
9 / *
21. 2 1 . 0"4 (4 ,4 ⋯ 4 )!3 & '
10. 1 0
. "( *

22. 01 2 1는
2 2 . !과 "사이에 있는 자연수3 & '

12-2 집합의 표현

[ !!"" ∼ !!"( ] 다음 집합을 원소나열법으로 나타내시오 .


23. 01 2 1는
2 3 .
,의 양의 배수3 & '

11. 01 2 1는 1 1
.
#의 양의 배수3
[ !!(# ∼ !!(- ] 다음을 구하시오.
24. ;&∅'
12. 01 2 1는
2 4 .

1 2
. (!의 양의 배수3

[ !!", ∼ !!"# ] 다음 집합을 조건제시법으로 나타내시오 .


25. ;&0=4 >4 ?3'
13.
2 5 .

1 3
. 0"4 (4 ,4 #4 ⋯3

26. ;&01 2 1( 9 , @ !4 1는 정수3'


14.
2 6 .

1 4
. 0)4 "!4 ")4 ⋯ 4 "!!3

- 1 -
SSEN 수학 (하) 12. 집합의 뜻과 표현

12-4 부분집합 12-5 서로 같은 집합


☯ ☯

[ !!!(: ∼!!!,! ] 다음 두 집합 A , B 의 포함 관계를 기호 ⊂ 를 [ !!,/ ∼ !!#( ] 다음 두 집합 사이의 관계를 기호 7 또는 ≠ 를


사용하여 나태시오. 사용하여 나타내시오.
27. 2 7
. A 7 01 2 1는 ,의 양의 배수3, B 7 0D 2 D는 -의 양의 배수3 39. 3 9 . * 7 0"4 (4 ,3, 6 7 0,4 "4 (3

28. 2 8
. A 7 01 2 1는 정사각형3, B 7 0D 2 D는 마름모3 40. 4 0 . * 7 01 2 1( 9 ,1 7 !3, 6 7 0"4 ,3

41. * 7 01 2 1는 .의 양의 배수3, 6 7 0.4 "-4 (#4 ,(4 ⋯3


29.
4 1 .

2 9
. A 7 01 2 1( 7 #3, B 7 0(3

30. A 7 01 2 1는 "보다 작은 자연수3,


42.
3 0
.

B 7 0D 2 D는 E D E≤ "인 정수3 4 2 . * 7 0(4 )4 "!3, 6 7 01 2 1는 "!의 양의 약수3

[ !!," ∼ !!,# ] 집합 0!4 "4 (3의 부분집합 중 다음 집합을 43. 집합


4 3 . 01 2 1는 /의 양의 약수3인 진부분집합을 모두 구하시오.
구하시오.
31. 3 1
.
원소가 !개인 것

32. 3 2
. 원소가 "개인 것

12-6 부분집합의 개수

33. 3 3
. 원소가 (개인 것
[ !!## ∼ !!#: ] 집합 * 7 0"4 (4 ,4 #3에 대하여 다음을 구하시오.
44. 집합 * 의 부분집합의 개수
4 4 .

34. 3 4
. 원소가 ,개인 것

45. 집합 * 의 진부분집합의 개수
4 5 .

[ !!,) ∼ !!,. ] 다음 집합의 부분집합을 모두 구하시오 .


35. 3 5
.
0∅3

46. 집합 * 의 부분집합 중 ( 를 반드시 원소로 갖는 집합의 개수


4 6 .

36. 3 6
. 09 "4 "3

47. 집합 * 의 부분집합 중 " , , 을 원소로 갖지 않는 집합의 개수


4 7 .

37. 3 7
.
014 D4 G 3

38. 3 8
. 0"4 (4 ,4 #3

- 2 -
SSEN 수학 (하) 12. 집합의 뜻과 표현

51.

B단계 유형 뽀개기 방정식 1, 8 (1( 9 ,1 7 ! 의 해집합을 * 라 할 때, 옳은


5 1 .

것만을 보기에서 있는 대로 고른 것은?

| 보 기 |

유형 01 집합의 뜻

ㄱ. !∈* ㄴ. "∈*
ㄷ. (∈* ㄹ. ,∈*
48. 다음 중 집합인 것은?
① ㄱ, ㄴ ② ㄱ, ㄷ ③ ㄱ, ㄹ
4 8
.

① 아름다운 꽃들의 모임
④ ㄴ, ㄹ ⑤ ㄷ, ㄹ
② "! 에 가까운 수의 모임
③ 작은 수의 모임
④ 교복이 잘 어울리는 학생들의 모임
⑤ " 보다 크고 ( 보다 작은 자연수의 모임

유형 03 집합의 표현방법

49. 4 9
.
보기에서 집합인 것은 모두 몇 개인가? 52. 오른쪽 그림과 같이 벤다이어그램으로
5 2 .

| 보 기 | 표현된 집합 * 를 조건제시법으로 바르게


ㄱ. 태양계 행성들의 모임 나타낸 것은?
ㄴ. 우리 반에서 혈액형이 O 형인 학생들의 모임 ① * 7 01 2 1는 ) 양의 약수3
ㄷ. 우리나라의 높은 산들의 모임 ② * 7 01 2 1는 "! 양의 약수3
ㄹ. ,!보다 작은 # 의 양의 배수의 모임 ③ * 7 01 2 1는 ") 양의 약수3
ㅁ. 소리가 큰 악기들의 모임
④ * 7 01 2 1는 ")이하의 ,의 양의 배수3
① "개 ② (개 ③ ,개 ⑤ * 7 01 2 1는 ")이하의 )의 양의 배수3
④ #개 ⑤ )개

유형 01 집합과 원소 사이의 관계 53.


다음 집합 중 나머지 넷과 다른 하나는?
5 3 .

① 0"4 (4 ,4 ⋯4 /3
50. 5 0
.
실수 전체의 집합을 J , 유리수 전체의 집합을 K 라 할 때, ② 01 2 1 @ "!4 1는 자연수3
다음 중 옳은 것은? (단, L 7 MN
9") ③ 01 2 1 ≤ /4 1는 자연수3
N
① M( ∉ J ② L∈J
#
③ L ∈J ④ 0D 2 D는 한 자리 자연수3
" ⑤ 0D 2 " @ D @ "!4 D는 자연수3
④ N ∈K ⑤ MN
/ ∉K
" 8 MN
(

- 3 -
SSEN 수학 (하) 12. 집합의 뜻과 표현

54. 5 4
. 다음 중 집합 * 7 01 2 1 7 (= × ,>4 =4 >는 자연수3의 원소가 58. 집합 * 7 01 2 1는 1 @ O인 #의 양의 배수3가 공집합이 되도록
5 8 .

아닌 것은? 하는 자연수 O 의 최댓값을 구하시오.


① - ② "( ③ ".
④ #) ⑤ )#

55. 5 5
.
집합 * 7 09 "4 !4 (3에 대하여 집합
A 7 0=> 2 =∈* 4 >∈* 3를 원소나열법으로 나타내시오.
유형 05 유한집합의 원소의 개수

59. 두 집합
5 9 .

* 7 01 2 1 7 (; 9 "4 ;은 )이하의 자연수3

6 7 01 2 1는 "!!보다 작은 ""의 양의 배수3


에 대하여 ; &6 ' 9 ; &* ' 의 값은?
① # ② ) ③ -
④ : ⑤ .

유형 04 유한집합과 무한집합

56. 5 6
.
다음 중 유한집합인 것은?
60. 두 집합 * 7 0"4 (4 ,3, 6 7 01 2 1는 " ≤ 1 ≤ "!인 소수3에
6 0 .

① 0(4 #4 -4 .4 "!4 ⋯3
대하여 집합 P 7 01 8 D 2 1∈* 4 D∈63일 때, ; &P ' 를 구하시오.
② 01 2 1 7 (; 9 "4 ;은 자연수3
③ 01 2 1는 가장 작은 자연수3
④ 01 2 1는 1( @ "인 유리수3
⑤ 0= 8 > 2 ! @ = @ "4 ! @ > @ "3

61. 다음 중 옳은 것은?
6 1 .

57. 5 7
.
다음 중 공집합인 것은? ① ;&0"4 (4 ,3'@ ;&0#4 )4 -3'
① 0∅3 ② * 7 0!3이면 ;&* '7 ! 이다.
③ ;&* '7 !이면 * 7 0∅3이다.
② 01 2 1 (
8 " 7 !4 1는 실수3
④ ;&0∅3'9 ;&∅'7 "
③ 01 2 E 1E@ "3
⑤ ;&0-!3'9 ;&0))3'7 )
④ 01 2 1 9 " @ !3
(

⑤ 01 2 &1( 8 ,'&1 8 "' 7 !4 1는 실수3

- 4 -
SSEN 수학 (하) 12. 집합의 뜻과 표현

62. 6 2
. 두 집합 65. 집합 * 7 0=4 0>4 ?33에 대하여 다음 중 옳은 것은?
6 5 .

* 7 0&14 D' 2 1 8 D 7 "4 14 D는


( ( 정수3 ① 0=3∈* ② 0>3∈* ③ 0>4 ?3∈*
6 7 01 2 1는 O이하의 자연수4 O는 자연수3 ④ 0=4 >3⊂ * ⑤ 0=4 >4 ?3⊂ *
에 대하여 ;&* '8 ;&6'7 "" 일 때, O 의 값을 구하시오.

유형 06 기호 ∈, ⊂의 사용

63. 집합 * 7 0=4 >4 ?3에 대하여 옳은 것만을 보기에서 있는


66.
6 3
.

집합 * 7 0∅4 "4 0"4 (34 ,3에 대하여 다음 중 옳지 않은


대로 고른 것은?
6 6 .

것은?
| 보 기 | ① ∅⊂* ② "∈* ③ 0"4 (3∈*
ㄱ. > ⊂ * ㄴ. Q ∉ * ④ 0"4 ,3⊂ * ⑤ 0(3∈*
ㄷ. 0?3∈* ㄹ. 0=4 >3⊂ *
ㅁ. * ⊂ 0=4 >4 ?4 Q3

① ㄱ, ㄷ ② ㄴ, ㄹ
③ ㄴ, ㄷ, ㅁ ④ ㄴ, ㄹ, ㅁ
⑤ ㄴ, ㄷ, ㄹ, ㅁ

유형 07 집합 사이의 포함 관계

67. 세 집합 A 7 09 "4 !4 "3,


6 7 .

64. 6 4
. 두 집합 * , 6 가 오른쪽 벤다이어그림과 B 7 01 2 1는 9 " @ 1 @ "인 정수3, S 7 0E 1E 2 1∈A 3사이의 포함
같을 때, 다음 중 옳지 않은 것은? 관계를 바르게 나타낸 것은?
① A⊂B⊂S ② A⊂S⊂B
① "∈6 ② #∉ *
③ B⊂A⊂S ④ B⊂S⊂A
③ 0"4 (4 ,3⊄ 6 ④ 0!3⊄ *
⑤ S⊂B⊂A
⑤ 0!4 "3⊂ 6

- 5 -
SSEN 수학 (하) 12. 집합의 뜻과 표현

68. 6 8
. 다음 중 두 집합 * , 6 사이의 포함 관계가 71. 세 집합 * 7 01 2 1 ≥ )3, 6 7 01 2 1 ≥ =3,
7 1 .

오른쪽 벤다이어그램과 같은 것은? P 7 01 2 1 ≥ (3에 대하여 * ⊂ 6 ⊂ P 가 성립하도록 하는 정수


① * 7 0"4 ,4 )4 -3, 6 7 0"4 ,4 )4 :3 = 의 개수는?
② * 7 01 2 1는 짝수3, 6 7 0(4 #4 -4 .4 "!3 ① " ② ( ③ ,
③ * 7 01 2 1는 "!이하의 자연수3, ④ # ⑤ )

6 7 01 2 1는 "!보다 작은 자연수3

④ * 7 01 2 1 7 ,;4 ; 7 "4 (4 ,3,


6 7 01 2 1는 ")의 양의약수3
⑤ * 7 ∅, 6 7 0∅3

72. 두 집합
7 2 .

* 7 01 2 1( 8 1 9 - 7 !3
6 7 01 2 1는 =보다 작은 정수3
에 대하여 * ⊂ 6 를 만족시키는 정수 = 의 최솟값을 구하시오.
69. 6 9
.
세 집합 * 7 0!4 "4 (3, 6 7 01 9 (D 2 1∈* 4 D∈* 3,
P 7 01 9 D 2 1∈* 4 D∈* 3사이의 포함 관계를 바르게 나타낸 것은?
① *⊂ 6⊂P ② *⊂P⊂6
③ 6⊂ *⊂P ④ 6⊂P⊂*
⑤ P⊂ 6⊂*

73. 두 집합 A 7 0,4 9 U3, B 7 0U 8 (4 U 9 #4 "3에 대하여


7 3 .
(

A ⊂ B 가 성립할 때, 실수 U의 값을 구하시오.

유형 08 집합 사이의 포함 관계가

성립하도록 하는 상수 구하기
70. 7 0
.
두 집합 * 7 01 2 ! @ 1 ≤ ,3, 6 7 01 2 = @ 1 @ (= 8 /3에
유형 09 부분집합 구하기

대하여 * ⊂ 6 가 성립할 때, 정수 =의 개수를 구하시오.

74. 집합
7 4 . 01 2 1 7 ,; 9 (4 ;은 " ≤ ; ≤ )인 소수3의 부분집합을

모두 구하시오.

- 6 -
SSEN 수학 (하) 12. 집합의 뜻과 표현

75. 7 5
. 집합 * 7 0"4 (4 ,3에 대한 설명 중 옳은 것은? 79. 두 집합 * 7 0,4 -4 .34 6 7 01 9 (4 1 8 "4 1 8 ,3에
7 9 .

① ∅ 은 * 의 부분집합이다. 대하여 * ⊂ 6 4 6 ⊂ * 일 때, 상수 1의 값은?


② 0"4 (4 ,3⊄ * 이다. ① , ② # ③ )
③ 원소가 하나뿐인 * 의 부분집합은 " 개이다. ④ - ⑤ :
④ 원소가 (개인 * 의 부분집합은 ( 개이다.
⑤ 원소가 ,개인 * 의 부분집합은 , 개이다.

76. 7 6
. 집합 * 7 01 2 1는 ")보다 작은 ,의 양의 배수3에 대하여
6 ⊂ * 이고 ;&6'7 , 을 만족시키는 집합 6의 개수를 구하시오.
80. 8 0 . * 7 0(4 #3일 때, 집합 * 와 서로 같은 집합인 것만을
보기에서 있는 대로 고른 것은?

| 보 기 |
ㄱ. 01 2 1는 )보다 작은 짝수인 자연수3
ㄴ. 01 2 1는 #의 양의 약수3
ㄷ. 01 2 " ≤ 1 ≤ -4 1는 짝수3
ㄹ. 01 2 &1 9 ('&1 9 #'7 !3

① ㄱ, ㄴ ② ㄱ, ㄹ ③ ㄴ, ㄷ
77. 집합 * 7 0(4 #4 -4 .4 "!3의 부분집합 중에서 원소의 합이
④ ㄴ, ㄹ ⑤ ㄷ, ㄹ
7 7
.

(!이상인 부분집합의 개수를 구하시오.

유형 10 서로 같은 집합

81. 두 집합 * 7 0= 8 (>4 #34 6 7 0(= 9 ,>4 9 )3에 대하여


8 1 .

* 7 6 일 때, = 8 > 의 값을 구하시오. (단, =4 >는 상수이다.)


78. 7 8
. 두 집합 * 7 0(=4 = 8 )4 ,34 6 7 0= 9 (=4 9 (4 #3에
(

대하여 * 7 6 일 때 , 상수 = 의 값을 구하여라.

- 7 -
SSEN 수학 (하) 12. 집합의 뜻과 표현

82. 8 2
. 두 집합 * 7 01 2 1는 "!의 양의 약수3, 유형 12 특정한 원소를 갖는(갖지 않는) ☯

6 7 0"4 "!4 = 9 (4 > 8 (3가 서로 같을 때, 자연수 =4 > 에 대하여 부분집합의 개수


=>의 값은?
① . ② "! ③ "( 86. 집합 * 7 01 2 1는 ".의 양의 약수3에 대하여 A ⊂ * 이고,
8 6 .

④ ") ⑤ ". A ≠ * 인 집합 A 중에서 "4 (를 반드시 원소로 갖는 집합의 개수를


구하시오.

유형 11 부분집합의 개수

83. 8 3
. 집합 * 의 부분집합의 개수가 -# 이고, 집합 6의
진부분집합의 개수가 "(: 일 때, ;&* '8 ;&6'의 값은?
① / ② "" ③ ",
④ ") ⑤ ":
87. 집합V 7 0"4 (4 ,4 #4 )4 -4 :3에 대하여 (∈A 4 ,∈A 4
8 7 .

: ∉ A 를 모두 만족시키는 집합 V 의 부분집합 A 의 개수는?


① ( ② # ③ .
④ "- ⑤ ,(

84. 8 4
. 집합 * 7 01 2 1, 9 ,1( 9 1 8 , 7 !4 1는 자연수3의
부분집합의 개수를 구하시오.

88. 집합 * 7 01 2 1는 O 이하의 자연수4 O는 자연수3의 부분집합


8 8 .

중에서 "4 (를 반드시 원소로 갖고 ,4 #4 ) 를 원소로 갖지 않는


부분집합의 개수가 ,( 일 때, O 의 값은?
① . ② / ③ "!
④ "" ⑤ "(

85. 8 5
. 집합 * 7 01 2 1는 (! 이하의 자연수3의 부분집합 중에서 모든
원소가 "( 의 약수로만 이루어진 집합의 개수를 구하시오.

- 8 -
SSEN 수학 (하) 12. 집합의 뜻과 표현

유형 13* ⊂ A ⊂ 6를 만족시키는 92.


두 집합 * 7 0"4 (4 ,4 ⋯4 ;34 6 7 0"4 (4 #3에 대하여


9 2 .

집합 A의 개수 6 ⊂ A ⊂ * 를 만족시키는 집합 A 의 개수가 "- 일 때, 자연수 ; 의


값을 구하시오.
89. 8 9
. 두 집합 * 7 01 2 1( 9 :1 8 "! 7 !3,
6 7 01 2 1는 (!의 양의 약수3에 대하여 * ⊂ A ⊂ 6를 만족시키는
집합 A 의 개수는?
① # ② . ③ "-
④ ,( ⑤ -#

유형 14 여러 가지 부분집합의 개수

93. 집합 * 7 01 2 1 7 ,O4 O는 O @ :인 자연수3의 부분집합


90.
9 3 .

오른쪽 그림과 같은
중에서 , 또는 - 을 원소로 갖는 부분집합의 개수는?
9 0
.

벤다이어그램의 두 집합 * 4 6 에 대하여 ① #. ② )- ③ -!
* ⊂ A ⊂ 6 를 만족시키는 집합 ④ -( ⑤ -#
A 중에서 Q를 원소로 갖지 않는 집합의
개수는?
① ( ② # ③ .
④ "( ⑤ "-

94. 집합 * 7 01 2 1는 ,! 미만의 )의 양의 배수3의 부분집합


9 4 .

중에서 적어도 한 개의 홀수를 원소로 갖는 부분집합의 개수를


91. 9 1
. 두 집합 * 7 01 2 1는 -의 양의 약수3, 구하시오.

67 1 0 2 1 7 N"(; 4 14 ; 3
은 자연수 에 대하여 * ⊂ A ⊂ 6를

만족시키는 집합 A 의 개수를 구하시오.

- 9 -
SSEN 수학 (하) 12. 집합의 뜻과 표현

98.

C단계 실력 굳히기 집합 Y 7 0"4 (4 ,4 ⋯4 "!!3의 부분집합 * 가 다음 조건을


9 8 .

모두 만족시킬 때, 집합 * 의 원소의 개수의 최솟값은?

95. 9 5
. 원소의 개수가 , 인 집합 V 가 다음 조건을 모두 만족시킬 때, (가) (∈* 4 )∈*

! 과 ) 를 제외한 집합 V 의 나머지 원소를 구하시오. (나) 1∈* 4 (1∈Y 이면 (1∈*

(가) !∈V4 )∈V ① / ② "! ③ ""


④ "( ⑤ ",
(나) W∈V4 X∈V 이면 &W 8 X'∈V (단, W ≠ X )

99. 두 자연수 =4 > 의 최대공약수를 Z&=4 >' 라 하자. 집합


9 9 .

Y 7 01 2 1는 "!! 이하의 자연수3의 부분집합 * O&;'을


* O&;'7 01 2 Z&;4 1'7 O3 ( O4 ;은 자연수)
라 할 때, 옳은 것만을 보기에서 있는 대로 고른 것은?
96. 9 6
. 자연수를 원소로 갖는 집합 V 가 조건
‘1∈V 이면 . 9 1∈V ’ | 보 기 |

를 만족시킬 때 , 다음 중 옳지 않은 것은? ㄱ. .∈* #&(!'


① -∈V 이면 (∈V 이다. ㄴ. * ,&-'7 * ,&"('
② 원소가 "개인 집합 V 가 존재한다. ㄷ. * "&:'의 원소의 개수는 .-이다.
③ 원소가 (개인 집합 V 는 , 개이다.
④ 원소가 )개인 집합 V 는 , 개이다. ① ㄱ ② ㄴ ③ ㄱ, ㄷ
④ ㄴ, ㄷ ⑤ ㄱ, ㄴ, ㄷ
⑤ " 을 원소로 갖는 집합 V 는 : 개이다.

100. 두 자연수 =4 > 에 대하여 =4 > 를 각각 # 로 나누었을 때의


0 0 .
1

나머지가 서로 같을 때, = ≡ >로 나타내기로 하자. 예를 들면 , 과


97. 9 7
. 두 집합 * 7 0"4 (4 ,4 #4 =34 6 7 0"4 ,4 )3에 대하여 : 은 # 로 나누었을 때의 나머지가 , 으로 같으므로 , ≡ :로 나타낼
집합 A 7 01 8 D 2 1∈* 4 D∈63라 할 때, ;&A '7 "!이 되도록 하는 수 있다. 집합 Y 7 01 2 1는 "!! 이하의 자연수3의 두 부분집합

자연수 = 의 최댓값을 구하시오. * 7 01 2 (1 ≡ "3, 6 7 0 1 2 1, 8 (1( ≡ 1( 9 13


에 대하여 ;&* ' 8 ;&6 ' 의 값은?
① () ② ,! ③ ,)
④ #! ⑤ #)

- 10 -
SSEN 수학 (하) 12. 집합의 뜻과 표현

101. 0 1
1 . ( 이상의 두 자연수 ;4 O 에 대하여 집합 * &;4 O'를 104. 집합 * 7 0=4 >4 ?3에 대하여 P&* ' 7 0A 2 A ⊂ * 3 라 할
0 4 .
1

* &;4 O'7 01 2 ; @ 1 @ ; 8 O4 1는 짝수3 때, 집합 P&* ' 의 부분집합의 개수를 구하시오.


라 할 때 , 옳은 것만을 보기에서 있는 대로 고른 것은?

| 보 기 |
ㄱ. #∈* &(4 ,'
ㄴ. 집합 * &;4 O' 의 원소의 개수와 집합 * &;(4 O' 의 원소의
개수는 같다.
ㄷ. * &(;4 O'⊂ * &(; 9 "4 O' 이면 * &O4 O'⊂ * &O 8 "4 O'

① ㄱ ② ㄱ, ㄴ ③ ㄱ, ㄷ
④ ㄴ, ㄷ ⑤ ㄱ, ㄴ, ㄷ

105. 0 5 .
1 ,이상의 자연수 ^에 대하여
* ^ 7 01 2 1는 ^ 이하의 소수34

102. 0 2
1 . 집합 * 7 0"4 (4 ,4 #4 )3의 부분집합 중 원소의 개수가
6^ 7 01 2 1는 ^의 양의약수3

일 때, 옳은 것만을 보기에서 있는 대로 고른 것은?


(인 부분집합은 "! 개이다. 이 집합을 각각
6O &O 7 "4 (4 ⋯4 "!'라 하고 집합 6O의 모든 원소의 합을 VO 라 | 보 기 |
할 때, V" 8 V( 8⋯8 V"! 의 값은? ㄱ. ;&* ,'8 ;&6,'7 #
① )- ② -! ③ -# ㄴ. * - ⊂ 6^ 을 만족시키는 ^ 의 최솟값은 ,! 이다.
④ -. ⑤ :( ㄷ. ^ _ - 일 때 , 집합 * ^ 의 부분집합의 개수는 집합 6^ 의
부분집합의 개수보다 크거나 같다.

① ㄱ ② ㄱ, ㄴ ③ ㄱ, ㄷ
④ ㄴ, ㄷ ⑤ ㄱ, ㄴ, ㄷ

103. 집합 01 2 E1E≤ )4 1는 정수3의 세 부분집합 * 4 64 P가


106.
0 3
1 .

다음 조건을 모두 만족시킨다. 집합 A 7 01 2 1는 "! 이하의 자연수3의 원소 ; 에 대하여


0 6 .
1

A 의 부분집합 중 ; 을 최소의 원소로 갖는 모든 집합의 개수를


(가) ;&* '7 ( `&;' 이라 하자. 보기에서 옳은 것만을 있는 대로 고른 것은?
(나) 6 7 01 8 " 2 1∈* 3
(
| 보 기 |
(다) P 7 0(1 8 O 2 1∈* 4 O는자연수3
ㄱ. `&.'7 #

6 7 P 일 때, 집합 * 의 모든 원소의 합을 U4 O 의 값을 \ 라 하자. ㄴ. =∈A 4 >∈A 일 때, = @ > 이면 ` &='@ ` &>'

U 8 \ 의 값을 구하시오. ㄷ. `&"'8 `&,'8 `&)'8 `&:'8 `&/'7 -.(

① ㄱ ② ㄱ, ㄴ ③ ㄱ, ㄷ
④ ㄴ, ㄷ ⑤ ㄱ, ㄴ, ㄷ

- 11 -
SSEN 수학 (하) 12. 집합의 뜻과 표현

107. 0 7
1 . 실수 전체의 집합의 부분집합의 * 에 대하여 집합 6 가 110. 집합 * 7 0"4 (4 ,4 #4 -4 "(3에 대하여 다음 조건을 모두
1 0 .

6 7 0(1 2 1∈* 3이고, 양수 = 와 # 는 모두 집합 * 의 원소이면서 만족시키는 집합 6 의 개수를 구하시오.


동시에 집합 6 의 원소이다. ;&* '7 # 이고 집합 * 의 모든 원소의
합이 (:일 때, 집합 6 의 원소 중 가장 큰 수는 W 또는 X 또는 (가 ) 6 ⊂ *
a이다. 이때 W 8 X 8 a 의 값을 구하시오. (단, = ≠ # ) (나 ) 0,3⊂ 6
"(
(다 ) 1∈6이면 N ∈6 이다.
1

108. 0 8
1 . 집합 * 7 0"4 (4 ,4 #4 )4 -4 :4 .4 /3의 부분집합 A 에
대하여 집합 A 의 모든 원소의 합을 V &A '라 하자. 집합 A 가 다음
조건을 모두 만족시킬 때, V &A '의 최솟값을 구하시오.
111. 집합 0"4 (4 ,4 #4 )4 -4 :3의 부분집합 중에서 짝수인
1 1 .

(가) 0,4 )4 :3⊂ A 4 0(4 #4 )3⊄ A 원소가 O 개인 부분집합의 개수를 =O 라 할 때, =" 8 =( 8 =,의 값을
(나) V &A ' 의 값은 짝수이다. 구하시오.
(다) ;&A ' 7 -

109. 0 9
1 .
집합 V 7 012 1 는 "! 이하의 자연수 3의 부분집합 * 가
다음 조건을 모두 만족시킬 때, 집합 * 의 모든 원소의 곱의
최솟값을 구하시오.
112. 집합 V 7 0"4 (4 ,4 #4 )4 -4 :4 .3의 두 부분집합 b 4 K에
1 2 .

(가) =∈* 이면 "! 9 =∈* 이다. 대하여 b 의 가장 큰 원소가 K 에 속할 때, b ≪ K로 나타내기로


(나) 집합 * 의 모든 원소의 합은 (! 보다 크고 ,! 보다 작다. 하자. 집합 V 의 세 부분집합 * 7 0"4 (4 #34 6 7 0-4 .34 A 에
대하여 * ≪ A ≪ 6를 만족시키는 집합 A 의 개수를 구하시오.

- 12 -
SSEN 수학 (하) 13. 집합의 연산

2-2 집합의 연산 법칙

A단계 기본 다잡기
121. 세 집합 % & '& D 에 대하여
2 1 .
1

% ∩' ) *,& -.& % ∩D ) *-& 0.


2-1 합집합과 교집합

일 때, % ∩E'∪D F를 구하시오.

[ !""# ∼ !""$ ] 다음 두 집합 % & ' 에 대하여 % ∪' 를 구하시오.


113. 1 3
. % ) *+& ,& -.& ' ) *,& /& 0 .
122. 세 집합 % ) *"& ?.& ' ) *"& @& $.& D ) *#& $.에
2 2 .
1

대하여 E% ∪' F∩E% ∪D F 를 구하시오.

114. 1 4
. % ) *12 1 는 "! 보다 작은 자연수.
' ) *1 2 1는 "! 이상인 자연수.
2-3 여집합과 차집합

[ !"?# ∼ !"?@ ] 전체집합 G ) *1 2 1 는 9 이하의 자연수 . 의 두


부분집합 % & ' 가 다음과 같을 때, 각 집합의 여집합을 구하시오.
115. 1 5
. % ) ∅& ' ) *4& 5& 6& 7. 123. 2 3 .
1 % ) *#& @& $.

124. 2 4 .
1 ' ) *1 21 는 짝수.

[ !""8 ∼ !""9 ] 다음 두 집합 % & ' 에 대하여 % ∩' 를 구하시오.


116. 1 6
. % ) *1& ;& <.& ' ) *<& ;& 1. [ !"?$ ∼ !"?8 ] 다음 두 집합 % & '에 대하여 % B ' 를 구하시오.
125. 2 5 .
1 % ) *+& ,& -& /.& ' ) *+& /.

117. 1 7
. % ) *+& ,& -& /.& ' ) *0& =& >.

126. % ) *1 21 는 @의 양의 약수. ,
118.
2 6 .
1

1 8
. % ) *?& #& @& $.& ' ) *1 2 1 는 8 의 양의 약수. ' ) *1 2 1 는 "? 의 양의 약수.

[ !"?H ∼ !"#? ] 전체집합 G 의 두 부분집합 % &


' 에 대하여 오른쪽 벤다이어그램에서 다음
집합을 구하시오.
119. 1 9
. 두 집합 % & ' 에 대하여 % ) *"& #& $.& % ∩' ) *#.& 127. 2 7 .
1 %D
% ∪' ) *"& ?& #& @& $.일 때, 집합 ' 를 구하시오.
128. 2 8 .
1 'D

129. 2 9 .
1 %B'

120. 2 0
1 . 보기에서 두 집합 % & '가 서로소인 것만을 있는 대로 130. 3 0 .
1 'B%
고르시오.

131. 3 1 .
1
E% ∪' FD
ㄱ. % ) *"& ?& $.& ' ) ∅
ㄴ. % ) *1 2 1 는 A 의 양의 약수 . , ' ) *#& 8& A.
132. E% ∩' FD
ㄷ. % ) *1 2 B " ≤ 1 ≤ ".& ' ) *1 2 1 ) @.
3 2 .
1

- 1 -
SSEN 수학 (하) 13. 집합의 연산

141. 전체집합 G ) *"& ?& #& ⋯& "!.의 두 부분집합


2-4 집합의 연산의 성질

4 1 .
1

% ) *"& #& $& H& A.& ' ) *"& ?& #& 8.에 대하여 % C∩'C 를
구하시오.
[ !"## ∼ !"#9 ] 전체집합 G 의 부분집합 % 에 대하여 □ 안에 알맞은
집합을 써 넣으시오.
133. 3 3
1 . % ∩∅ ) □

134. 3 4
1 . % ∪∅ ) □

2-6 유한집합의 원소의 개수


135. 3 5
1 . % ∩% D ) □
142. 두 집합 % & ' 에 대하여 LE% F ) $& LE' F ) @&
4 2 .
1

LE% ∪' F ) H 일 때, LE% ∩' F 를 구하시오.

136. 3 6
1 . % ∪% D ) □

137. 3 7
1 . % ∩G ) □

[ !"@# ∼ !"@8 ] 전체집합 G 의 두 부분집합 % & ' 에 대하여


LEG F ) $!& LE% F ) #?& LE' F ) "9& LE% ∩' F ) 8 일 때, 다음을
138. 3 8
1 . % ∪G ) □
구하시오.
143. 4 3 .
1 LE% D F

139. 전체집합 G 의 서로 다른 두 부분집합 % & ' 에 대하여


144.
3 9
1 .

L E' B % F
% ⊂ ' 일 때, 다음 중 옳지 않은 것은?
4 4 .
1

① % ∪' ) ' ② % ∩' ) %


D D
③ E% ∩' F ⊂ ' ④ E'∩% F ⊂ %
D D 145. L E% ∩'D F
⑤ ' ⊂%
4 5 .
1

146. 4 6 .
1 LE% ∪' F

[ !"@H ∼ !"@9 ] 전체집합 G 의 두 부분집합 % & ' 에 대하여


LEG F ) $!& LE% F ) #$& LE' F ) ?#& LE% ∩' F ) "! 일 때, 다음을
구하시오.
2-5 드모르간의 법칙

147. 4 7 .
1 L E% D∩'D F

140. 4 0
1 . 오른쪽은 전체집합 G 의 두
부분집합 % & ' 에 대하여
C
148. 4 8 .
1 LE% D∪'D F
% ∩E% ∪' F ) ∅ 임을 보이는 과정이다.
(가), (나)에 사용된 법칙을 보기에서 각각
고르시오.

| 보 기 | 149. 전체집합 G 의 세 부분집합 M & N & O 에 대하여


4 9 .
1

ㄱ. 교환법칙 ㄴ. 결합법칙 LEM F ) ?!& LEN F ) $& LEO F ) "#& LEM ∩N F ) #&
ㄷ. 분배법칙 ㄹ. 드모르간의 법칙 LEN ∩O F ) ?& LEO ∩M F ) "!& LEM ∩N ∩O F ) ?
일 때, LEM ∪N ∪O F를 구하시오.

- 2 -
SSEN 수학 (하) 13. 집합의 연산

유형 02 서로소인 집합

B단계 유형 뽀개기
153. 다음 보기의 집합은 자연수 전체 집합의 부분집합이다. 이
5 3 .
1

중 집합 *?& @& 8.과 서로소인 집합의 개수는?


유형 01 합집합과 교집합

| 보 기 |

150. 5 0
1 . 세 집합
ㄱ. *1 21 는 짝수 . ㄴ. *1 21 는 홀수 .

% ) *1 2 1 는 "! 보다 작은 홀수인 자연수. ,


ㄷ. *1 2 1 는 #의 배수. ㄹ. *1 2 1? P ?1 ) !.
ㅁ. *1 21 는 "?의 약수. ㅂ. *1 2 1는 "$ 의 약수 .
' ) *1 2 1 는 "! 의 양의 약수. ,
D ) *1 2 1 는 @ 의 양의 약수. ① " ② ? ③ #
에 대하여 집합 E% ∪' F∩D는? ④ @ ⑤ $
① *"& ?. ② *#& $& H.
③ *"& ?& $& H. ④ *"& #& $& H.
⑤ *"& ?& #& @& $& H.

154. 집합 % ) *+& ,& -& /& 0 .의 부분집합 중에서 집합


5 4 .
1

' ) *+& ,.와 서로소인 집합의 개수를 구하시오.

151. 5 1
1 .
세 집합 % ) *?& #& @& $.& ' ) *@& 8& 9.&
D ) *1 2 1는 9의 양의 약수.에 대하여 다음 중 옳지 않은 것은?

① % ∩' ) *@.
② '∩D ) *@& 9.
③ % ∪D ) *"& ?& #& @& 8& 9.
④ E% ∩' F∪D ) *"& ?& @& 9.
⑤ % ∪'∪D ) *"& ?& #& @& $& 8& 9.
155. 두 집합 % ) *1 2 " ≤ 1 ≤ #.& ' ) *1 2 1 ≥ +.가 서로소일
5 5 .
1

때, 정수 + 의 최솟값을 구하시오.

152. 오른쪽 벤다이어그램과 같이


유형 03 여집합과 차집합

5 2
1 .

% ) *+& ,& -& /.& % ∩' ) *,& /.일 때, 다음


중 집합 ' 가 될 수 있는 것은?
① *+& ,& -. ② *+& ,& /.
156. 전체집합 G ) *"& ?& #& @& $& 8.의 두 부분집합
5 6 .
1

③ *,& -& 0 . ④ *,& /. % ) *1 2 1 는 8 의 약수. , ' ) *1 2 1 는 ? 의 배수.


D
⑤ */& 0 . 에 대하여 집합 % B ' 의 모든 원소의 합을 구하시오.

- 3 -
SSEN 수학 (하) 13. 집합의 연산

유형 04 조건을 만족시키는 집합 구하기


157. 5 7
1 . 전체집합 G ) *1 2 B $ ≤ 1 ≤ $.의 두 부분집합
% ) *1 2 B ? ≤ 1 R #.& ' ) *1 2 B ? R 1 ≤ #.
D
에 대하여 집합 E% ∩' F 는?
160. 전체집합 G ) *1 2 1 는 "! 보다 작은 자연수 .의 두 부분집합
6 0 .
1

① ∅ % & ' 에 대하여 % B ' ) *"& $& H.& % ∩' ) *#& A.&
② *B ?& #. E% ∪' FD ) *?& @.일 때, 집합 ' 는?
③ *1 2 B ? ≤ 1 ≤ #. ① *8. ② *#& A.
④ *1 2 B $ ≤ 1 ≤B ? 또는 # ≤ 1 ≤ $. ③ *#& @& A. ④ *#& 9& A.
⑤ *1 2 B $ ≤ 1 RB ? 또는 # R 1 ≤ $ . ⑤ *#& 8& 9& A.

161. 전체집합 G ) *1 2 1 는 8 이하의 자연수. 의 두 부분집합


6 1 .
1

% & ' 에 대하여 ' ) *1 2 1 는 8 의 약수. , % ∩' ) *1 2 1 는 # 의


배수., % ∪' ) G 일 때 , 집합 % 를 구하시오.

158. 5 8
1 . 전체집합 G 의 두 부분집합 % & ' 에
대하여 오른쪽 벤다이어그램에서 집합
E% ∪' F∩E% B ' FD 를 구하시오.

162. 전체집합 G 의 두 부분집합 % & '에 대하여


6 2 .
1

% ) *"& ?& #& @& $.& E% B ' F∪E' B % F ) *"& ?& 8& H.일 때,
집합 ' 의 모든 원소의 합을 구하시오.

159. 5 9
1 . 전체집합 G ) *"& ?& #& ⋯& "!.의 두 부분집합
% ) *1 2 1 ) ?S B "& S 는 자연수., 유형 05 벤다이어그램의 색칠한 부분이

' ) *1 2 1 ) #S B ?& S 는 자연수.


나타내는 집합
에 대하여 집합 E% ∪' F B E% ∩' F 를 구하시오.

163. 다음 중 오른쪽 벤다이어그램의 색칠한


6 3 .
1

부분을 나타내는 집합과 항상 같은 집합은?


① M ∩ EN ∪O F ② M ∪ EN ∩O F
③ M B EN ∩O F ④ M B EN B O F
⑤ M ∩ EN B O F

- 4 -
SSEN 수학 (하) 13. 집합의 연산

유형 07 집합의 연산의 성질

164. 6 4
1 . 다음 중 오른쪽 벤다이어그램의
색칠한 부분을 나타내는 집합과 항상 같은
집합은? 168. 전체집합 G 의 두 부분집합 % & ' 에 대하여 다음 중 항상
6 8 .
1

① % ∩' D
② % ∪' D 옳은 것은?
D D
③ E% B ' F
D
④ E% ∪' F B ' ① GB% )' ② % ∩% ) ∅
D
⑤ G B E% ∩' F ③ G ⊂ E% ∪' F ④ %⊂G
⑤ G ∩' D ) %

유형 06 집합의 연산; 미지수 구하기


165. 6 5
1 .
?
두 집합 % ) *B "& !& + P + B @ . ,

' ) *?& B + P #& +? B + B # . 에 대하여 % ∩' ) *B "& ? . 일 때,


상수 +의 값을 구하시오.
169. 전체집합 G 의 두 부분집합 % & ' 에 대하여 다음 중 항상
6 9 .
1

옳은 것은?
DD
① E% F ) G B % ② % ∪∅ ) ∅
D D
③ % ∪% ) % ④ % ∩' ) % B '
⑤ % ∪EG ∩' F ) % ∪'

166. 6 6
1 .
두 집합 % ) *"& @& $& #+ B ,.& ' ) *"& H& + B ?,. 에
대하여 % B ' ) *$. 일 때, + P ,의 값을 구하시오.
(단, +& ,는 상수이다.)

170. 전체집합 G 의 공집합이 아닌 서로 다른 두 부분집합


7 0 .
1

% & ' 에 대하여 다음 중 나머지 넷과 다른 하나는?


D D
① %B' ② % ∩EG B ' F
D D
③ 'B% ④ % ∩ E' ∪' F
167. ?
두 집합 % ) *#& B + P ?& + B ?.& ' ) *?& #& + B ".에
⑤ E% ∩' F∪E% ∩% D F
6 7
1 .

대하여 % ∪' ) *!& "& ?& #. 일 때, 집합 % 의 모든 원소의 합을


구하시오.
(단, +는 상수이다.)

- 5 -
SSEN 수학 (하) 13. 집합의 연산

유형 08 집합의 연산의 성질 ☯

174. 전체집합 G 의 두 부분집합 % & ' 에 대하여


7 4 .
1

; 포함 관계가 있는 두 집합 E% ∩' F∪E'∩% D F ) ∅ 일 때, 다음 중 항상 옳은 것은?


D

D D D
① %)' ② % )∅ ③ % ∩' ) ∅
171. 전체집합 G 의 서로 다른 두 부분집합 % & ' 에 대하여 D
④ % ∪' ) G ⑤ % ∩' ) %
7 1
1 .

% ∪' ) % 일 때, 다음 중 옳지 않은 것은?
D D D D D
① % ∪' ) ' ② ' ⊂% ③ % ∩' ) '
D
④ %B ')∅ ⑤ '∩% ) ∅

유형 09 조건을 만족시키는 부분집합의


172. 전체집합 G 의 서로 다른 두 부분집합 % & ' 에 대하여 개수


175. 전체집합 G ) *B ?& B "& !& "& ?& #. 의 세 부분집합
7 2
1 .

% ⊂ 'D 일 때, 다음 중 나머지 넷과 다른 하나는?


D 7 5 .
1

① % ∪' ② % ∩ E% ∪' F % & '& M 에 대하여 % ) *B ?& !& ".& ' ) *B ?& B "& !. 일 때,
③ E% ∩' F∪' ④ % ∪ E' B % F E' B % F∪M ) M & % ∪M ) M 를 만족시키는 집합 M 의 개수를
구하시오.
⑤ E% ∪' F∪ E% ∩' F

173. 7 3
1 . 전체집합 G 의 두 부분집합 % & ' 에 대하여 % B ' ) % 일
때, 항상 옳은 것만을 보기에서 있는 대로 고른 것은?
176. 전체집합 G ) *B #& B ?& B "& !& "& ?& #. 의 두
7 6 .
1

| 보 기 | 부분집합 % & ' 에 대하여 % ) *B ?& !& ? . 일 때, % ∪' ) G 를


ㄱ. % ∩' ) ∅ ㄴ. ' ⊂ % 만족시키는 집합 ' 의 개수는?
D
ㄷ. ' B % ) ' ㄹ. ' ⊂ % ① 9 ② "8 ③ #?
④ 8@ ⑤ "?9
① ㄱ, ㄷ ②ㄱ, ㄹ ③ ㄴ, ㄷ
④ ㄷ, ㄹ ⑤ ㄱ, ㄷ, ㄹ

- 6 -
SSEN 수학 (하) 13. 집합의 연산

177. 7 7
1 . 전체집합 G ) *+& ,& -& /& 0& =. 의 두 부분집합 181. 전체집합 G ) *1 2 1는 9 이하의 자연수 . 의 두 부분집합
8 1 .
1

% & M 에 대하여 % ) *+& / . 일 때, % B M ) % 를 만족시키는 집합 % & ' 에 대하여


M 의 개수를 구하시오. % B ' ) *"& 9.& % ∩' ) *?& @.& % D∩'D ) *#& $. 일 때,
집합 ' 를 구하시오.

178. 7 8
1 .
전체집합 G ) *1 2 1는 A 이하의 자연수 . 의 세 부분집합
% ) *"& ?& #& @.& ' ) *#& @& $& 8.& D ) *8& H.에 대하여
M B E% ∩' F ) M ∪E'∩D F 를 만족시키는 집합 G 의 부분집합
M 의 개수는?
① 9 ② "8 ③ #?
④ 8@ ⑤ "?9 182. 전체집합 G ) *1 2 1는 "! 이하의 자연수 . 의 세 부분집합
8 2 .
1

% & '& D 에 대하여 % ) *"& ?& @& $& 9& A. ,


E% B ' F∪E% B D F ) *"& @& $& 9. 일 때, 다음 중 반드시 집합
'∩D 의 원소인 것은?
① " ② ? ③ #
④ @ ⑤ $

179. 7 9
1 . 전체집합 G ) *"& ?& #& @& $& 8& H . 의 두 부분집합
% ) *?& @ .& ' ) *"& ?& #& 8 . 에 대하여
% ∪M ) M & E' B % F∩M ) *"& 8 . 을 만족시키는 집합 G 의
부분집합 M 의 개수를 구하시오.

183. 전체집합 G ) *"& ?& #& @& $ . 의 두 부분집합 % & ' 에


8 3 .
1

대하여
% D∪'D ) *"& ?& #& $.
E' B % FD∩*% ∩E% ∩' FD . ) *".
일 때, 집합 % 의 모든 원소의 합을 구하시오.

유형 10 드모르간의 법칙

180. 8 0
1 . 전체집합 G ) *"& ?& #& ⋯& A . 의 두 부분집합
% ) *"& #& $& H.& ' ) *"& @& 8& H.에 대하여 집합
E% ∪' F∪E% D∪'D FD 의 원소의 개수를 구하시오.

- 7 -
SSEN 수학 (하) 13. 집합의 연산

유형 11 항상 같은 집합 구하기

187. 전체집합 G 의 공집합이 아닌 두 부분집합 % & '에 대하여


8 7 .
1

% B ' ) ∅ 일 때, 다음 중 집합 E% ∪'D F∩E% D∪' F 와 항상 같은


184. 8 4
1 .
전체집합 G 의 두 부분집합 M & N 에 대하여 다음 중 집합 집합은?
EM ∩N F∪N 와 항상 같은 집합은? ① ∅ ② % ∪'D ③ % D∪'
D
① EM B N F∪N ② EN B M F ∩N ④ % ∩' ⑤ 'B%
③ M B EM ∩N F ④ M B EN B M F
⑤ N B EM B N F

185. 8 5
1 . 전체집합 G 의 세 부분집합 % & '& D 에 대하여 다음 중 188. 전체집합 G 의 공집합이 아닌 세 부분집합 M & N & O
8 8 .
1

집합 E% B ' F B E% B D F 와 항상 같은 집합은? 사이에 EM ∪N F ⊂ O 인 포함 관계가 성립할 때 , 다음 중 집합


① E% ∩' F B D ② E% ∪D F B ' EO B M FD∩N 와 항상 같은 집합은?
③ % B E'∩D F ④ % B E'∪D F ① EO ∩M F∪EO ∩N F
D
② EO ∪M F ∩EO ∪N F
D

⑤ E% ∩D F B ' D D
③ EO B M F∩ EO B N F ④ EM B O F∩ EN B O F
D D
⑤ EO ∩M F∪EO ∩N F

186. 전체집합 G 의 세 부분집합 % & '& D 에 대하여 항상 옳은


유형 12 두 집합의 포함관계

8 6
1 .

것만을 보기에서 있는 대로 고르시오.

| 보 기 | 189. 전체집합 G 의 두 부분집합 % & ' 에 대하여


8 9 .
1

D
ㄱ. E% ∪' F∪E% ∩' F ) ∅D *E% ∩' F∪E% B ' F.∩' ) %
ㄴ. % B E'∩D F ) E% B ' F∪E% B D F 일 때, 다음 중 항상 성립하는 것은 ?
ㄷ. E% ∩' F B E% ∩D F ) E% B ' F B D ① % ∪' ) ∅ ② %⊂' ③ '⊂%
ㄹ. E% B ' F∪E% ∩D F ) % B E' B D F ④ %)' ⑤ % ∪' ) G

- 8 -
SSEN 수학 (하) 13. 집합의 연산

190. 9 0
1 . 전체집합 G 의 서로 다른 두 부분집합 % & ' 에 대하여 192. 자연수 L 의 양의 배수의 집합을 % L이라 할 때, 다음 중
9 2 .
1

D D
E% B ' F ∩' ) ' D 집합 E% @∪% 9 F∩ E% #∪% "? F 와 같은 집합은?
일 때, 두 집합 % & ' 의 포함 관계를 벤다이어그램으로 바르게 ① %? ② %8 ③ %9
나타낸 것은? ④ % "? ⑤ % "8
① ②

③ ④

193. 자연수 L 의 양의 약수의 집합을 % L이라 할 때, 다음 중


9 3 .
1

집합 % "?∩% "9∩% ?@ 에 속하는 원소가 아닌 것은?


① " ② ? ③ #
④ @ ⑤ 8

유형 13 배수와 약수의 집합의 연산


191. 9 1
1 . 전체집합 G ) *"& ?& #& ⋯& "!! . 의 부분집합 % S를
194. 두 집합 % T& 'L에 대하여
9 4 .
1

% T ) *1 2 1 는 T 의 양의 배수, T은 자연수.
% S ) *1 2 1 는 S 의 배수, S 는 자연수.
'L ) *1 2 1 는 L 의 양의 약수, L은 자연수.
라 할 때 , 집합 % #∩ E% @∪% 8 F의 원소의 개수를 구하시오.
라 하자. % 4 ⊂ E% 8∩% A F 를 만족시키는 자연수 4 의 최솟값과
'5 ⊂ E'"?∩'"9 F 을 만족시키는 자연수 5 의 최댓값의 합을
구하시오.

- 9 -
SSEN 수학 (하) 13. 집합의 연산

유형 14 방정식 또는 부등식의 해의 유형 15 새롭게 약속된 집합의 연산


집합의 연산
199. 전체집합 G 의 두 부분집합 % & ' 에 대하여 연산 ◇를
9 9 .
1

195. 9 5
1 . 두 집합 % ◇' ) E% B 'F∪E' B % F
% ) *1 2 1? B #1 B @ ≤ ! .& ' ) *1 2 1? P 41 P 5 ≤ ! .에 대하여 로 약속할 때, 다음 중 옳지 않은 것은?
-
% ∩' ) *1 2 ? ≤ 1 ≤ @ . , ① G ◇% ) % ② ∅◇% ) %
% ∪' ) *1 2 B " ≤ 1 ≤ $. ③ % ◇% ) % ④ G ◇∅ ) G
일 때, 5 B 4 의 값을 구하시오. (단, 4& 5 는 상수이다.) ⑤ % ◇' ) ' ◇%

196. 9 6
1 . 두 집합

% ) *1 2 1? B ?1 P + ) !.& ' ) *1 2 1# P ,1 P "? ) !.에 대하여


% ∩' ) *#. 일 때, 집합 % ∪' 를 구하시오.
(단, +& ,는 상수이다.)
200. 전체집합 G 의 두 부분집합 % & ' 에 대하여 연산 ◎를
0 0 .
2

% ◎' ) E% ∪'F∩E% ∩'F-


로 약속할 때, 다음 중 벤다이어그램의 색칠한 부분이 나타내는
집합이 % ◎E' ◎DF 인 것은?

① ②

197. 9 7
1 . 두 집합 % ) *1 2 E1 B @FE1 B ?!F ≥ !. ,

' ) *1 2 E1 B UV
S FE1 B SF ≤ !. 에 대하여 % ∩' ) ∅ 이 되도록
③ ④
하는 자연수 S 의 개수는?
① " ② ? ③ #
④ @ ⑤ $

198. 9 8
1 . 두 집합 % ) *1W1 B 1 B 8 R !.&
?

' ) *1W1? B ?E+ B "F1 B @+ R !.에 대하여 % ∩' ) % 일 때,


실수 +의 최솟값을 구하시오.

- 10 -
SSEN 수학 (하) 13. 집합의 연산

201. 0 1
2 . 전체집합 G 의 두 부분집합 % & ' 에 대하여 연산 ▷ 를 205. 전체집합 G 의 두 부분집합 % & ' 에 대하여
0 5 .
2

% ▷' ) E% ∪'F∩E% ∪'F -


LEG F) #!& LE% -∪'-F) ?$& LE% -F) "8
로 약속할 때, 다음 중 E% ▷'F▷' 와 항상 같은 집합은? 일 때, LE% ∩' F를 구하시오.
-

① % ② ' ③ % ∩'
④ % ∪ ' ⑤ % B '

유형 16 유한집합의 원소의 개수

206. 세 집합 % & '& D 에 대하여 % 와 D 가 서로소이고


0 6 .
2

LE% F) 8& LE'F) $& LEDF) @& LE% ∪'F) A& LE'∪DF) 8일 때,
202. 0 2
2 . 전체집합 G 의 두 부분집합 % & ' 에 대하여 LE% ∪'∪DF 는?
LEG F) #!& LE% F) ?!& LE'F) "$& LE% -∩'-F) @ ① A ② "! ③ ""
일 때, LE% ∩'F 는? ④ "? ⑤ "#
① # ② $ ③ H
④ A ⑤ ""

유형 17 유한집합의 원소의 개수의


최댓값과 최솟값
203. 0 3
2 . 두 집합 % & ' 에 대하여
LE% F) ?!& LE'F) "!& LE% B 'F) "$ 207. 전체집합 G 의 두 부분집합 M & N 에 대하여 LEG F) ?!&
0 7 .
2

일 때, LE% ∪'F 를 구하시오.


LEM F) "@& LEN F) 9 일 때, LEM ∩N F 의 최댓값 Y 과 최솟값 T 에
대하여 Y B T 의 값은?
① # ② @ ③ $
④ 8 ⑤ H

204. 0 4
2 . 전체집합 G 의 두 부분집합 % & ' 에 대하여 % ⊂ ' 이고
-

LE% F) H& LE% ∪'F) "# 일 때, LE' B % F를 구하시오.


208. 두 집합 % & ' 에 대하여 LE% F) 8& LE'F) A&
0 8 .
2

LE% ∩'F≥ #
일 때, LE% ∪'F의 최댓값과 최솟값의 합을 구하시오.

- 11 -
SSEN 수학 (하) 13. 집합의 연산

유형 18 유한집합의 원소의 개수의 활용


212. 1 2 .
2 @!명의 학생을 대상으로 세 개의 수학 문제 A & B& C 를
풀게 하였더니 A 문제, B 문제, C 문제를 맞힌 학생은 각각 "8 명,
209. 0 9
2 .
어느 학급의 학생 @!명 중에서 A 은행의 통장을 갖고 있는 ?! 명, ?? 명이고, 세 문제를 모두 맞힌 학생은 #명이었다. 한 문제도
학생이 ?9 명, B은행의 통장을 갖고 있는 학생이 "8 명, A 은행과 맞히지 못한 학생은 없다고 할 때, 세 문제 중 두 문제만 맞힌 학생
B은행의 통장 중 어느 것도 갖고 있지 않은 학생이 @ 명일 때, 수를 구하시오.
A 은행과 B은행의 통장을 모두 갖고 있는 학생 수를 구하시오.

유형 19 유한집합의 원소의 개수의 활용


; 최댓값과 최솟값

210. 1 0
2 . 어느 운동 동호회 회원 $!명을 대상으로 야구와 축구의 213. 어느 화장품 가게를 방문한 고객 #$명 중에서 반지를
1 3 .
2

선호도를 조사하였다. 그 결과 야구를 좋아하는 회원이 ?@명, 축구를 착용한 고객이 "?명, 목걸이를 착용한 고객이 "9 명, 반지와 목걸이

좋아하는 회원이 #?명 , 야구와 축구 중 어느 것도 좋아하지 않는 중 어느 것도 착용하지 않은 고객이 + 명이다. 이때 +의 최댓값

회원이 $ 명이었다. 이때 축구만 좋아하는 회원 수를 구하시오. Y 과 최솟값 T 에 대하여 Y P T 의 값을 구하시오.

214. 영지네 반 학생 #$명 중에서 영어를 좋아하는 학생은 ?" 명,


1 4 .
2

수학을 좋아하는 학생은 ?H 명이다. 이때 영어와 수학을 모두


좋아하는 학생 수의 최댓값과 최솟값을 구하시오.

211. 1 1
2 . 다음은 학생 수가 $!인 어느 학급에서 인터넷을 이용하여
수학과 과학 강의를 수강하는 학생 수에 대한 설명이다.

(가) 수학 강의를 수강하는 학생은 #?명이다.


(나) 수학 강의를 수강하지만 과학 강의는 수강하지 않는
학생은 "#명이다.
(다) 두 과목 모두 수강하지 않는 학생은 "! 명이다.
215. 1 5 .
2 @!명의 주부를 대상으로 A 통조림과 B 통조림을 구입해 본
경험을 조사하였더니 A 통조림과 B통조림을 구입해 본 주부가 각각
과학 강의를 수강하는 학생 수는?
?# 명, ?H 명이었다. B 통조림만 구입해 본 주부 수의 최댓값은?
① ?# ② ?@ ③ ?$
① @ ② "! ③ "#
④ ?8 ⑤ ?H
④ "H ⑤ ?"

- 12 -
SSEN 수학 (하) 13. 집합의 연산

219.

C단계 실력 굳히기 두 집합
1 9 .
2

% ) *1 W 1 는 "!! 이하의 자연수 .


' ) *1 W 1 는 $!과 서로소인 자연수 .
216. 1 6
2 . 자연수 L 에 대하여 에 대하여 다음 조건을 만족시키는 집합 M 의 개수를 구하시오.
% L ) *1W?L B # ≤ 1 ≤ #L P "!& 1 는 정수 .
라 하자. % "∩% ?∩% #∩ ⋯ ∩% T ≠ ∅ 을 만족시키는 자연수 T 의 (가 ) M ⊂ % & M ≠ ∅
최댓값을 4 라 하고, *"& ?& #& ⋯& $!.⊂ E% "∪% 9∪% ]F 을 (나 ) M ∩' ) ∅
(다 ) 집합 M 의 모든 원소는 "? 와 서로소이다.
만족시키는 자연수 ] 의 최댓값을 5 라 할 때, 4 P 5의 값을 구하시오.

217. 1 7
2 .
두 집합 % ) *B "& ?.& ' ) *1WT1 P " ) 1.에 대하여 220. 전체집합 G 의 공집합이 아닌 세 부분집합 % & '& D 에
2 0 .

-
% ∪' ) % 를 만족시키는 모든 실수 T 의 값의 합은? 대하여 % 와 ' 가 서로소이고, % 와 D 가 서로소일 때, 항상
$ " 성립하는 것만을 보기에서 있는 대로 고른 것은?
① BV ② B" ③ V
? ?
| 보 기 |
H
④ ? ⑤ V ㄱ. '∩D ) ∅
?
-
ㄴ. E% ∩'F ∩D ) D
ㄷ. E' B % F∪E' B DF) '

① ㄱ ② ㄴ ③ ㄱ, ㄴ
④ ㄱ, ㄷ ⑤ ㄴ, ㄷ

218. 1 8
2 . 두 집합
% ) *1 W 1 는 "8의 양의 약수 . 221. 전체집합 G ) *"& ?& #& @& $& 8.의 두 부분집합 % & ' 에
2 1 .

' ) *1 W 1 는 ?@의 양의 약수 . - -
대하여 연산 *를 % *' ) E% B 'F ∩E' B % F 로 약속할 때, 옳은
에 대하여 E% ∩'FB M ) ∅& E% ∪'F∩M ) M 를 만족시키는 집합
것만을 보기에서 있는 대로 고른 것은?
M 의 모든 원소의 합이 #! 일 때, 집합 M 의 원소의 개수는?
① @ ② $ ③ 8 | 보 기 |
④ H ⑤ 9 ㄱ. *"& ?.**#& @.) *?& @& $& 8.
ㄴ. % *' ) % -*' -
ㄷ. % *' ) ∅ 을 만족시키는 두 집합 % & ' 의 순서쌍
E% & 'F 의 개수는 8@이다.

① ㄱ ② ㄱ, ㄴ ③ ㄱ, ㄷ
④ ㄴ, ㄷ ⑤ ㄱ, ㄴ, ㄷ

- 13 -
SSEN 수학 (하) 13. 집합의 연산

222. 2 2
. 전체집합 G ) *1 W 1 는 자연수 .의 두 부분집합 225. 전체집합 G 의 부분집합 % 에 대하여 % 의 부분집합의
2 5 .

% ) *+& ,& -.& ' ) *UV


+ & UV
, & UV
-. 개수를 7 E% F 라 하자. G ) *+& ,& -& /& 0 .의 두 부분집합
가 다음 조건을 모두 만족시킬 때, 집합 ' 의 모든 원소의 합은? % & ' 에 대하여 옳은 것만을 보기에서 있는 대로 고른 것은?
| 보 기 |
(가) LE% ∪'F) $ -
ㄱ. 7 E% F ) @ 이면 7 E% F ) 9
(나) + R , R -& + P - ) $# - -
ㄴ. % ⊂ ' 이면 7E' F ≤ 7E% F
ㄷ. % ∩' ) ∅ 이면 7E% ∪' F ) 7E% F P 7E' F
① "" ② "? ③ "#
④ "@ ⑤ "$ ① ㄱ ② ㄴ ③ ㄷ
④ ㄱ, ㄴ ⑤ ㄱ, ㄴ, ㄷ

223. 2 3
. 전체집합 G ) *1 2 1 는 "$ 이하의 자연수 .의 공집합이
아닌 두 부분집합 % ) *1 2 + ≤ 1 ≤ ,., ' ) *1 2 - ≤ 1 ≤ /.에
대하여
"
L E% ∩' F ) L E% ∩'- F ) V × L E% -∩' F
?
- -
가 성립할 때, 집합 % ∩' 의 모든 원소의 합을 _ 라 하자.
_ 의 값이 최소일 때, + P , P - P / 의 값을 구하시오. (단, 226. 어느 회사의 사원들을 대상으로 간기능, 심혈관질환 ,
2 6 .

+& ,& -& / 는 "$ 이하의 자연수, + R -) 신장기능의 세 항목에 대하여 건강검진을 실시한 결과, 적어도 한
항목에서 정상 수치를 초과한 사람이 "$ 명이었다. 간기능,
심혈관질환 , 신장기능의 정상 수치의 최댓값을 각각 +& ,& - 라 할
때, 다음은 ? 개 이상의 항목에서 정상 수치를 초과한 사람의
명단이다.

+& ,& - 모두 초과한 사람 김○○


+& , 만 초과한 사람 이○○, 박○○, 정○○
,& - 만 초과한 사람 최○○
+& - 만 초과한 사람 권○○, 황○○

+ 또는 , 를 초과한 사람이 "? 명, , 또는 - 를 초과한 사람이


"? 명일 때, + 또는 - 를 초과한 사람의 수를 구하시오.
224. 2 4
. 집합 G ) *+& ,& -& /.에 대하여
% ) *M 2 +∈M & M ⊂ G ., ' ) *N 2 ,∈N & N ⊂ G .
일 때, LE% ∪' F 는?
① "! ② "" ③ "?
④ "# ⑤ "@

- 14 -
SSEN 수학 (하) 13. 집합의 연산

227. 2 7
. 모든 자연수 L 에 대하여 230. 전체집합 G ) *B @& B #& B ?& B "& !& "& ?& #& @.의
3 0 .
2

L
% ) *1 2 1 는 9 B "을 "!으로 나누었을 때의 나머지 ., 두 부분집합 % & ' 가 다음 조건을 모두 만족시킨다.
' ) *1 2 1 는 HL을 "!으로 나누었을 때의 나머지 .
- -
일 때, 집합 % ∩' 의 부분집합의 개수를 구하시오. (가 ) % ∩' ) *B #& @.
(나 ) LE% F ) ? × LE' F

집합 M 의 모든 원소의 합을 _ EM F 라 할 때, _ E% ∩' F ) ! ,
_ E% F R _ E' F 를 만족시키는 두 집합 % & ' 의 순서쌍 E% & ' F 의
개수를 구하시오.

228. 전체집합 G 의 두 부분집합 % & ' 에 대하여


231. 두 집합 M & N 에 대하여 연산 ∆를
2 8
.

3 1 .
2

-
% ∪' ) *#& 9& A& ""., % ∪' ) *#& @& 8& 9& A.이고, #∈% 일
M ∆N ) EM B N F∪EN B M F
때, 집합 % ∩' 의 원소의 합의 최댓값을 Y , 최솟값을 T 이라
로 약속하자. 세 집합 % & '& D 가 L E% ∪'∪D F ) 8$ ,
하자. Y P T 의 값을 구하시오.
LE% ∆' F ) #8 , LE'∆D F ) #9 , LED∆% F ) #? 를 만족시킬 때,
L E% ∩'∩D F 를 구하시오.

229. 집합 % L 이
232.
2 9
.

다음은 어느 고등학교 "학년 학생 "?! 명을 대상으로


% L ) *1 2 1 는 L 의 양의 배수 . ( L ) "& ?& #& ⋯ )
3 2 .
2

인문학 특강과 자연과학 특강 신청을 발은 결과이다.


일 때, 다음 조건을 모두 만족시키는 $! 이하의 자연수 L 의 개수를
구하시오.
(가 ) 인문학 특강을 신청한 학생은 자연과학 특강을 신청한
학생보다 "8 명이 적다.
(가) % L∩% ? ) % ?L
(나 ) 적어도 하나의 특강을 신청한 학생은 어떤 특강도
(나) E% L B % # F ⊂ E% L B % ? F
신청하지 않은 학생보다 9!명이 많다.

자연과학 특강만 신청한 학생 수의 최댓값을 구하시오.

- 15 -
SSEN 수학 (하) 13. 집합의 연산

- 16 -
SSEN 수학 (하) 14. 명제

[!"."∼!".# ] 전체집합 5 & 6( 7 ( 는 )! 이하의 자연수 8에 대하여


다음 조건의 부정을 말하고, 그것의 진리집합을 구하시오.

A단계 기본 다잡기 242. 4 2 .


2 2 3 ( 는 ' 의 약수이다.

14-1 명제

243. 4 3 .
2 4 3 (" % $( 1 , & !
[!"##∼!"#$ ] 다음 식 또는 문장이 명제이면 ○를, 명제가 아니면
×를 ( ) 안에 써넣으시오.
[!"..∼!".$ ] 실수 전체의 집합에서 다음 조건의 부정을 말하시오.
233. #%$&' ( )
244.
3 3
2 .

4 4 .
2 ( & ) 또는 ( & "

234. "( % ) * + ( )
245.
3 4
2 .

4 5 .
2
%"≤(: #

235. 3 5
2 . 이 꽃은 예쁘다. ( )

14-3 명제 2 → 4 의 참, 거짓

[!"#,∼!"#- ] 다음 명제의 참 , 거짓을 판별하시오.


236. 3 6
2 . ' 의 배수이면 . 의 배수이다. [!".,∼!".' ] 다음 명제의 가정과 결론을 말하시오.
246. 4 6 .
2 )'의 약수이면 + 의 약수이다.

237. 3 7
2 .
/0
$ 1 /0
" & /0
-

247. 4 7 .
2 ( &% ) 이면 #( % " &% ) 이다.

14-2 조건과 진리집합


248. 4 8 .
2 <= & ! 이면 < & ! 또는 = & ! 이다.
[!"#'∼!"#+ ] 정수 전체의 집합에서 다음 조건의 진리집합을
구하시오.
238. 3 8
2 . 2 3 (" 1 "( % # & !
[!".+∼!"$" ] 다음 명제의 참, 거짓을 판별하시오.
249. 4 9 .
2 ( 가 실수이면 (" * !이다.

239. 3 9
2 . 4 3 ( 는 )! 보다 작은 소수이다.

250. 자연수 (> ? 에 대하여 (? 가 홀수이면 (> ? 는 모두


5 0 .
2

홀수이다.
[!".!∼!".) ] 다음 명제의 부정을 말하고 , 그것의 참, 거짓을
판별하시오.
240. 4 0
2 .
/0
. 는 무리수이다.

251. 자연수 (> ? 에 대하여 ( 1 ? 가 짝수이면 ( 또는 ? 가


5 1 .
2

짝수이다.
241. 4 1
2 . ) 은 합성수도 아니고, 소수도 아니다.

252. " "


실수 (> ? 에 대하여 (? ≠ ! 이면 ( 1 ? ≠ ! 이다.
5 2 .
2

- 1 -
SSEN 수학 (하) 14. 명제

14-4 ‘모든’이나 ‘어떤’이 들어 있는 명제


264. 6 4 .
2 (" 1 ?" & ! 이면 ( & !이고 ? & ! 이다.

[!"$#∼!"$, ] 다음 명제의 참 , 거짓을 판별하시오.


253. 5 3
2 . 모든 실수 ( 에 대하여 A(A* ! 이다.

265. 6 5 .
2 < 1 = * ! 이면 < * ! 또는 = * !이다.

254. 5 4
2 .
"
모든 실수 ( 에 대하여 ( 1 ( 1 ) * ! 이다.

266. 명제 ‘(? ≠ ! 이면 ( ≠ ! 이고 ? ≠ !이다.’에 대하여 다음에


6 6 .
2

답하시오.
(1) 명제의 대우를 말하시오.
255. 5 5
2 .
"
어떤 실수 ( 에 대하여 ( ≤ ! 이다. (2) 명제의 대우의 참, 거짓을 판별하시오.
(3) 명제의 참, 거짓을 판별하시오 .

256. 5 6
2 . 어떤 실수 ( 에 대하여 /0
( : ! 이다.

14-6 충분조건과 필요조건


[!"$-∼!",! ] 다음 명제의 부정을 말하시오.


[ !",- ∼ !"-) ] 두 조건 2> 4 가 다음과 같을 때, 2 는 4 이기 위한
257. 5 7
2 . 모든 실수 ( 에 대하여 (" & . 이다. 어떤 조건인지 말하시오.

267. 6 7 .
2 2 3 (&" 4 3 (" & .

258. 5 8
2 . 모든 실수 ( 에 대하여 (" 1 ) ≥ ! 이다.

268. 6 8 .
2 2 3 %)≤(≤) 4 3 A ( A: )

259. "
어떤 실수 ( 에 대하여 ( * . 이다.
269.
5 9
2 .

6 9 .
2 2 3 ( * !> ? * ! 4 3 ( 1 ? * ! > (? * !
(단, (> ? 는 실수이다.)

260. "
어떤 실수 ( 에 대하여 ( 1 ) & ! 이다.
270.
6 0
2 .

7 0 .
2 2 3 ( 는 )" 의 양의 약수, 4 3 ( 는 , 의 양의 약수

14-5 명제의 역과 대우 271.


7 1 .
2 2 3 ( 는 정수 4 3 ( 는 유리수

261. 6 1
2 . 다음 안에 역, 대우 중 알맞은 것을 써넣으시오.
(1) 명제 ∼ 2 → 4 는 명제 4 → ∼ 2 의 이다.
(2) 명제 4 → ∼ 2 는 명제 2 → ∼ 4 의 이다.
272. 7 2 .
2 < > = 가 실수일 때, 다음 조건은 <" 1 =" & ! 이기 위한 어떤
조건인지 말하시오.
D)E <= & ! D"E< 1 = & ! D#E A < A1 A =A& !
[!","∼!",$ ] 다음 명제의 역 , 대우를 말하시오.
262. 6 2
2 . (" & ) 이면 ( & )이다.

263. 6 3
2 . ( * "이면 ( * # 이다.

- 2 -
SSEN 수학 (하) 14. 명제

276. 다음은 < * ! > = * ! 일 때 , 부등식


14-7 여러 가지 증명법

7 6 .
2

/0
< 1 /0
= * /0
< 1 = 가 성립함을 증명하는 과정이다.

| 증 명 |
273. 7 3
2 . 두 조건 2 > 4 에 대하여 명제 2 → ∼ 4 가 참일 때, 다음 D/0< 1 /0= E" % D/0 "
< 1 = E & (가) * !
중 반드시 참인 명제는?
∴ D/0 < 1 /0= E
"
(나) D/0<1 = E
"

① 2→4 ② 4→2 ③ 4 → ∼2
그런데 /0 < 1 /0 = * !> /0 < 1 = * ! 이므로
④ ∼4 → 2 ⑤ ∼4 → ∼ 2
/0
< 1 /0
= (다) /0
<1 =

위의 과정에서 (가), (나), (다 )에 알맞은 것을 써넣으시오.

274. 7 4
2 . 다음은 명제 ‘실수 <> = 에 대하여 < 1 = : ! 이면 <> = 중
14-9 여러 가지 절대부등식

적어도 하나는 음수이다.’가 참임을 증명하는 과정이다.

| 증 명 |
<> = 모두 음이 아닌 실수, 즉
277. <> = 가 실수일 때, 부등식 <" 1 <= 1 =" ≥ ! 이 성립함을
< D가E !> = D가E !
7 7 .
2

증명하시오.
이라 가정하면
< 1 = D나E !
따라서 < 1 = : ! 이라는 가정에 모순이므로 < 1 = : ! 이면 <> =
중 적어도 하는 음수이다.
<1 =
위의 과정에서 (가), (나 )에 알맞은 것을 써넣으시오. 278. 7 8 .
2 <> = 가 양수일 때, 부등식 0 ≥ /0
"
<= 가 성립함을

증명하시오.

G "-+ ∼ "'! H ( * ! 일 때 다음 식의 최솟값을 구하시오.

)
279. (1 0
14-8 절대부등식 (

7 9 .
2

+
280. 8 0 .
2 .( 1 0
(
275. 7 5
2 .
" "
다음은 실수 < > = 에 대하여 부등식 < 1 "= ≥ "<= 가
성립함을 증명하는 과정이다.
281. 다음은 <> => (> ? 가 실수일 때, 부등식
8 1 .
2

| 증 명 |
D< 1 = ED(" 1 ?"E≥ D<( 1 =?E" 이 성립함을
" "
증명하는 과정이다.
<" 1 "=" % "<= & ( (가) " "
) 1= ≥ !
| 증 명 |
∴ <" 1 "=" (나) "<=
(다)
D<" 1 ="ED(" 1 ?"E% D<( 1 =?E"
이때 등호는 일 때 성립한다.
& <"(" 1 <"?" 1 ="(" 1 ="?" % D<"(" 1 "<=(? 1 ="?"E
위의 과정에서 (가), (나 ), (다 )에 알맞은 것을 써넣으시오. & ="(" % "<=(? 1 <"?" & D D가E E" ≥ !
∴ D<" 1 ="ED(" 1 ?"E≥ D<( 1 =?E"
이때 등호는 D나E 일 때 성립한다.

위의 과정에서 (가), (나)에 알맞은 것을 써넣으시오 .

- 3 -
SSEN 수학 (하) 14. 명제

286.

B단계 유형 뽀개기 2 임의의 실수 <> => I 에 대하여 조건


8 6 .

" " "


‘D< % =E 1 D= % IE 1 DI % <E & !’
의 부정과 서로 같은 것은?
① D< % =ED= % IEDI % <E& !
유형 01 명제

② < ≠ =이고 = ≠ I 이고 I ≠ <


③ D< % =ED= % IEDI % <E≠ !
④ <> => I는 서로 다르다.
282. 8 2
2 . 다음 중 명제가 아닌 것은?
⑤ <> => I 중에 서로 다른 것이 적어도 하나 있다 .
① "( 1 ' * % ( % )
② , 의 약수는 )" 의 약수이다,
③ ( & )이면 ( % . & # 이다.
④ 두 자연수 <> = 가 홀수이면 < 1 = 도 짝수이다.
⑤ 두 직선이 만날 때 생기는 맞꼭지각의 크기는 서로 같다.

유형 03 진리집합

283. 8 3
2 .
명제인 것만을 보기에서 있는 대로 고른 것은?

| 보 기 |
ㄱ. # 1 " & - ㄴ. ( % # & ( 1 $ 287. 전체집합 5 & 6( 7 ( 는 "! 이하의 자연수8에 대하여 조건
8 7 .
2

ㄷ. #( & ( 1 "( ㄹ. ( & "D( % )E 2 가 다음과 같을 때, 조건 2 의 진리집합을 구하여라.


① ㄱ, ㄴ ② ㄱ, ㄴ, ㄷ ③ ㄱ, ㄷ, ㄹ
④ ㄴ, ㄷ, ㄹ ⑤ ㄱ, ㄴ, ㄷ, ㄹ 2 3 ( 는 .의 배수이고 ), 의 약수이다.

유형 02 명제와 조건의 부정

284. 8 4
2 . 두 조건 2 3 % " : ( ≤ # , 4 3 % # ≤ ( : $에 대하여 288. 실수 전체의 집합에서 두 조건 2 3 ( * #> 4 3 ( ≤% ) 의
8 8 .
2

조건 ‘2 또는 ∼ 4 ' 의 부정을 말하시오. 진리집합을 각각 J > K라 할 때, 다음 중 조건 ′ % ) : ( ≤ #′ 의


진리집합을 나타내는 것은?
① J ∩K ② J ∪KI ③ J I∪K
I I
④ DJ ∩K E ⑤ DJ ∪K E

285. 8 5
2 .
다음 명제 중 그 부정이 참인 것은
① # 은 소수이다. ② ,* "
$ $
③ D% ) E 1 ) &% ) 1 ) ④ . 는 )'의 약수이다.
⑤ 정삼각형은 이등변삼각형이다.

- 4 -
SSEN 수학 (하) 14. 명제

289. 8 9
2 . 전체집합 5 & 6(A% # ≤ ( ≤ #> ( 는 정수 8에 대하여 두 292. 다음 보기의 명제 중 거짓인 것의 개수를 구하시오.
9 2 .
2

# "
조건 2> 4 가 2 3 ( % ( & !> 4 3 ( 1 "( % # & !일 때, 조건 ‘∼ 2 | 보 기 |
또는 4’의 진리집합의 모든 원소의 합을 구하시오.
ㄱ. (가 . 의 양의 약수이면 (는 ' 의 양의 약수이다.
ㄴ. <> = 가 무리수이면 < 1 = 도 무리수이다.
ㄷ. 삼각형 ABC 가 정삼각형이면 ∠T & ∠U 이다.
ㄹ. D< % "ED= % #E& ! 이면 < & " 또는 = & #이다.

유형 04 명제의 참, 거짓

유형 05 거짓인 명제의 반례

290. 9 0
2 . (> ?> N 가 실수일 때, 참인 명제인 것만을 보기에서 있는
대로 고른 것은? 293. 전체집합 5 에 대하여 두 조건
9 3 .
2

| 보 기 | 2> 4 의 진리집합을 각각 J > K 라 하자.


ㄱ. ( : ? : N 이면 (? : ?N 이다. 두 집합 J > K 가 오른쪽 그림과 같을 때,
ㄴ. ( ≤ ), ? ≤ ) 이면 ( 1 ? ≤ " 이다. 명제 ‘ 2이면 ∼ 4 이다.’ 가 거짓임을 보이는
원소는?
ㄷ. (" 1 ?" * ! 이면 ( ≠ !, ? ≠ ! 이다.
① < ② = ③ I ④ V ⑤ W
① ㄱ ② ㄴ ③ ㄷ
④ ㄱ, ㄷ ⑤ ㄴ, ㄷ

294. 9 4 .
2 O이 "! 보다 작은 자연수일 때, 명제
‘O 이 " 의 배수이면 O 은 # 의 배수이다.’
가 거짓임을 보이는 반례를 모두 구하시오.
291. 9 1
2 . 다음 명제 중 참인 것은?
"
① ( 1 ( & !이면 ( ≥ ! 이다.
"
② "( % ) & #이면 ( 1 ( % , & ! 이다.
③ ( 가 #의 배수이면 ( 는 +의 배수이다.
"
④ 자연수 O이 소수이면 O 은 홀수이다.
" "
⑤ 실수 (> ? 에 대하여 (? & !이면 ( 1 ? & ! 이다.

295. 전체집합 5 에 대하여 두 조건 2> 4 의 진리집합을 각각


9 5 .
2

J > K 라 할 때, 다음 중 명제 ‘ ∼ 2 이면 ∼ 4 이다.’ 가 거짓임을


보이는 원소가 속하는 집합은?
I I
① J ∩K ② J ∩K ③ J ∪K
I I I
④ J ∩K ⑤ J ∩K

- 5 -
SSEN 수학 (하) 14. 명제

유형 06 명제의 참, 거짓과 진리집합 유형 07 명제가 참이 되도록 하는 상수 구하기


☯ ☯

296. 9 6
2 . 전체집합 5 에 대하여 세 조건 299. 두 조건 2 3 A( % #A: \ , 4 3 % " ≤ ( ≤ ,에 대하여 명제
9 9 .
2

2> 4> X 의 진리집합을 각각 J > K> Y 라 2 → 4 가 참이 되도록 하는 자연수 \ 의 개수를 구하시오.


할 때, 세 집합 J > K> Y 사이의 포함
관계는 오른쪽 그림과 같다 . 다음 중
거짓인 명제는?
① 4 →2 ② 4→X
③ ∼2 → ∼4 ④ ∼2 → ∼ X
⑤ ∼X → ∼4

300. 명제 ‘) : ( ≤ .이면 < % # : ( : < 1 "이다.’ 가 참이


0 0 .
3

되도록 하는 실수 < 의 값의 범위를 구하시오.

297. 9 7
2 . 전체집합 5 에 대하여 두 조건 2> 4의 진리집합을 각각
J > K 라 하자. 명제 ∼ 2 →4 가 참일 때, 다음 중 항상 옳은 것은?
① J ∩K & ∅ ② J ∪K & 5 ③ J ∪KI & K I
④ J% K&∅ ⑤ K%J & ∅

301. 세 조건
0 1 .
3

2 3 % ) ≤ ( ≤ # 또는 ( ≥ $ , 4 3 ( ≥ < , X 3 ( ≥ =
에 대하여 두 명제 4 → 2 , 2 → X 가 모두 참이 되도록 하는 <의
최솟값과 =의 최댓값의 합을 구하시오. (단, <> = 는 실수이다.)

298. 9 8
2 .
전체집합 5 에 대하여 세 조건 2> 4> X 의 진리집합을 각각
J > K> Y 라 할 때, Y ⊂ DJ % KE 인 관계가 성립한다. 이때 항상
참인 명제인 것만을 보기에서 있는 대로 고른 것은?

| 보 기 |
ㄱ. 2 → ∼ 4 ㄴ. 4 → ∼ X ㄷ. 2 → X
ㄹ. X → 2 ㅁ. X → ∼ 4

① ㄱ, ㄴ, ㄷ ② ㄱ, ㄷ, ㅁ ③ ㄴ, ㄷ, ㄹ
302. 두 조건 2 3 7( % )7 ≥ \> 4 3 7( 1 "7 : .에 대하여 명제
0 2 .
3

④ ㄴ, ㄹ, ㅁ ⑤ ㄷ, ㄹ, ㅁ
∼ 4 → 2가 참이 되도록 하는 양수 \ 의 최댓값을 구하시오.

- 6 -
SSEN 수학 (하) 14. 명제

유형 08 ‘모든’이나 ‘어떤’이 있는 집합 유형 09 명제와 역, 대우의 참, 거짓


☯ ☯

303. 0 3
3 . 다음 중 거짓인 명제는? 306. 0 6 .
3 (> ? 가 실수일 때. 다음 중 그 역이 참인 명제는?
① 어떤 소수는 짝수이다. ① ( & !이면 (? & ! 이다.
"
② 모든 실수 ( 에 대하여 ( ≥ ! 이다. ② #( % - * ! 이면 ( * " 이다.
"
③ 어떤 양의 실수 ( 에 대하여 ( : ( 이다. ③ (> ? 가 짝수이면 (? 는 짝수이다.
"
④ 어떤 실수 ( 에 대하여 ( 1 ( & ! 이다. ④ ( * !이고 ? * ! 이면 ( 1 ? * ! 이다.
" ⑤ A(A1 A?A& !이면 ( & ! 이고 ? & ! 이다.
⑤ 모든 무리수 (에 대하여 ( 은 유리수이다.

유형 09 명제와 역, 대우의 참 거짓

<새 미주 번 호>
307. 대우가 거짓인 명제인 것만을 보기에서 있는 대로 고른
0 7 .
3

것은?
(단 , <> =는 실수이다.)
304. 0 4
3 . 전체집합 5 & 6% )> !> )8에 대하여 (∈5 , ?∈5 일 때,
| 보 기 |
다음 중 참인 명제는?
ㄱ. <=가 유리수이면 <> = 가 모두 유리수이다.
① 모든 ( 에 대하여 "(∈5 이다.
ㄴ. <= * ! 이면 < * !> = * ! 이다.
"
② 모든 ( 에 대하여 ( * ! 이다. "
ㄷ. < :% ) 또는 < ≥ # 이면 < % .< 1 # ≥ !이다.
③ 어떤 ( 에 대하여 ( % ) * ! 이다.
"
④ ( & ) 을 만족시키지 않는 ( 가 있다. ① ㄱ ② ㄴ ③ ㄷ
" " ④ ㄱ, ㄴ ⑤ ㄴ, ㄷ
⑤ 모든 (> ? 에 대하여 ( 1 ? & ) 이다.

308. 다음 중 그 역과 대우가 모두 참인 명제는?


0 8 .
3

(단 , <> => I> ( 는 실수이다.)


305. 0 5
3 .
"
명제 ‘어떤 실수 ( 에 대하여 ( % #( 1 # ≤ ! 이다.’에 ① ( * !이면 ( * ) 이다.
"
대하여 다음에 답하시오. ② ( & ) 이면 ( 1 ) & ! 이다.
(1) 명제의 부정을 말하시오. ③ ( & "이면 ( D( % " E & !이다.
(2) 명제의 부정의 참, 거짓을 판별하시오. ④ < & =이면 <I & =I 이다.
⑤ 두 집합 ] > T에 대하여 ] ⊂ T 이면 ] ∩T & ] 이다.

- 7 -
SSEN 수학 (하) 14. 명제

유형 10 명제의 대우를 이용하여 상수 313.


네 조건 2> 4> X> ^ 에 대하여 세 명제 2 → ∼ 4 , 4 → X ,


1 3 .
3

구하기 ^ → 4가 모두 참일 때, 항상 참인 명제인 것만을 보기에서 있는


대로 고른 것은?
309. 0 9
3 . 두 실수 <> = 에 대하여 명제
| 보 기 |
‘< 1 = : .이면 < : \ 또는 = :% ) 이다.’
가 참일 때, 실수 \ 의 최솟값을 구하시오. ㄱ. 4 → 2 ㄴ. X → ^ ㄷ. 2 → ∼ ^

① ㄱ ② ㄷ ③ ㄱ, ㄴ
④ ㄱ, ㄷ ⑤ ㄴ, ㄷ

310. 1 0
3 .
"
명제 ‘( 1 <( % # ≠ ! 이면 ( ≠ ) 이다.’ 가 참일 때 , 실수
<의 값을 구하시오.

314. 네 조건 2> 4> X> ^ 에 대하여 두 명제 2 → ∼ 4 ,


1 4 .
3

∼ X → ∼ ^가 모두 참일 때, 다음 중 명제 2 → X 가 참임을
보이기 위해 필요한 참인 명제는?
① 2 → ∼^ ② 4 → ∼X ③ 4 → ∼^
④ ^ → ∼2 ⑤ ∼^ → 4

311. 1 1
3 . 두 조건 2 3 A( % )A≥ " , 4 3 A( % <A≥ . 에 대하여
명제 4 → 2 가 참이 되도록 하는 실수 < 의 값의 범위를 구하시오.

유형 12 삼단논법과 명제의 추론

315. 다음 두 명제가 모두 참일 때, 항상 참인 명제인 것은 ?


1 5 .
3

(가 ) 음악을 좋아하는 사람은 미술을 좋아한다.


(나 ) 음악을 좋아하지 않는 사람은 체육을 좋아하지 않는다.

① 음악을 좋아하는 사람은 체육을 좋아한다.


유형 11 삼단논법

② 미술을 좋아하는 사람은 체육을 좋아한다.


③ 음악을 좋아하지 않는 사람은 미술을 좋아하지 않는다.
312. 1 2
3 . 세 조건 2> 4> X 에 대하여 두 명제 2 → ∼ 4, X → 4가 ④ 체육을 좋아하는 사람은 미술을 좋아한다.
모두 참일 때, 다음 명제 중 반드시 참이라고 할 수 없는 것은? ⑤ 체육을 좋아하는 사람은 음악을 좋아하지 않는다.
① 2 → X ② 2 → ∼X ③ 4 → ∼2
④ ∼ 4 → ∼ X ⑤ X → ∼2

- 8 -
SSEN 수학 (하) 14. 명제

316. 1 6
3 .
A > B> C 세 학생에 대하여 다음이 모두 참일 때, 세 학생 319. 두 조건 2> 4에 대하여 2 가 4 이기 위한 충분조건이지만
1 9 .
3

중 여학생을 모두 고르시오. 필요조건이 아닌 것만을 보기에서 있는 대로 고른 것은?


(단 , <> => I 는 실수이다.)
(가) 적어도 한 사람은 여학생이다. | 보 기 |
(나) A 가 남학생이면 B가 남학생이거나 C 가 여학생이다.
ㄱ. 2 3 < & = & I & !
(다) A ,C 의 성별은 같다
4 3 D< % =E" 1 D= % IE" 1 DI % <E" & !
(라) C 가 여학생이면 B는 여학생이다
ㄴ. 2 3 < % I * = % I 4 3 <*=
ㄷ. 2 3 <= 1 ) * " 4 3 < * )> = * )

① ㄱ ② ㄴ ③ ㄱ, ㄴ
④ ㄱ, ㄷ ⑤ ㄴ, ㄷ

유형 14 충분조건, 필요조건과 명제의 참,


유형 13 충분조건, 필요조건, 필요충분조건


거짓
317. 1 7
3 . 두 조건 2> 4 에 대하여 다음 중 2 가 4 이기 위한 320. 세 조건 2> 4> X 에 대하여 2 는 4이기 위한 필요조건이고
2 0 .
3

필요조건이지만 충분조건이 아닌 것은? X 는 4 이기 위한 충분조건일 때, 참인 명제인 것만을 보기에서 있는


(단, (> ?> N 는 실수이고, N * !이다.) 대로 고른 것은?
① 2 3 (" 1 ? " & ! 4 3 (? & ! | 보 기 |
② 2 3 (*? 4 3 (N * ?N
③ 2 3 (? & A(?A 4 3 ( * !> ? * ! ㄱ. 2 → 4 ㄴ. 4 → X
④ 2 3 (&) 4 3 (" & ( ㄷ. X → 2 ㄹ. ∼ 2 → ∼ 4
⑤ 2 3 (&? 4 3 (" & ? "
① ㄱ ② ㄴ ③ ㄷ
④ ㄴ, ㄹ ⑤ ㄷ, ㄹ

318. 1 8
3 . 두 조건 2> 4 에 대하여 다음 중 2가 4이기 위한 321. 세 조건 2> 4> X 에 대하여 두 명제 4 → 2 ,
2 1 .
3

필요충분조건인 것은? (단, (> ? 는 실수이다.) ∼ X → ∼ 2가 모두 참일 때, 옳은 것만을 보기에서 있는 대로


① 2 3 A( 1 ?A& A(A1 A?A 4 3 ( ≥ !> ? ≥ ! 고른 것은?
② 2 3 ( ≥ !> ? ≥ ! 4 3 (? ≥ !
" " | 보 기 |
③ 2 3 ( *? 4 3 A(A* A?A
" " ㄱ. 4는 2 이기 위한 필요조건이다.
④ 2 3 ( 1? * ! 4 3 (1? * !
ㄴ. X는 2 이기 위한 필요조건이다.
" "
⑤ 2 3 ( 1? * ! 4 3 (? : ! ㄷ. 4는 X 이기 위한 충분조건이다.

① ㄱ ② ㄴ ③ ㄷ
④ ㄴ, ㄷ ⑤ ㄱ, ㄴ, ㄷ

- 9 -
SSEN 수학 (하) 14. 명제

유형 15 충분 ․ 필요 ․ 필요충분조건과 유형 16 집합의 연산과 필요충분조건


진리집합
325. 전체집합 5 의 두 부분집합 ] > T 에 대하여
2 5 .
3

322. 2 2
3 . 세 조건 2> 4> X 에 대하여 2 는 4 이기 위한 충분조건이고, D] ∪T E∩ DT % ] EI & ] ∪T
4는 X 이기 위한 충분조건이다. 전체집합 5 에 대하여 세 조건 가 성립하기 위한 필요충분조건인 것은?
2> 4> X 의 진리집합을 각각 J > K> Y 라 할 때, 다음 중 항상 옳은 ① ]⊂T ② T⊂] ③ ] ∩T & ∅
것은? ④ ]&5 ⑤ ] ∪T & 5
① Y ⊂ DJ ∪K E ② DJ ∩K E ⊂ Y ③ DK∩Y E ⊂ J
I I
④ Y% K & J ⑤ DJ ∪K E ⊂ Y

326. 전체집합 5 의 두 부분집합 ] > T 에 대하여 ] > T 가


2 6 .
3

서로소이기 위한 필요충분조건인 것만을 보기에서 있는 대로 고른


것은?

| 보 기 |

323. 2 3
3 . 전체집합 5 에 대하여 세 조건 ㄱ. ] ∩T & ∅ ㄴ. ] ∩T & ]
ㄷ. ] ∪T & ] ㄹ. T ⊂ ] I
2> 4> X 의 진리집합을 각각 J > K> Y 라
하자. 세 집합 사이의 포함 관계가 오른쪽 ① ㄱ, ㄴ ② ㄱ, ㄷ ③ ㄱ, ㄹ
그림과 같을 때 , 다음 중 옳은 것은 ? ④ ㄴ, ㄷ ⑤ ㄴ, ㄹ
① 2 는 X이기 위한 충분조건이다.
② 2 는 ∼ X이기 위한 필요조건이다.
③ 4 는 2이기 위한 충분조건이다.
④ ∼ 2 는 4이기 위한 필요충분조건이다.
⑤ ∼ X 는 ∼ 2이기 위한 충분조건이다.

유형 17 충분 ․ 필요 ․ 필요충분조건이

되는 상수 구하기

327. 2 7 .
3 ( ≥ < 는 % $ ≤ ( ≤ - 이기 위한 필요조건이고,
= ≤ ( ≤ . 는 % $ ≤ ( ≤ - 이기 위한 충분조건일 때, < 의 최댓값과
= 의 최솟값의 합을 구하시오. (단, <> = 는 실수이고, = ≤ . 이다.)

324. 2 4
3 . 전체집합 5 에 대하여 세 조건 2> 4> X 의 진리집합을 각각

J > K> Y 라 할 때, DJ % Y IE∪ DK % J E & ∅ 이 성립한다. 다음 중


항상 옳은 것은 ? (단, J > K> Y는 공집합이 아니다.
① 2 는 4이기 위한 충분조건이다.
② 2 는 X이기 위한 필요조건이다.
③ X 는 4이기 위한 충분조건이다.
④ 4 는 ∼ X이기 위한 충분조건이다. 328. "
두 조건 2 3 .( % .( % # ≠ ! , 4 3 "( 1 < ≠ ! 에 대하여
2 8 .
3

⑤ X 는 ∼ 4이기 위한 필요충분조건이다.
2 가 4 이기 위한 충분조건이 되도록 하는 모든 실수 < 의 값의 합을
구하시오.

- 10 -
SSEN 수학 (하) 14. 명제

329. 2 9
3 . ( 1 ) & "( % ) 은 (" % <( 1 = & ! 이기 위한 필요충분조건일 332. 대우를 이용하여 명제 ‘실수 <> =에 대하여 <" 1 =" & ! 이면
3 2 .

때, 실수 <> = 에 대하여 < 1 =의 값을 구하시오. < & ! 이고 = & !이다.’가 참임을 증명하시오.

333. O이 자연수일 때, 명제 ‘O" 이 짝수이면 O 도 짝수이다.’에


330.
3 3 .

3 0
.
세 조건 2 3 % ) : ( : ! 또는 ( ≥ # , 4 3 ( ≤ < , 대하여 다음에 답하시오.
X 3 ( ≥ =에 대하여 ∼ 2 는 4 이기 위한 충분조건이고 ∼ 2 는 (1) 명제의 대우를 말하시오.
∼ X 이기 위한 필요조건이다. 이때 실수 <> = 에 대하여 < % = 의 (2) (1)을 이용하여 주어진 명제가 참임을 증명하시오.
최솟값을 구하시오.

유형 19 귀류법

유형 18 대우를 이용한 명제의 증명


331. 3 1
. 다음은 자연수 O 에 대하여 명제 ‘O 이 # 의 배수이면 O 도
" 334. 다음은
3 4 .
/0
$ 가 무리수임을 증명하는 과정이다.
#의 배수이다.’가 참임을 그 대우를 이용하여 증명하는 과정이다. | 증 명 |
| 증 명 | /0
$가 D가E 라고 가정하면
주어진 명제의 대우는 ‘O 이 # 의 배수가 아니면 O 도 # 의 배수가 <
/0
$ & 0 ( <> = 는 D나E 인 정수, = ≠ ! )
아니다.’이다. =
O& D가E 또는 O & D나E (\ 는 자연수) 꼴로 나타낼 수 있다. 양변을 제곱하여 정리하면
(이)라 하면 <" & $=" ⋯⋯ ㉠
"
(ⅰ) O & D가E "
일 때, O & #D D다E E 1 ) 따라서 < 이 D다E 이므로 <도 D다E 이다.

(ⅱ) O & D나E "


일 때, O & #D D라E E1 ) < & $\ ( \는 정수)로 놓으면 ㉠에서
"
즉 O 은 # 으로 나누면 나머지가 )인 자연수가 되므로 O 이 # 의
"$\" & $=" ∴ =" & $\"
"
" 따라서 = 이 D다E 이므로 =도 D다E 이다.
배수가 아니면 O 도 # 의 배수가 아니다.
따라서 주어진 명제의 대우가 참이므로 주어진 명제도 참이다. 그러므로 <> =가 D나E 라는 가정에 모순이므로 /0
$는
무리수이다.
위의 과정에서 (가)~(라)에 알맞은 것은 ?
(가) (나) (다) (라) 위의 과정에서 (가), (나), (다 )에 알맞은 것은?
"
① #\ % " #\ % ) #\ 1 "\ #\" 1 .\ 1 ) (가) (나) (다)
② #\ % " #\ % ) #\" % .\ 1 ) #\" % "\ ① 유리수 <≠= 홀수
③ #\ % ) #\ % " #\" % .\ 1 ) #\" % "\ ② 유리수 서로소 홀수
" " ③ 유리수 서로소 $ 의 배수
④ #\ 1 ) #\ 1 " #\ 1 "\ #\ 1 .\ 1 )
④ 무리수 <≠= $ 의 배수
⑤ #\ 1 " #\ 1 ) #\" 1 "\ #\" 1 .\ 1 )
⑤ 무리수 서로소 홀수

- 11 -
SSEN 수학 (하) 14. 명제

335. 3 5
.
/0
" 가 무리수임을 이용하여 명제 ‘유리수 <> = 에 대하여 338. 실수 <> = 에 대하여 다음 부등식이 성립함을 증명하고,
3 8 .

< 1 =/0
" & ! 이면 < & = & ! 이다.’ 가 참임을 증명하시오. 등호가 성립하는 경우를 구하시오.

<" 1 =" 1 ) ≥ <= 1 < 1 =

유형 21 절댓값 기호를 포함한 절대부등식


유형 20 실수의 성질을 이용한 ☯

절대부등식의 증명 339. 3 9 .

다음은 실수 <> = 에 대하여 부등식 A<A1 A=A≥ A< 1 =A 가 성립함을


336. 3 6
.
" "
다음은 실수 <> = 에 대하여 부등식 < 1 = ≥ <= 가 증명하는 과정이다.

성립함을 증명하는 과정이다. | 증 명 |

| 증 명 | DA<A1 A=AE" % A< 1 =A" & "D D가E E≥ !


# " "
<" 1 =" % <= & D< % D가E E" 1 0 =" ∴DA<A1 A=AE ≥ A< 1 =A
.
그런데 A<A1 A=A≥ ! , A< 1 =A≥ ! 이므로
#
<> = 가 실수이므로 D< % D D가E E" ≥ ! , 0 =" ≥ ! A<A1 A=A≥ A< 1 =A (단, 등호는 D나E 일 때 성립)
.
#
따라서 D< % D가E E" 1 0 =" ≥ ! 이므로 <" 1 =" ≥ <= 이고,
. 위의 과정에서 D가E> D나E 에 알맞은 것을 차례대로 나열한 것은?
등호는 D나E 일 때 성립한다. ① A<=A1 <=> <= ≤ ! ② A<=A1 <=> <= ≥ !
③ A<=A% <=> <= ≤ ! ④ A<=A% <=> <= ≥ !
위의 과정에서 (가), (나 )에 알맞은 것을 써넣으시오. ⑤ A<=A% <=> <= & !

340. 4 0 .
3

<>= 가 실수일 때, 옳은 것만을 보기에서 있는 대로 고른 것은?


337. 3 7
. <> = 가 실수일 때, 두 수 | 보 기 |
" " "
] & D<= % ) E , T & D< % ) ED= % ) E ㄱ. A<A1 A=A≥ A< % =A
의 대소 관계는? ㄴ. A< 1 =A≥ A< % =A
① ]*T ② ]≥T ③ ]:T ㄷ. A< % =A≥ A<A% A=A
④ ]≤T ⑤ ]&T
① ㄱ ② ㄴ ③ ㄱ, ㄴ
④ ㄱ, ㄷ ⑤ ㄴ, ㄷ

- 12 -
SSEN 수학 (하) 14. 명제

유형 22 산술평균과 기하평균의 관계 345. ) .


( * !> ? * ! 일 때, ( 1 ? 1 0 1 0 의 최솟값을 구하시오.


( ?
4 5 .
3

; 곱의 최솟값 구하기

341. 4 1
3 .

D "
ED '
E
양수 (> ? 에 대하여 ( 1 0 ? 1 0 은 (? & < 일 때,
? (
최솟값 = 를 갖는다. 이때 상수 <>= 에 대하여 < 1 =의 값을 구하시오.

.
346. 4 6 .
3 ( * " 일 때, .( % . 1 0 ≥ `이 항상 성립하도록 하는
(%"
342. 4 2
3 .

D )
ED .
E
< * !일 때, < % 0 < % 0 의 값이 최소가 되도록 하는
< < 실수 ` 의 최댓값을 구하시오.
<의 값을 구하시오.

343. 4 3
3 .
)
D)
< * !, = * ! , I * ! 일 때, D< 1 = 1 IE 0 1 0 의
< =1I E
최솟값을 구하시오.

) .(
347. /0
(" & ( 일
4 7 .
3 때, ( 1 0 1 0
( "
의 최솟값은? (단, ( ≠ ! )
( 1)
① " ② "/0
" ③ "/0
#
④ . ⑤ #/0
"

유형 23 산술평균과 기하평균의 관계

; 합의 최솟값 구하기

+
344. ( * % )일 때, ( 1 0 의 최솟값을 ` , 그때의 ( 의
( 1)
4 4
3 .

값을 O이라 하자. ` 1 O 의 값은? (


348. ( * ! 일 때, 0 의 최댓값을 구하시오.
① # ② $ ③ - (" 1 "( 1 +
4 8 .
3

④ + ⑤ ))

- 13 -
SSEN 수학 (하) 14. 명제

353. ( * ! , ? * ! 이고 #( 1 $? & )! 일 때, /0
#( 1 /0
$? 의
유형 24 산술평균과 기하평균의 관계
5 3 .
3

최댓값을 구하시오.
; 합 또는 곱이 일정할 때

349. 4 9
3 . 양수 (> ? 에 대하여 "( 1 #? & , 일 때, (? 의 최댓값을 a ,
그때의 (> ? 의 값을 각각 b> c 라 하자. 이때, a 1 b 1 c 의 값은?
① " ② . ③ ,
④ ' ⑤ )!

유형 25 산술평균과 기하평균의 관계

; 복잡한 식의 최대,최소

?1N N1( (1?


354. ( * ! , ? * ! , N * ! 일 때, 0 1 0 1 0 의
350. ( ? N
5 4 .
3

" "
5 0
3 . 양수 ( , ?에 대하여 .( 1 +? & .'일 때, (?의 최댓값을
최솟값을 구하시오.
구하시오.

355. 5 5 .
3

D ED ED
< = = I I <
E
< * ! , = * ! , I * ! 일 때, 0 1 0 0 1 0 0 1 0 의
= I I < < =
최솟값은?
) )
351. 5 1
3 . 양수 <> = 에 대하여 < 1 = & . 일 때, 0 1 0 의 최솟값은?
< = ① " ② . ③ ,
④ ' ⑤ )!

< =
356. 5 6 .
3 <" % "< 1 0 1 0 의 값이 최소가 되도록 하는 양수 <> = 에
= <

352. 5 2
3 . 양수 <> = 에 대하여 <= & )' 일 때, .< 1 "= 의 최솟값을 대하여 < 1 = 의 값을 구하시오.

구하시오.

- 14 -
SSEN 수학 (하) 14. 명제

" " 유형 27 코시-슈바르츠의 부등식 ☯

357. < * !, = * ! 이고 ( & < 1 0 , ? & = 1 0 일 때, (" 1 ?"의


= <
; <( 1 =?의 최대⋅최소
5 7
3 .

a
최솟값을 a , 그때의 <> =의 값을 각각 b> c라 하자. 이때 0 의
bc 361. 6 1 .
3
" "
실수 (> ? 가 ( 1 ? & $" 를 만족시킬 때, "( 1 #? 의
값을 구하시오.
최댓값을 a , 그때의 (> ? 의 값을 각각 b> c 라 하자. 이때
a 1 b 1 c 의 값은?
① )" ② )' ③ ".
④ #! ⑤ #,

유형 26 산술평균과 기하평균의 관계

; 도형에서의 활용

362. " "


실수 (> ? 가 ( 1 ? & $ 를 만족시킬 때, ( 1 "? 의
6 2 .
3

358. 5 8
3 . 길이가 )!!` 인 줄을 모두 최댓값과 최솟값의 곱은?
사용하여 오른쪽 그림과 같은 네 개의 작은 ① % "$ ② % )$ ③ %$
직사각형으로 이루어진 구역을 만들려고 ④ )$ ⑤ "$
한다. 구역의 전체 넓이가 최대가 되도록 할
때, 구역 전체 테두리인 바깥쪽 직사각형의 둘레의 길이를 구하시오.
(단, 줄의 굵기는 고려하지 않는다.)

363. " "


실수 (> ? 에 대하여 ( 1 ? & # 일 때,
6 3 .
3

359. 5 9
3 . 오른쪽 그림과 같이 수직인 두 벽면
(" 1 .( 1 ?" 1 #? 의 최댓값을 구하시오.

사이를 길이가 )"m 인 철망으로 막은 삼각형


모양의 창고가 있다. 이 창고의 밑면의 넓이의
"
최댓값이 \ m 일 때, \ 의 값을 구하여라.

364. 6 4 .
3 (" 1 ?" & < 를 만족시키는 실수 (> ?에 대하여 #( 1 ? 의
최댓값과 최솟값의 차가 "! 일 때, 양수 < 의 값을 구하시오.
360. 6 0
3 . 오른쪽 그림과 같이 점 AD"> # E 을
( ?
지나는 직선 0 1 0 & ) 이 ( 축, ? 축과 만나는
< =
점을 각각 B> C 라 할 때, 삼각형 OBC 의
넓이의 최솟값을 구하시오. (단, O 는 원점이고
< * !> = * ! 이다.)

- 15 -
SSEN 수학 (하) 14. 명제

유형 28 코시-슈바르츠의 부등식 369.


< * !> = * ! > I * ! 이고 < 1 = 1 I & ). 일 때,


6 9 .
3

; ( 1 ? 의 최대⋅최소
" " /0
< 1 "/=0 1 #/0
I 의 최댓값을 구하시오.

365. 6 5
3 . 실수 (> ? 에 대하여 #( 1 .? & $ 일 때, (" 1 ?" 의
최솟값은?
① ) ② " ③ #
④ . ⑤ $

유형 30 코시-슈바르츠의 부등식

; 도형에서의 활용

366.
< =
실수 <> = 에 대하여 0 1 0 & /0
)# 일 때, <" 1 =" 의
370. 오른쪽 그림과 같이 반지름의 길이가
7 0 .
3

" #
6 6
3 .

) 인원에 내접하는 직사각형의 둘레의 길이의


최솟값은? 최댓값은?
① )# ② "# ③ ", ① "/0
" ② "/0
#
④ #, ⑤ #+
③ #/0
" ④ ./0
"
⑤ ./0
#

367. " " "


실수 (> ?> N 가 ( 1 ? 1 N & ) > ( 1 ? 1 N & # 을
371. 오른쪽 그림과 같이 대각선의 길이가 #/0
6 7
3 .

#인
만족시킬 때, ( 의 최댓값을 구하시오.
7 1 .
3

직육면체의 모든 모서리의 길이의 합의 최댓값을


구하시오.

유형 29 코시-슈바르츠의 부등식

; <( 1 =? 1 IN의 최대⋅최소 372. 오른쪽 그림과 같이 한 변의 길이가 " 인


7 2 .
3

정삼각형 ABC 의 내부의 점 P 에서 세변


368. 6 8
3 .
" " "
실수 (> ?> N 에 대하여 ( 1 ? 1 N & " 일 때, AB> BC> CA 까지의 거리를 각각 < , = , I 라 할
( % "? 1 #N 의 최솟값은? 때, <" 1 =" 1 I" 의 최솟값을 구하시오.
① % "/ 0 - ② %$ ③ % "/0
$
④ %. ⑤ % "/0
#

- 16 -
SSEN 수학 (하) 14. 명제

376.

C단계 실력 굳히기 7 6 .
3
A , B , C , D 네 사람이 공놀이를 하다가 그중 한 사람이
유리창을 깼다. 그런데 네 사람은 다음과 같이 서로 엇갈린 진술을
하고 있다.
373. 7 3
3 . 전체집합 5 의 두 조건 2> 4 에 대하여
A : B 가 유리창을 깼어.
명제 2 → ∼ 4 가 참이면 2 ⇨ 4 , 거짓이면
B : A 가 한 말은 거짓말이야.
h 4 로 나타내기로 하자. 세 조건 2> 4> X 의
2⇨ C : A 가 유리창을 깼어.
진리집합을 각각 J > K> Y 라 할 때, D : 나는 안 깼어.
J > K> Y 사이의 포함 관계가 위의 그림과 같다. 이때 옳은 것만을
보기에서 있는 대로 고른 것은? 네 명 중 한 명의 진술만 참이라 할 때, 유리창을 깬 사람과 옳게
진술한 사람을 차례대로 구하시오.
| 보 기 |
h4
ㄱ. 2 ⇨ h ∼X
ㄴ. 2 ⇨
ㄷ. ∼ X ⇨ 4

① ㄱ ② ㄴ ③ ㄷ
④ ㄱ, ㄴ ⑤ ㄴ, ㄷ

374. 7 4
3 . 명제 ‘% # : ( ≤ " 인 어떤 실수 (에 대하여
< % . ≤ ( : "< 1 # 이다.’ 가 참이 되도록 하는 모든 정수 <의 값의 377. 두 실수 <> =에 대하여 세 조건 2> 4> X 는
7 7 .
3

합을 구하시오. (단, < *% - ) 2 3 A<A1 A=A& ! ,


4 3 <" % "<= 1 =" & ! ,
X 3 A< 1 =A& A< % =A
이다. 옳은 것만을 보기에서 있는 대로 고른 것은?

| 보 기 |
ㄱ. 2는 4 이기 위한 충분조건이다.
ㄴ. ∼ 2 는 ∼ X 이기 위한 필요조건이다.
ㄷ. 4이고 X 는 2 이기 위한 필요충분조건이다.

① ㄱ ② ㄷ ③ ㄱ, ㄴ
④ ㄴ, ㄷ ⑤ ㄱ, ㄴ, ㄷ

375. 7 5
3 . 실수 ( 에 대한 두 조건

2 3 D(" % `( 1 `ED(" 1 "( % #E : !


4 3 (" 1 "( % # : !
에 대하여 명제 2 → 4 가 참이 되도록 하는 실수 ` 의 최댓값을 < ,
최솟값을 = 라 할 때, % '<= 의 값을 구하시오.

- 17 -
SSEN 수학 (하) 14. 명제

378. 7 8
3 . 두 조건 2> 4 에 대하여 2 는 4 이기 위한 필요조건이지만 381. 실수 <, = 에 대하여 옳은 것만을 보기에서 있는 대로 고른
8 1 .
3

충분조건이 아닌 것만을 보기에서 있는 대로 고른 것은? 것은?


(단, ] > T> U 는 모두 공집합이 아니고, <> => I 는 실수이다.)
| 보 기 |
| 보 기 | ㄱ. D< 1 =E ≥ #<= "

ㄱ. 2 3 ] ∩T & ] 43 ]%T &∅ ㄴ. /0


7 <7 1 /0
7 = 7 ≥ /0
"D7 < 7 1 7 = 7E
ㄴ. 2 3 ] ⊂ DT∪U E 4 3 ] ⊂ T 또는 ] ⊂ U 0
ㄷ. 7 <7 1 7= 7 ≥ /<" 1 ="
ㄷ. 2 3 A< % =A: A< % IA 43 <: =:I " "
ㄹ. < 1 = 1 ) ≥ " D< 1 = % <= E
ㄹ. 2 3 <" 1 =" % <= & ! 43 <&=&!
① ㄱ, ㄴ ② ㄴ, ㄷ ③ ㄱ, ㄴ, ㄹ
① ㄱ, ㄴ ② ㄱ, ㄷ ③ ㄴ, ㄷ
④ ㄱ, ㄷ, ㄹ ⑤ ㄴ, ㄷ, ㄹ
④ ㄴ, ㄹ ⑤ ㄷ, ㄹ

382. 8 2 .
3 O개의 양수 <)> <"> ⋯> <O 에 대하여
379. 7 9
3 .
세 조건 2> 4> X 의 진리집합을 각각 ) ) )
j DO E & <) 1 <" 1⋯1 <O , k DO E & 0 1 0 1⋯1 0
<) <" <O
J & 6)> .8> K & 6% '> A<A> = 8> Y & 6)> "<=8
"
이라 할 때, 옳은 것만을 보기에서 있는 대로 고른 것은?
라 하자. 2 는 4 이기 위한 충분조건이고, 4 는 X 이기 위한 필요조건일
때, "< % = 의 최댓값은 ? (단, <> = 는 실수이다.) | 보 기 |
① %+ ② %$ ③ ! ㄱ. jD"EkD"E ≥ '
④ $ ⑤ + ㄴ. jDOE 1 kDOE ≥ "O
ㄷ. jDOE , kDOE 중 적어도 하나는 O 보다 크거나 같다.

① ㄴ ② ㄷ ③ ㄱ, ㄷ
④ ㄴ, ㄷ ⑤ ㄱ, ㄴ, ㄷ

)
380. 8 0
3 . 실수 < , =가 < * ! , = & ) % 0 을 만족시킬 때 , 세 수
<1 )

] & /0
<
)1< , T & )1 0, U & 0
) 383. 정사각형 모양의 A > B> C 세 개의 옥수수 밭을 경작하는
8 3 .
3

" /0
)%= "
농부가 있다. 세 밭의 넓이의 합은 '!! m 이고 A > B> C세 밭에서
의 대소 관계는?
수확한 옥수수의 양Dkg E 은 각 밭의 한 변의 길이 Dm E 의 # 배, . 배,
① ]:T:U ② ] :U:T
$ 배이다. 농부가 수확할 수 있는 최대의 수확량을 n kg 이라 할
③ ]&U:T ④ T:]:U 때, n 의 값을 구하시오.
⑤ T&U:]

- 18 -
SSEN 수학 (하) 14. 명제

384. 8 4
3 . 좌표평면 위에 두 점 A D)> .E> B D% #> !E과 직선
o 3 ? &% #( 1 \ 가 있다. 명제 ‘직선 o 위의 어떤 점 P 에 대하여
388. 8 8 .
3

D )
E
)
< *% )> = *% ) 일 때, D< 1 = 1 " E 0 1 0 의
<1 ) =1 )
최솟값을 구하시오.
∠APB & +!° 이다.’가 참이 되도록 하는 정수 \ 의 개수를 구하시오.
(단, 점 P 는 두 점 A> B 가 아니다.)

389. 집합 q 의 모든 원소들의 합을 r Dq E 라 하자. 전체집합


385. < , =가 정수일 때, 다음 명제가 참임을 귀류법으로
8 9 .
3

8 5
3 .

증명하시오.
5 & 6( 7 (는 )! 이하의 자연수8의 두 부분집합 ] , T 에 대하여
] ∪T & 5 , ] ∩T & 6)> . 8 일 때, r D] E r DT E 의 최댓값을
"
구하시오.
이차방정식 ( 1 <( % = & ! 이 정수인 해를 가지면 < , = 중
적어도 하나는 짝수이다.

390. 오른쪽 그림과 같이


9 0 .
3

길이가 )! 인 선분 AB 의
386. 8 6
3 .
" "
모든 실수 (> ? 에 대하여 부등식 ( % "<(? 1 =? ≥ ! 이
중점을 C 라 하고 두 선분
성립하도록 하는 )! 이하의 자연수 <> = 의 순서쌍 D<> = E 의 개수를 AC> AB 를 각각 지름으로
구하시오. 하는 두 반원을 같은 쪽에
그린다. 호 AC 위에
0
AD& . 가 되도록 점 D를
잡고 직선 AD 가 호 AB와 만나는 점을 E 라 하자. 두 꼭짓점
F> I 가 각각 선분 AD> DC 위에 있고 변 GH 가 선분 AC 위에
있는 직사각형 FGHI의 넓이의 최댓값을 r), 두 변 JE> EL 이 각각
선분 AE> BE 위에 있고 꼭짓점 K 가 선분 AB 위에 있는
직사각형 EJKL의 넓이의 최댓값을 r"라 할 때, r) 1 r" 의 값을
구하시오.

387. 8 7
3 .

D) .E
두 양수 (> ? 에 대하여 D( % ?E 0 % 0 ≤ \ 가 항상
( ?
성립하도록 하는 실수 \ 의 최솟값을 구하시오.

- 19 -
SSEN 수학 (하) 14. 명제

- 20 -
SSEN 수학(하) 15. 함수

[0402~0403] 정의역이 다음과 같은 함수 # $ & ' + 의 그래프를


A단계 기본 다잡기 좌표평면 위에 나타내시오.


402. 0 2 .
4 /+0 %0 51

15-1 함수

403. 0 3 .
4 /& ) & 는 실수 1

[0391~0394] 다음 대응중에서 집합 ! 에서 집합 " 로의 함수인


것을 모두 찾고, 함수인 것은 정의역, 공역, 치역을 구하시오.

391. 9 1
3 . 392. 9 2
3 .

15-2 여러 가지 함수

[0404~0407] 정의역과 공역이 /' +0 90 +0 %1인 보기의 함수의


그래프 중에서 다음에 해당하는 것을 있는 대로 고르시오.
393. 394.
| 보 기 |
9 3
3 . 9 4
3 .

ㄱ. ㄴ.

ㄷ. ㄹ.

[0395~0398] 다음 함수의 정의역과 치역을 구하시오 .


395. 9 5
3 . # $ %& ' (

396. 9 6
3 . # $ )& ) * + 404. 0 4
4 .
일대일함수 405. 0 5
4 .
일대일대응

397.
-
# $' , 406. 0 6
4 .

항등함수 407. 0 7
4 .

상수함수
&
9 7
3 .

398. 9 8
3 . # $' &% * .
[0408~0411] 보기의 함수 중에서 다음에 해당하는 것을 있는 대로
고르시오.

| 보 기 |
[0405~0407] 다음 두 함수가 서로 같은 함수인지 알아보아라.
399. 9 9
3 .
정의역과 공역이 /' +0 + 1 인 두 함수 ㄱ. # $' & ㄴ. # $ % ㄷ. # $ &

+
2 3& 4 $ &50 6 3& 4 $ ,
&
408. 일대일함수
0 8 .
4

400. 0 0
4 . 2 3& 4 $ 7&70 6 3& 4 $ 8,
&% 409. 일 대일대응
0 9 .
4

410. 항 등함수
1 0 .
4

&% ' .
401. 0 1
4 . 23& 4 $ & ' %0 63&4 $ ,
&*%
411. 상수함수
1 1 .
4

- 1 -
SSEN 수학(하) 15. 함수

15-3 합성함수 15-4 역함수


☯ ☯

421. 다음에 답하시오.


2 1 .
4

[0412~0415] 두 함수 2 : ! → " 0 6 : " → ! 가 아래 그림과


(1) 오른쪽 그림에서 함수 2 : ! → " 의
같을 때, 다음을 구하시오.
역함수가 존재하도록 대응 관계를
완성하시오.

'+
(2) (1)의 함수 2 에서 2 3% 4 * 2 3% 4 의
값을 구하시오.

'+
(3) (1)의 함수 2 에서 2 3A 4 * 2 ' + 35 4 $ 5 을 만족시키는 상수
412. 1 2
4 .
36 ∘ 2 43- 4 A 의 값을 구하시오.

413. 1 3
4 . 36 ∘ 243=4 [0422~0423] 함수 2 3& 4 $' & * . 에 대하여 다음 등식을
만족시키는 상수 A 의 값을 구하시오.
422. 2 ' + 35 4 $ A
414.
2 2 .
4

1 4
4 . 32 ∘ 643>4

423. 2
'+
3A 4 $ (
415.
2 3 .
4

1 5
4 .
32 ∘ 6 43? 4

[0424~0425] 다음 함수의 역함수를 구하시오 .

%
424. 2 4 .
4 # $ %& * 5
[0416~0419] 두 함수 23&4 $ 5& ' +0 63& 4 $ & 에 대하여 다음
합성함수를 구하시오.
416. 6 ∘2
425. + +
# $ ,& ' ,
1 6
4 .

% .
2 5 .
4

417. 2 ∘6
[0426~0429] 오 른쪽 그림과 같은 함수
1 7
4 .

2 : ! → " 에 대하여 다음을 구하시오.


426. 2 ' + 3+ 4
418.
2 6 .
4

1 8
4 . 2 ∘2

427. 32 ' + 4' + 35 4


419.
2 7 .
4

1 9
4 . 6 ∘6

428. 2 8 .
4
32 ∘ 2 ' + 43( 4

420. 2 0
4 .
%
세 함수 23&4 $ & * +0 63&4 $ %& ' +0 @3&4 $ & 에 429. 2 9 .
4
32 ' + ∘2 43. 4

대하여 32 ∘ 6 4 ∘ @ $ 2 ∘ 36 ∘ @ 4 가 성립함을 확인하시오.

- 2 -
SSEN 수학(하) 15. 함수

[0438~0441] 함 수 # $ 2 3& 4 의 그래프가


15-5 함수와 그 역함수의 그래프

오른쪽 그림과 같을 때, 다음 식의
그래프를 그리시오.
[0430~0432] 함 수 # $ 2 3& 4 의 그래프가 438. 3 8 .
4 # $ ) 2 3& 4)
오른쪽 그림과 같을 때, 다음에 답하시오.
430. 3 0
4 . 함수 # $ 2 3& 4 를 구하시오.

439. 3 9 .
4 # $ 2 3) & )4

440. ) # ) $ 2 3& 4
431. 함수 # $ 2 3& 4 의 역함수 # $ 2
'+
3& 4 를 구하시오.
4 0 .

3 1
4 .

441. 4 1 .
) # ) $ 2 3) & )4

432. 3 2
4 . 함수 # $ 2
'+
3& 4 의 그래프를 좌표평면 위에 나타내시오.

[0442~0445] 다음 식의 그래프를 그리시오 .


442. 4 2 . # $ ) & * +)

15-6 절댓값 기호를 포함한 식의 그래프


443. 4 3 . # $ %) & )' 5

433. 3 3
4 . 함수 2 3& 4 $ )& * %)' + 에 대하여 다음에 답하시오.
(1) & ≥' % 일 때, 2 3& 4 $ A& * > 꼴로 나타내시오. 444. 4 4 .
) # ) $ &% * .& * 5
(2) & C' % 일 때, 2 3& 4 $ A& * > 꼴로 나타내시오.
(3) # $ 2 3& 4 의 그래프를 그리고, 치역을 구하시오.

445. 4 5 .
) # ) $ ) &% ) ' -) & ) * (

[0434~0437] 함 수 # $ 2 3& 4 의 그래프가


오른쪽 그림과 같을 때, 다음 식의
그래프를 그리시오. [0446~0449] 다음 식의 그래프를 그리시오 .

434. 3 4
4 . # $ ) 2 3& 4) 446. 4 6 .
)& ) * )# ) $ +

435. 3 5
4 . # $ 2 3) & )4 447. 4 7 .
)& ) ' )# ) $ %

436. 3 6
4 .
) # ) $ 2 3& 4 448. 4 8 .
# $ ) & )* & ' 5

437. 3 7
4 .
) # ) $ 2 3) & )4 449. 4 9 . # $ ) & * %) * ) & ' + )

- 3 -
SSEN 수학(하) 15. 함수

유형 02 함숫값

B단계 유형 뽀개기
453. 음이 아닌 정수 전체의 집합에서 정의된 함수 2 3& 4 가
5 3 .
4

유형 01 함수의 뜻

2 3& 4 $ /&2 3*&+' . 4 39 ≤ & ≤ . 4


3& F . 4
일 때, 2 35 4* 2 3%G 4 의 값을 구하시오.
450. 5 0
4 . 두 집합 ! $ /' %0 ' +0 910 " $ /90 +0 %1 에 대하여
! 에서 " 로의 함수인 것만을 보기에서 있는 대로 고른 것은?

| 보 기 |

ㄱ. 2 3& 4 $ ) & )
ㄴ. 6 3& 4 $ &% ' +
+ 3& ≥ 9 4
ㄷ. @ 3& 4 $ /
% 3& C 9 4
454. 5 4 .
4
%
함수 2 3& 4 에 대하여 2 3& ' % 4 $ & ' ( 일 때, 2 3. 4 의
값을 구하시오.
① ㄱ ② ㄱ, ㄴ ③ ㄱ, ㄷ
④ ㄴ, ㄷ ⑤ ㄱ, ㄴ , ㄷ

451. 5 1
4 .
실수 전체의 집합 D 에 대하여 D 에서 D 로의 함수의
그래프인 것만을 보기에서 있는 대로 고르시오.

| 보 기 |

ㄱ. ㄴ. 455. 실수 전체의 집합에서 정의된 함수 2 3& 4 가


5 5 .
4

2 3& 4 $ /& * + 3&는


' & 3&는
유리수 4
무리수 4

일 때, 이차방정식 & * -& ' 5 $ 9 의 두 근 H0 I 에 대하여


%

2 3H 4 * 2 3I 4 ' 2 3HI 4 의 값은?


ㄷ. ㄹ. ① . ② - ③ =
④ +9 ⑤ +%

456. 임 의의 자연수 J 에 대하여 함수 2 가 다음 조건을 모두


452. 두 집합 ! $ /& ) ' + ≤ & ≤ +10 " $ /# ) ' + ≤ # ≤ 51 에
5 6 .
4

5 2
4 .

만족시킬 때, 2 3GGG 4 * 2 3+999 4 의 값을 구하시오.


대하여 다음 중 ! 에서 " 로의 함수가 아닌 것은?
① 2 3& 4 $ & * + ② 2 3& 4 $' %& * +
+ (가 ) 2 3%J 4 $ 2 3J 4
③ 2 3& 4 $ , ) & ' + ) ④ 2 3& 4 $ &% * % (나 ) 2 3%J ' + 4 $ J ' +
%
⑤ 2 3& 4 $ 3& * + 4% ' %

- 4 -
SSEN 수학(하) 15. 함수

유형 03 함수의 정의역, 공역, 치역 유형 04 조건을 이용하여 함숫값 구하기


☯ ☯

457. 5 7
4 .
집합 ! $ /& )' + ≤ & ≤ 51 에 대하여 ! 에서 ! 로의 461. 임의의 실수 &0 # 에 대하여 함수 2 가
6 1 .
4

함수 2 3& 4 $ A& * > 의 공역과 치역이 서로 같다. 이때 실수 2 3& * # 4 $ 2 3& 42 3# 4


A0 > 에 대하여 A> 의 값을 구하시오 . (단 , A>≠ 9 ) 를 만족시키고 2 3+ 4 $ % 일 때, 2 3N 4 의 값을 구하시오.

458. 5 8
4 . 정의역이 /' +0 A0 %0 >1 인 함수 # $ &% ' 5& * + 의
치역이 /' +0 (1 일 때, 상수 A0 > 에 대하여 A ' > 의 값은? (단,
A C >)
① '% ② '5 ③ '.
④ '( ⑤ '- 462. 임 의의 실수 &0 # 에 대하여 함수 2 가
6 2 .
4

2 3& * # 4 $ 2 3& 4 * 2 3# 4
를 만족시키고 2 3% 4 $ . 일 때, 옳은 것만을 보기에서 있는 대로
고른 것은?

| 보 기 |

ㄱ. 2 39 4 $ 9
ㄴ. 2 3' + 4 $' %
ㄷ. 임의의 자연수 J 에 대하여 2 3J& 4 $ J2 3& 4 이다.

459. 5 9
4 .
집합 L $ /90 +0 %0 50 ⋯1 에 대하여 L 에서 L 로의 ① ㄱ ② ㄴ ③ ㄱ, ㄴ
함수 2 3& 4 가 ④ ㄱ, ㄷ ⑤ ㄱ, ㄴ , ㄷ
2 3& 4 $ 3&% 을 . 로 나누었을 때의 나머지)
일 때, 함수 2 3& 4 의 치역은?
① /91 ② /+1 ③ /90 +1
④ /90 +0 %1 ⑤ /90 +0 %0 51

463. 함수 2 3& 4 에 대하여


6 3 .
4

52 3& 4 * %2 3+ ' &4 $ (&


일 때, 2 3A 4 $ +5 을 만족시키는 실수 A 의 값을 구하시오.

460. 6 0
4 . 일차함수 # $ A& * + 의 정의역이 /& ) + ≤ & ≤ %1 이고
공역이 /# ) % ≤ # ≤ -1 일 때, 실수 A 의 값의 범위를 구하시오.

- 5 -
SSEN 수학(하) 4. 함수

유형 05 서로 같은 함수 유형 06 일대일대응 찾기
☯ ☯

464. 6 4
4 .
집합 ! $ /' +0 +1 을 정의역으로 하는 두 함수 468. 보기의 함수 중 일대일대응이 것만을 있는 대로 고른
6 8 .
4

%
2 3& 4 $ A& * >0 6 3& 4 $' & * 5& ' + 것은?
에 대하여 2 $ 6 일 때, A> 의 값을 구하시오. (단, 정의역과 공역은 모두 실수 전체의 집합이다.)
(단 , A0 > 는 상수이다.)
| 보 기 |
ㄱ. # $ %& ' + ㄴ. # $ & * 7&7
%
ㄷ. # $' & * %&

① ㄱ ② ㄱ, ㄴ ③ ㄱ, ㄷ
④ ㄴ, ㄷ ⑤ ㄱ, ㄴ , ㄷ

465. 6 5
4 . 집합 ! $ /' +0 90 +1 에 대하여 ! 에서 ! 로의 두 함수
2 0 6 와 보기와 같을 때, 2 $ 6 인 것만을 있는 대로 고른 것은?

| 보 기 |
,
ㄱ. 2 3& 4 $ 8&% 0 6 3& 4 $' & 469. 일 대일대응의 그래프인 것만을 보기에서 있는 대로
6 9 .
4

ㄴ. 2 3& 4 $ ) & )0 6 3& 4 $ &% 고르시오.


ㄷ. 2 3& 4 $ &0 6 3& 4 $ &5 (단, 정의역과 공역은 모두 실수 전체의 집합이다.)

| 보 기 |
① ㄱ ② ㄷ ③ ㄱ, ㄴ ㄱ. ㄴ.
④ ㄴ, ㄷ ⑤ ㄱ, ㄴ , ㄷ

ㄷ. ㄹ.

466. 6 6
4 .
집합 ! $ /+0 %1 를 정의역으로 하는 두 함수 2 0 6 가
2 3& 4 $ &5 * %A&% * 5>0 6 3& 4 $' A& * >
이고 두 함수가 서로 같을 때, 함수 2 의 치역을 구하시오.
(단 , A0 > 는 상수이다.)

유형 07 일대일대응이 되기 위한 조건

470. 두 집합 ! $ /& ) ' 5 ≤ & ≤ 51,


7 0 .
4

" $ /# ) + ≤ # ≤ +51에 대하여 ! 에서 " 로의 함수


467. 6 7
4 . 집합 ! 를 정의역으로 하는 두 함수 23&4$ A& * > 3 A F 9 4가 일대일대응일 때, A * >의 값을 구하시오.
2 3& 4 $ &%0 6 3& 4 $ .& ' 5 에 대하여 2 $ 6 가 되도록 하는 집합 (단 , A0 > 는 상수이다.)
! 를 모두 구하시오.
(단 , ! ≠ ∅)
SSEN 수학(하) 15. 함수

471. 7 1
4 . 실수 전체의 집합에서 정의된 함수 474. 집 합 ! $ /' +0 90 +1에 대하여 ! 에서 ! 로의 세 함수
7 4 .
4

20 60 @는 각각 일대일대응, 항등함수, 상수함수이고


3& ≥ %4
23&4$
3& C %4/ %&& **.A 23' +4$ 63+4$ @3940 23' +4* 23+4$ 2394
가 일대일대응이 되도록 하는 상수 A 의 값을 구하시오. 일 때, 2 3+463' +4@3' +4 의 값을 구하시오.

472. 7 2
4 . 두 집합 ! $ /& ) & ≥ 51, " $ /# ) # ≥ %1에 대하여

! 에서 " 로의 함수 23&4$ &% ' %& * A가 일대일대응일 때, 23.4의 475. 집 합 ! $ /A0 >0 ?1를 정의역으로 하는 함수
7 5 .
4

값은? PS + 3& C %4
(단 , A 는 상수이다.) 23&4$ QS 5& ' = 3% ≤ & C -4
① ( ② N ③ G R &% ' =& * +. 3& ≥ -4
④ ++ ⑤ +5 가 항등함수일 때, A * > * ? 의 값을 구하시오.
(단, A0 >0 ?는 상수이다.)

유형 09 조건을 만족시키는 함수의 개수


476. 집 합 ! $ /+0 %0 51에 대하여 ! 에서 ! 로의 함수 중


유형 08 항등함수와 상수함수

7 6 .
4

일대일대응의 개수를 T , 항등함수의 개수를 U , 상수함수의 개수를


V 라 할 때, T * U * V의 값을 구하시오.
473. 7 3
4 . 집합 ! $ / 90 +0 %0 50 . 1에 대하여 ! 에서 ! 로의 함수
중 항등함수인 것만을 보기에서 있는 대로 고른 것은?

| 보 기 |
ㄱ. 23&4$ ( & 를 ( 로 나누었을 때의 나머지)
ㄴ. 63&4$ 5
ㄷ. @3&4$ 7&7

① ㄱ ② ㄱ, ㄴ ③ ㄱ, ㄷ 477. 두 집합
7 7 .
4

④ ㄴ, ㄷ ⑤ ㄱ, ㄴ , ㄷ ! $ /+0 %0 50 .1, " $ /' 50 ' %0 ' +0 90 +0 %0 51


에 대하여 다음 조건을 모두 만족시키는 함수 2 : ! → " 의
개수를 구하시오.

(가 ) &+ ≠ &% 이면 2 3&+4≠ 2 3&%4 이다.


(나 ) 72 3&47≤ %
SSEN 수학(하) 15. 함수

478. 집합 ! $ /' +0 90 +1에서 집합


482.
7 8
4 .

" $ /& * # ) &∈! 0 #∈! 1로의 함수 중 상수함수의 개수를 세 함수 20 60 @ 에 대하여


8 2 .
4

구하시오. 23&4$ & * %0 3@ ∘ 643&4$ N& ' -


일 때, 3@ ∘ 36 ∘ 2443A4$' - 을 만족시키는 상수 A의 값을
구하시오.

479. 7 9
4 . 집합 ! $ /' 50 90 51에 대하여 ! 에서 ! 로의 함수 중 483. 실 수 전체의 집합에서 정의된 함수 2 3&4 가
8 3 .
4

23&4' 23' &4$ 9


/
% 3+ ≤ & C 54
23&4$ & ' %& * 5
을 만족시키는 함수 2 의 개수를 구하시오. +% ' %& 35 ≤ & C (4
이고 23&4$ 23& * .4 를 만족시킬 때, 32 ∘ 243+.4의 값을
구하시오.

유형 10 합성함수의 함숫값

480. 8 0
4 . 실수 전체의 집합에서 정의된 함수 23&4가 484. 집합 ! $ /+0 %0 51에 대하여 ! 에서 ! 로의
8 4 .
4

일대일대응인 두 함수 20 6 가 있다. 2354$ 63%4$ + ,


( &는 유리수)
23&4 $ /'&%&
%
( &는 무리수)
36 ∘ 24354$ 32 ∘ 643%4$ % 일 때, 23%4* 6354의 값은?
① % ② 5 ③ .
일 때, 32 ∘ 2 438,
% 4 의 값을 구하시오. ④ ( ⑤ -

유형 11 2 ∘ 6 $ 6 ∘ 2인 경우

481. 8 1
4 . 오른쪽 그림과 같은 함수
2 : ! → ! 에 대하여 485. 두 함수 23&4$ A& * +0 63&4$' & ' % 에 대하여
8 5 .
4

2354* 32 ∘ 24354* 32 ∘ 2 ∘ 24354 2 ∘ 6 $ 6 ∘ 2 가 항상 성립할 때, 상수 A 의 값은?


의 값은? ① + ② % ③ 5
① - ② N ④ . ⑤ (
③ = ④ G
⑤ +9
SSEN 수학(하) 15. 함수

486. 8 6
4 . 함수 2 : ! → ! 가 오른쪽 그림과 490. 9 0 .
4
%
두 함수 2 3&4$ %& ' +0 63&4$ & ' .& * A 가 모든 실수
같고, 함수 6 : ! → ! 가 63+4$ 5 , & 에 대하여 36 ∘ 243&4 ≥ 9 을 만족시킬 때, 실수 A의 최솟값은?
2 ∘ 6 $ 6 ∘ 2 를 만족시킬 때, ① + ② % ③ 5
63%4' 63.4 의 값은? ④ . ⑤ (
① + ② %
③ 5 ④ .
⑤ (

487. 두 함수 23&4$ %& * 5, 63&4$ A& * > 가 2 ∘ 6 $ 6 ∘ 2 를


491.
8 7
4 .

함수 2 3&4$' %& * A 에 대하여 함수 63&4 를


만족시킬 때, 함수 # $ 63&4의 그래프는 A의 값에 관계없이 한
9 1 .
4

점을 지난다. 이 점의 좌표를 구하시오. (단, A0 > 는 상수이다.)


63&4$ 32 ∘ 2 ∘ 243&4 라 하자. > ≤ & ≤ .에서 # $ 63&4 의
최솟값이 ' +N , 최댓값이 N 일 때 , 상수 A0 >에 대하여 A> 의 값을
구하시오.

유형 12 2 ∘ 6에 대한 조건이 주어진

경우
488. 두 함수 23&4$ & * A0 63&4$ >& * ? 에 대하여
8 8
4 .

유형 13 2 ∘ 6 $ @ 를 만족시키는

32 ∘ 643&4$ .& ' % 이고 23%4$ 5일 때, A>? 의 값을 구하시오.


(단 , A0 >0 ? 는 상수이다.)
함수 2 또는 6 구하기

492. %
두 함수 2 3&4$ %& * +0 63&4$ %& ' 5 에 대하여
9 2 .
4

36 ∘ 243&4$ 23&4 를 만족시키는 함수 @3&4는?


% %
① @3&4$ & ' % ② @3&4$ & ' +
% %
③ @3&4$ & * % ④ @3&4$ %& ' 5
%
⑤ @3&4$ %& * .

489. 8 9
4 . 함수 23&4$ &% ' A 에 대하여 32 ∘ 243&4 가 & ' %로
나누어떨어질 때, 모든 실수 A 의 값의 합은?
① 'G ② '. ③ +
④ . ⑤ G
SSEN 수학(하) 15. 함수

493. 9 3
4 . 두 함수 2 3&4$ 5& ' %0 63&4$' & * + 에 대하여 다음을 497. 자 연수 전체의 집합에서 정의된 함수 2 3&4가
9 7 .
4

구하시오. &
(1) 2 ∘ @ $ 6 를 만족시키는 함수 @3&4 , ( & 는 짝수)
23&4 $ / %
&*+
( & 는 홀수)
,%
+ J *+
$ 2J ∘ 2 로 정의할 때, 2J3+994$ + 을
이고 2 $ 20 2
만족시키는 자연수 J 의 최솟값은?
(2) W ∘ 2 $ 6 를 만족시키는 함수 W3&4
① 5 ② ( ③ N
④ G ⑤ ++

494. 9 4
4 . 두 함수 2 0 6 에 대하여
%& * +
23&4$ , 0 36 ∘ 243&4$' %& * +
5
일 때, 6354의 값을 구하시오.

498. 함 수 2 3& 4 $
9 8 .
4

/ & *& '%&%' %


%
3& ≥ 9 4
3& C 94
에 대하여

2 $ 2+0 2 ∘ 2 $ 2%0 2 ∘ 2% $ 250 ⋯0 2 ∘ 2J $ 2J * +


%9+=
로 정의할 때, 2 35 4 * 2%9+= 3. 4 의 값을 구하시오.
(단, J 은 자연수이다.)

유형 14 합성함수의 추정

495. 9 5
4 . 집합 L $ /+0 %0 50 .1에 대하여 L 에서 L 로의 함수
23&4가

23&4$ / & '. + 3& $ +4


3& ≥ %4
이고
2+ 3& 4 $ 2 3& 40 2J * + 3& 4 $ 2 32J 3& 44
로 정의할 때, 2 (93%4 의 값을 구하시오. (단, J 은 자연수이다.)

유형 15 함수의 그래프와 합성함수


499. 오 른쪽 그림은 두 함수
9 9 .
4

# $ 2 3& 40 # $ &의 그래프를 나타낸


것이다. 이때 32 ∘ 2 ∘ 2 43> 4 의 값과
같은 것은? (단, 모든 점선은 & 축 또는
# 축에 평행하다.)
496. 9 6
4 .
함수 23&4$' & * %에 대하여 ① A ② >
2+3&4$ 23&40 2%3&4$ 32 ∘ 243&40 ③ ? ④ X
253&4$ 32 ∘ 2 ∘ 243&40 ⋯ ⑤ X
로 정의할 때, 2 J3&4 를 구하시오. (단 , J 은 자연수이다.)
SSEN 수학(하) 15. 함수

500. 0 0
5 . + ≤ & ≤ ( 에서 정의된 함수
# $ 2 3& 4 의 그래프가 오른쪽 그림과 같을
503. 0 3 .
5

35'& 4
함 수 2 , $ .& * + 에 대하여 2
%
'+
394 의 값을

구하시오.
때, 32 ∘ 2 43A 4 $ .를 만족시키는 모든
실수 A의 값의 합은?
① ( ② -
③ N ④ =
⑤ G

504. 함 수 2 3& 4 $
0 4 .
5

/ &%&*'5% 3& C (4
3& ≥ (4
에 대하여

2' + 3-4 * 2' + 3+%4 의 값은?


① G ② +9 ③ ++
④ +% ⑤ +5

유형 16 역함수의 정의

501. 0 1
5 . 실수 전체의 집합에서 정의된 함수 2 3& 4 $ A& * > 에
'+
대하여 2 3. 4 $ %0 2' + 3' ( 4 $' + 일 때, A% * >% 의 값을
구하시오.
(단 , A0 > 는 상수이다.)

유형 17 역함수가 존재하기 위한 조건

505. 두 집합 ! $ /& ) % ≤ & ≤ - 10 " $ /# ) A ≤ # ≤ > 1에


0 5 .
5

대하여 ! 에서 " 로의 함수 2 3& 4 $ 5& ' % 의 역함수가 존재할 때,


A * > 의 값은? (단, A0 > 는 상수이다.)
① +% ② +- ③ %9
④ .= ⑤ -.

502. 0 2
5 . 정의역이 /& ) & ≥' - 1 인 이차함수 2 3& 4 에 대하여

2394 $ 9 , 2' + 3' +94 $' % , 2' + 35%4 $ . 일 때, 23%4 의 값은?


① - ② +9 ③ +.
④ += ⑤ %%

506. 집 합 ! $ /& ) & ≥ A 1 에 대하여 ! 에서 ! 로의 함수


0 6 .
5

23&4 $ &% ' %& ' .9 의 역함수가 존재할 때, 상수 A 의 값을


구하시오.
SSEN 수학(하) 15. 함수

507. 0 7
5 . 함수 2 3& 4 $ W& * ) & ' + ) * %의 역함수가 존재하도록 하는 511. +
두 함수 23&4 $ , & ' +0 63&4 $ %& * A 에 대하여
%
1 1 .
5

실수 W의 값의 범위를 구하시오.
36 ∘ 24' + $ 6' + ∘ 2' +
가 성립할 때, 상수 A 의 값은?
+
① '% ② '+ ③ ,
%
④ + ⑤ %

508. 0 8
5 . 실수 전체의 집합에서 정의된 함수
512. 두 집합 L $ /& ) & ≥ + 10 Y $ /# ) # ≥ 5 1 에 대하여
/ &5&*'A+ 3& C 5 4 1 2 .
5

23&4 $
3& ≥ 5 4 L 에서 Y 로의 함수 2 3& 4 $ ) %& ' % ) * ) & ' . ) 의 역함수를
'+
의 역함수 존재할 때, 32 ∘ 2' + 43++4 의 값을 구하시오. 구하시오.
(단 , A 는 상수이다.)

유형 19 2 $ 2' +인 함수

유형 18 역함수 구하기

513. 보 기의 함수 중 2 $ 2
1 3 .
5
'+
를 만족시키는 것만을 있는 대로
+
509. 0 9
5 . 일차함수 2 3& 4 $ A& ' - 의 역함수가 2
'+
3& 4 $ , & * > 일
%
고른 것은?

| 보 기 |
때, 상수 A0 >에 대하여 A> 의 값은?
① ' +% ② '- ③ '5 ㄱ. 23&4 $ & * +
④ - ⑤ +% ㄴ. 23&4 $' & * +
+
ㄷ. 23&4 $' ,
&

① ㄱ ② ㄴ ③ ㄱ, ㄷ
④ ㄴ, ㄷ ⑤ ㄱ, ㄴ , ㄷ

510. 1 0
5 . 실수 전체의 집합에서 정의된 함수 2 에 대하여
514. 함 수 2 3& 4의 역함수 2
1 4 .
5
'+
가 존재하고 2
'+
3% 4 $' 5 ,
23%& ' +4 $ .& * 5 일 때, 함수 # $ 23&4의 역함수 # $ 2' + 3&4 의 32 ∘ 243&4 $ & 일 때, 23%4의 값을 구하시오.
그래프의 & 절편과 # 절편의 합을 구하시오.
SSEN 수학(하) 15. 함수

515. 1 5
5 . 점 3-0 ' % 4 를 지나는 일차함수 # $ 2 3& 4 의 그래프와 519. +
두 함수 23&4 $ A& ' 50 63&4 $ , & * > 에 대하여
5
1 9 .
5

# $ 2' + 3&4 의 그래프가 일치할 때, 23' +4 의 값은?


① + ② % ③ 5
32 ∘ 643&4 $ & * ? 이고 6' + 354 $ % 일 때, 다음을 구하시오.
④ . ⑤ ( (단, A0 >0 ?는 상수이다.)
(1) A * > * ? 의 값

'+
(2) 6 32 3' + 44의 값

516. 1 6
5 .
집합 ! $ /A0 >0 ?1 에 대하여 ! 에서 ! 로의 함수 중
'+
2$2 를 만족시키는 함수 2 의 개수를 구하시오.

유형 21 역함수의 성질

520. 두 함수 23&4 $' 5& * +0 63&4 $ .& * +에 대하여


2 0 .
5

32 ∘ 36 ∘ 24' + ∘ 243=4의 값을 구하시오.

유형 20 합성함수와 역함수

517. 1 7
5 . 두 함수 2 3& 4 $ 5& ' %0 6 3& 4 $ & ' +에 대하여
'+
32 ∘ 6 43A 4 $ + 을 만족시키는 상수 A 의 값은?
① '% ② '+ ③ 9
521. 2 1 .
5
%
두 함수 2 3& 4 $ & * + 3& ≥ 9 40 6 3& 4 $ & ' 5 에 대하여

④ + ⑤ % 36 ∘ 2 4' + 3% 4 * 32' + ∘ 6' + 43% 4 의 값은?


① . ② - ③ N
④ = ⑤ +9

522. 두 함수
2 2 .
5

518. 1 8
5 . 두 함수 2 0 6 를 오른쪽 그림과 2 3& 4 $ / %& ' 5 3& ≥ 54
3& C 54
0 6 3& 4 $ & * .
'+ '+
같이 정의할 때, 32 ∘ 6 43+4 의 값은? 에 대하여 32 ∘ @ 43& 4 $ 6 3& 4 를 만족시키는 함수 @ 3& 4를
① 9 ② + 구하시오.
③ % ④ 5
⑤ .
SSEN 수학(하) 15. 함수

유형 21 역함수의 성질

526. 집 합 ! $ /+0 %0 50 .0 (1에 대하여


2 6 .
5

함수의 2 : ! → ! 가 있다. 오른쪽 그림은


523. 2 3
5 .
두 집합 ! $ /+0 %0 50 .10 " $ /+0 50 (0 N1에 대하여 함수 # $ 2 3&4 의 그래프의 일부를 나타낸
! 에서 " 로의 함수 20 6 가 각각 것이다. 함수 23&4 의 역함수 2
'+
3&4 가 존재할
23J4$ %J ' + , 63J4$ 3NJ 을 G 로 나누었을 때의 나머지4 때, 2
'+
35 4 * 2 '+
3( 4 의 값을 구하시오.
'+ '+
일 때, 32 ∘ 64 3(4* 36 ∘ 2 4 3N4의 값을 구하시오.

524. 2 4
5 . 세 함수 2 3&4$ %&0 6 3& 4 $' 5& * .0 @3&4에 대하여

36 ∘ 2' + ∘ @43&4$ 23&4가


'+
성립할 때, @3+4 의 값은?
① '+ ② '% ③ '5
유형 23 역함수의 그래프의 성질

④ '. ⑤ '(

+
527. 함 수 2 3&4$ , & ' 5 의 그래프와 그 역함수 # $ 2
'+
3&4 의
.
2 7 .
5

그래프의 교점의 좌표가 3A0 > 4 일 때, A * > 의 값은?


① '. ② '- ③ '=
④ ' +9 ⑤ ' +%

유형 22 함수의 그래프와 역함수


525. 2 5
5 . 오른쪽 그림은 함수 # $ 2 3&4의
그래프와 작선 # $ & 를 나타낸 것이다. 5
이때 32
'+
∘ 2' +43?4 의 값은?
528. 함 수 23&4 $ %& ' , 의 역함수를 2
2 8 .
5

%
'+
3&4 라 할 때, 두
(단 , 모든 점선은 &축 또는 # 축에 '+
3&4 의 그래프의 교점을 P 라 하자. 이때
함수 # $ 23&4 와 # $ 2
평행하다.) OP
선분 의 길이를 구하시오 .(단, O 은 원점이다)
① A ② >
③ ? ④ X
⑤ Z
SSEN 수학(하) 15. 함수

529. 2 9
5 . 함수 23&4$ &% ' %& * A 3& ≥ +4 의 그래프와 그 역함수 532. 함 수 # $ 23&4 의 그래프가 오른쪽
3 2 .
5

'+
# $ 2 3&4 의 그래프가 서로 다른 두 점에서 만나도록 하는 실수 그림과 같을 때, 다음 보기 중 # $ 2 37 & 74 와
A의 값의 범위를 구하시오. 7 # 7$ 2 3& 4 의 그래프의 개형을 차례대로
고르시오.

| 보 기 |
ㄱ. ㄴ.

PS + ㄷ. ㄹ.
530. 3 0
5 .
함수 2 3& 4 $ Q
S
,
%
&*G 3& ≥ 94
의 역함수를 63&4 라 할
R %& * G 3& C 9 4
때, 함수 # $ 2 3& 4 , # $ 6 3& 4 의 그래프로 둘러싸인 부분의 넓이를
구하시오.

유형 25 절댓값 기호을 포함한 식의


그래프

533. 다 음 중 함수 # $ ) %& ' 5 )의 그래프와 직선


유형 24 절댓값 기호를 포함한 식의 그래프
3 3 .
5

# $ ]3& * +4 ' % 가 만나도록 하는 상수 ] 의 값으로 적당하지


; # $ 23&4 의 그래프가 주어진 경우 않은 것은?
N ( +
① ', ② ', ③ ,
531. 3 1
5 .
함수 # $ 23&4의 그래프가 % % .
오른쪽 그림과 같을 때, 다음 중 . (
④ , ⑤ ,
# $ 7 23&4 7의 그래프의 개형은? ( .

① ②

③ ④
534. %
함수 # $ ) & ' %& ) 의 그래프와 직선 # $ A 가 서로 다른
3 4 .
5

세 점에서 만날 때, 상수 A 의 값을 구하시오.


SSEN 수학(하) 15. 함수

535. 3 5
5 . % 7 & 7* 7 # 7$ = 의 그래프가 나타내는 도형의 넓이는? 유형 26 두 개 이상의 절댓값 기호를 ☯

① (% ② (- ③ -9 포함한 함수
④ -. ⑤ -=
539. 함 수 # $ 7& * 57' 7& ' %7 의 최댓값을 ^ , 최솟값을
3 9 .
5

] 이라 할 때, ^ ] 의 값을 구하시오.

536. 3 6
5 .
) # ' % ) $ & * A 의 그래프와 # 축으로 둘러싸인 도형의
넓이가 G 일 때, 양수 A의 값을 구하시오.

540. 함 수 # $ 7 & * +7 * 7 & ' ( 7 * 7 & ' N 7 은 & $ A 일 때


4 0 .
5

최솟값 > 를 갖는다. 이 때 A * > 의 값은?


① ++ ② +5 ③ +(
④ +N ⑤ +G

537. 3 7
5 . 정의역이 /& ) 9 ≤ & ≤ %1인 함수 2 3&4$ ) & ' + ) 에 대하여
방정식 32 ∘ 243&4$ A& * > 의 근이 무수히 많을 때, 상수 A0 > 에
% %
대하여 A * > 의 값을 구하시오. (단, > ≠ 9)

541. 함 수 # $ 7 & * .7* 7 & ' + 7 의 그래프와 직선 # $ = 로


4 1 .
5

둘러싸인 도형의 넓이를 구하시오.

538. 3 8
5 . 함수 # $ 7 %& ' . 7' . 의 그래프와 직선 # $ A 로 둘러싸인
도형의 넓이가 += 일 때, 상수 A 의 값을 구하시오.
SSEN 수학(하) 15. 함수


544. 두 함수 # $ 2 3& 40 # $ 6 3& 4의 그래프가 다음 그림과
4 4 .
5

C단계 실력 굳히기 같을 때, 함수 # $ 32 ∘ 643&4의 그래프의 개형으로 알맞은 것은?

542. 4 2
5 . 두 집합 L $ /& ) & 는 % 이상의 자연수1 ,
Y $ /& ) & 는 자연수1 에 대하여 함수 2 : L → Y 를 J∈L 일 때,
> > >_
J $ A+ + A% % ⋯ A_
(단 , A`는 서로 다른 소수, >`는 자연수, + ≤ ` ≤ _)
이면 23J4 $ _ 로 정의하자 . 옳은 것만을 보기에서 있는 대로 고른
것은?
① ②
| 보 기 |

ㄱ. 2 3G9 4 $ 5
ㄴ. J∈L 일 때, J 의 값이 증가하면 23J4 도 증가한다.
ㄷ. 집합 L 에 속하는 서로소인 두 자연수 ]0 J 에 대하여
2 3]J 4 $ 2 3] 4 * 2 3J 4 이다. ③ ④

① ㄱ ② ㄴ ③ ㄱ, ㄷ
④ ㄴ, ㄷ ⑤ ㄱ, ㄴ, ㄷ

543. 4 3
5 . 집합 ! $ /+0 %0 51에 대하여 두 함수
2 : ! → !0 6 : ! → !
가 있다. 옳은 것만을 보기에서 있는 대로 고른 것은?

| 보 기 |

ㄱ. 20 6 가 모두 항등함수이면 6 ∘ 2 는 항등함수이다.
ㄴ. 6 ∘ 2 가 항등함수이면 2 는 일대일함수이다.
ㄷ. 6 ∘ 2 가 일대일함수이면 2 또는 6중 어느 하나는 항등함수이다.
545. 두 함수
4 5 .
5

① ㄱ ② ㄴ ③ ㄱ, ㄴ
/
%
* * 3& C 94
④ ㄱ, ㄷ ⑤ ㄴ, ㄷ 23&4$ & %A& - 0 63&4$ & * +9
&*- 3& ≥ 94
에 대하여 합성함수 36 ∘ 243&4 의 치역이 /# ) # ≥ 91일 때, 상수
A 의 값을 구하시오.
SSEN 수학(하) 15. 함수

546. 4 6
5 . 정수 전체의 집합 a 에 대하여 a 에서 a 로의 함수 2 가 548. 오 른쪽 그림과 같이 중심이 O 인 원의 둘레를 +% 등분한 각
4 8 .
5

임의의 정수 &0 #에 대하여 23& * #4$ 23&4* 23#4 를 만족시킬 때, 점을 +0 %0 ⋯0 +% 라 하고, 집합 ! $ /+0 %0 50 ⋯0 +%1에
옳은 것만을 보기에서 있는 대로 고른 것은? 대하여 ! 에서 ! 로의 두 함수 2 0 6 를 다음과 같이 정의한다.
| 보 기 | 2 3& 4 $ ( &를 점 O를 중심으로 시곗바늘이 도는 반대 방향으로
ㄱ. 2394$ 9
-9° 만큼 회전시킨 점)

ㄴ. 임의의 정수 &에 대하여 23&4$ 23' &4 이다.


6 3& 4 $ ( &를 점 O를 중심으로 시곗바늘이 도는 반대 방향으로
ㄷ. 임의의 정수 &0 # 에 대하여
G9° 만큼 회전시킨 점)

32 ∘ 243& * #4$ 32 ∘ 243&4* 32 ∘ 243#4 이다. % 이상의 자연수 J 과 함수 W 3& 4에 대하여


W3J 4 3&4 $ 3W ∘ W ∘⋯∘ W43&4
dce cf
① ㄱ ② ㄱ, ㄴ ③ ㄱ, ㄷ J개

④ ㄴ, ㄷ ⑤ ㄱ, ㄴ, ㄷ 일 때, 옳은 것만을 보기에서 있는 대로 고른 것은?

| 보 기 |
35 4 3% 4
ㄱ. 32 ∘6 43& 4 $ &
35 4
ㄴ. 6 3+4 $ 2' + 3%4
3-4
ㄷ. @ 3& 4 $ 32 ∘ 6 43& 4 라 할 때, @ 3& 4 $ &이다.

① ㄱ ② ㄴ ③ ㄱ, ㄴ
④ ㄱ, ㄷ ⑤ ㄱ, ㄴ , ㄷ

547. 4 7
5 .
함수 # $ 23&4 3' + ≤ & ≤ +4 의
그래프가 오른쪽 그림과 같고 다음 두
조건을 만족시킬 때,
549. 두 집합 ! $ /& ) & F 9 10 " $ /# ) # F 9 1 에 대하여
4 9 .
5

! 에서 " 로의 함수 2 가 다음 조건을 모두 만족시킨다.


3 4 +
%
+
34
2 %9+= ' , * 2 %9+G , 의 값을 구하시오.
%
(가 ) 2 의 역함수가 존재한다.
(나 ) 함수 # $ 23&4 의 그래프가 점 3+0 +4 을 지난다.
+
(가 ) 2 3& 4 $ 2 3& 4 (다 ) &+∈! 0 &%∈! 일 때, &+ C &%이면 23&+ 4 F 23&% 4 이다.
J *+ J
(나 ) 2 3&4 $ 2 323&44 3J $ +0 %0 50 ⋯4
옳은 것만을 보기에서 있는 대로 고른 것은? (단, 9 C A C > C + )

| 보 기 |
ㄱ. 32 ∘ 2 43A 4 F 2 3> 4
'+
ㄴ. 2 3A4 F 2' + 3>4
'+
ㄷ. 32 ∘ 2' + 43A 4 C 32' + ∘ 2' + 43> 4

① ㄴ ② ㄱ, ㄴ ③ ㄱ, ㄷ
④ ㄴ, ㄷ ⑤ ㄱ, ㄴ , ㄷ
SSEN 수학(하) 15. 함수

550. 5 0
. 집합 ! $ /+0 %0 50 .0 ( 1 에 대하여 553. 세 집합 ! $ /+0 % 10 " $ /%0 5 10 a $ /90 + 4 에 대하여
5 3 .

함수 2 : ! → ! 가 그림과 같다. 함수 두 함수 20 6 가 2 : ! → " 이고, 6 : " → a 이다.


6 : ! → ! 는 다음 조건을 만족시킨다. 함수 6 ∘ 2 : ! → a 가 상수함수가 되도록 하는 함수 2 0 6 이
순서쌍 32 0 6 4 의 개수를 구하시오.
(가 ) 63+4 $ 50 63%4 $ (
(나 ) 6 의 역함수가 존재한다.

36 ∘ 2 43. 4 * 32 ∘ 6 43. 4 의 최댓값은?


① ( ② - ③ N
④ = ⑤ G

554. 두 함수 2 3& 40 6 3& 4 에 대하여


5 4 .

2 3& 4 $ %& * .0 2' + 3& 4 $ 6 3.& * 5 4


일 때, 함수 # $ 63&4 의 그래프와 & 축 및 #축으로 둘러싸인
도형의 넓이를 구하시오.

551. 5 1
. 임의의 실수 &0 # 에 대하여 함수 2 가
2 3& * # 4 $ 2 3& 4 * 2 3# 4
를 만족시키고 2 3+ 4 * 2 3% 4 * 2 35 4 * 2 3. 4 $ =9일 때, 2 3+ 4 의 값을
555. 실 수 전체의 집합에서 정의된 함수 2 가 다음 조건을 모두
5 5 .

구하시오. 만족시킨다.

(가 ) 9 ≤ & ≤ .에서 2 3& 4 $ . ' ) %& ' . ) 이다.


(나 ) 임의의 실수 & 에 대하여 2 3+ ' & 4 $ 2 35 * & 4이다.
(다 ) 임의의 실수 & 에 대하여 2 3& 4 $ 2 3' & 4 이다.

' = ≤ & ≤ = 에서 함수 # $ 23&4 의 그래프와 & 축으로 둘러싸인


도형의 넓이를 구하시오.

552. 5 2
. 전체집합 g $ /+0 %0 50 . 1 의 두 부분집합 L 0 Y 에
대하여 L ∪Y $ g 0 J 3L ∩Y 4 $ + 이다. 함수 2 : L → Y 가
일대일함수일 때, 함수 2 의 개수를 구하시오.
556. 함 수 # $ ) & * +) ' )& ' % ) 의 그래프와 직선
5 6 .

]& ' # * 5] ' . $ 9 이 서로 다른 세 점에서 만나도록 하는 실수


] 의 값의 범위를 구하시오.
SSEN 수학(하) 15. 함수
SSEN 수학 (하) 5. 유리식과 유리함수

[ !"0" ∼ !"00 ] 다음 식을 계산하시오.


A단계 기본 다잡기
()' (,*
565. 6 5 .
5

& ,
(,+ &(,'

5-1 유리식의 뜻과 성질

(' ) 5( ) # (' ) '( , '


[ !""# ∼ !""% ] 보기의 식에 대하여 다음에 답하시오.
566. 6 6 .
5

& ()*
,&
(,*
| 보 기 |
()* ( * (' , "(
ㄱ. & ㄴ. & ) & ㄷ. &
'( ' + +
(,* * [ !"0# ∼ !"0% ] 다음 식을 계산하시오.
ㄹ. & ㅁ. ( , &
(-( ) *. (
* *
567. 6 7 .
5

& )
-( ) * .-( ) ' . &-( ) ' .-( ) + .
557. 5 7
. 다항식인 것만을 있는 대로 고르시오.

558. 다항식이 아닌 유리식인 것만을 있는 대로 고르시오. * '


568.
5 8
.

& )
(-( ) *. &-( ) *.-( ) +.
6 8 .
5

[ !""/ ∼ "0! ] 다음 유리식을 통분하시오.


3 *
559. & 4 &'
' [ !"0/ ∼ !"#! ] 다음을 간단히 하시오.
5 9
.

'1 (2 +13(
*
569. 6 9 .
5

& *
*, &
()* (,* (
560. & 4 &
(' , +( ) ' (' , 5
6 0
5 .

*
(, &
(
570. 7 0 .
5

&
( )*
[ !"0* ∼ !"0' ] 다음 유리식을 약분하시오.
& (
*%('26 +
561. &
'5('2'6 '
6 1
5 .

5-3 비례식의 성질

(+ , '(' , %(
562. & (' , 5(
6 2
5 .

'( , 2
571. 7 1 .
5 ( 9 2 : ' 9 + 일 때, & 의 값을 구하시오.
() 2

5-2 유리식의 계산

[ !"0+ ∼ !"05 ] 다음 식을 계산하시오. 1' , 13 ) 3'


(,+ "( ) +
572. 1 9 3 : + 9 5 일 때, & 의 값을 구하시오.
563. 1' ) 3'
7 2 .
5

& ) '
(,* &
6 3
5 .

( ) '( , +

( 2 6 -( , '2 ) +6.'
(,+ ' '
+( ) '( , % +( ) "( , *' 573. & : & : & 일 때, & 의 값을 구하시오.
564. ' 5 " (' ) 2 ' ) 6 '
7 3 .
5

& × ÷
(' ) ( , ' &'(' ) ( , * & (' , *
6 4
5 .

- 1 -
SSEN 수학 (하) 5. 유리식과 유리함수

;
5-4 유리함수의 뜻 16-6 유리함수 2 : & ) > -; ≠ !. 의 그래프
☯ ☯

(,=

[ !"#5 ∼ !"#" ] 보기의 함수에 대하여 다음에 답하시오. *


584. 함수 2 : & 의 그래프를 ( 축의 방향으로 * 만큼, 2 축의
8 4 .
5

(
| 보 기 |
방향으로 '만큼 평행이동한 그래프의 방정식을 구하시오.
() * 5 "( , "
ㄱ. 2 : & ㄴ. 2 : & ㄷ. 2 : &
+ ( ()'
* (' ) +
ㄹ. 2 : '(' , & ㅁ. 2 : &
+ '( , *

574. 다항함수인 것만을 있는 대로 고르시오.


'
585.
7 4
5 .

함수 2 :, & 의 그래프를 ( 축의 방향으로 , ' 만큼, 2 축의


(
8 5 .
5

방향으로 , *만큼 평행이동한 그래프의 방정식을 구하시오.


575. 7 5
5 . 다항함수가 아닌 유리함수인 것만을 있는 대로 고르시오.

[ !"#0 ∼ !"#/ ] 다음 함수의 정의역을 구하시오. [!"%0∼!"%/ ] 다음 함수의 그래프를 그리고, 정의역과 치역을
* 구하시오.
576. 2:& +
'( ) +
586.
7 6
5 .

2 : &,'
(
8 6 .
5

'( ) *
577. 2:& *
(,"
587.
7 7
5 .

2: &
( )*
8 7 .
5

*
578. 2:& *
(' , 5 588.
7 8
5 .

2 : &) +
( )'
8 8 .
5

'(
579. 2:& '
(' ) * 589.
7 9
5 .

2 :, & , *
(, '
8 9 .
5

;

[!"/!∼!"/+ ] 다음 함수의 그래프를 그리고, 점근선의 방정식을


5-5 유리함수 2 : & -; ≠ ! . 의 그래프 구하시오.
( '( ) *
590. 2: &
(,*
9 0 .
5

[ !"%! ∼ !"%+ ]
'
580. 8 0
5 . 2:&
(
591. 9 1 .
5 2: &
(
+,(
*
581. 8 1
5 . 2 :, &
(

'( ) +
582. 2:&
* 592. 9 2 .
5 2: &
'( , *
'(
8 2
5 .

5
583. 2 :, & +( ) '
( 593. 2: &
8 3
5 .

+( ) *
9 3 .
5

- 2 -
SSEN 수학 (하) 5. 유리식과 유리함수

598.

B단계 유형 뽀개기 9 8 .
5 1' ) 3' : "4 13 : +일 때,

-& )
1, 3 &1 ) 3 . -&
1) 3 1, 3
÷ ,
1, 3 &1) 3 .
1) 3 1, 3
의 값을 구하시오.

유형 1 유리식의 덧셈과 뺄셈

(,* * +
594. & , ) + 을 계산하시오.
(' , ( ) * &()* &
9 4
5 .

( )*

@,A
599. 두 다항식 @ 4 A 에 대하여 @ △A : & 라 할 때,
@)A
9 9 .
5

-('△(.÷ C('△-, "( ) 0.D을 계산하시오.


1 3 ?
595. 9 5
5 .

& )
-1 , 3.-1 , ?. &
)
-3 , 1.-3 , ?. &-? , 1.-? , 3.

계산하면?
① ,' ② ,* ③ !
④ * ⑤ '

유형 3 유리식과 항등식

* * * *
596. ) , & , & 을 계산하시오.
&
(,' &
( ()* ( )+ 600. 다음 식의 분모를 ! 으로 만들지 않는 모든 실수 (에
9 6
5 .

0 0 .
6

대하여
()# 1 3( ) ?
& : ) '
(+ ) * & ()* & ( ,()*
가 성립할 때, 1 , 3 , ? 의 값은? (단, 14 34 ? 는 상수이다.)
① ,+ ② ,' ③ ,*
④ * ⑤ '

유형 2 유리식의 곱셈과 나눗셈


1' , 01 1' ) "1 ) 0 1' , +1 , *%


597. 9 7
5 .

&'
× & ÷ & 을 계산하시오.
1) * 1, *
1 ) 1, '
601. 다음 식의 분모를 ! 으로 만들지 않는 모든 실수 (에
0 1 .
6

대하여
* 1 3 ?
& : ) &) &
(-( ) *.' &( ( ) * -( ) *.'
가 성립할 때, 13? 의 값을 구하시오 . (단 , 14 34 ?는 상수이다.)

- 3 -
SSEN 수학 (하) 5. 유리식과 유리함수

유형 5 부분분수로의 변형

602. 0 2
6 . ( ≠, *4 ( ≠ * 인 모든 실수 ( 에 대하여
* * ' 5 1
& ,
(,* &
, '
()* &
, &: &
( ) * (5 ) * (3 , * 606. 다음 식의 분모를 ! 으로 만들지 않는 모든 실수 (에
0 6 .
6

가 성립할 때, 13의 값을 구하시오. (단, 14 3 는 상수이다.) 대하여


' + 5
& ) )
(-( ) '. & -( ) '.-( ) ". &-( ) ".-( ) /.
1
:&
(-( ) 3.
가 성립할 때, 1 ) 3 의 값을 구하시오 . (단 , 14 3 는 상수이다.)
603. 0 3
6 . ( ≠ *인 모든 실수 (에 대하여
(/ ) * 1* 1' 1/
& *!
: &) & ) ⋯ )&
-( , *. ( , * -( , *.'
-( , *./
가 성립할 때, 1* ) 1' ) ⋯ ) 1/의 값은? (단, 1*4 1'4 ⋯4 1/ 는
상수이다.)
① "!# ② "*+ ③ "*# *
607. 자연수 F 에 대하여 G-F.: & 이라 하자.
0 7 .
6

-( ) F.-( ) F ) *.
④ "'* ⑤ "'"
등식
?
G-+.) G-5.) G-".) ⋯ ) G-*!.: &
-( ) 1.-( ) 3.
가 분모를 ! 으로 만들지 않는 모든 실수 (에 대하여 성립할 때,
1 ) 3 , ? 의 값을 구하시오. (단, 14 34 ? 는 상수이다.)

유형 4 (분자의 차수) ≥ (분모의 차수)인


유리수

( '( ) + ( ) + '( , +
604. 0 4
6 .

& )
(,* &()* &
, ,
()' & (,'
* * * *
:&
-( ) '.-( ) *.-( , *.-( , '.
608. 0 8 .
6

& )
% &
)
'5 & 5%
) ⋯ ) & 의 값은?
55!
일 때, □안에 알맞은 식은? ' / "
① & ② & ③ &
① , 0( ② 0( ③ 0( ) " ** 55 ''
④ (' , 0( ⑤ (' ) '( , 0 * +
④ & ⑤ &
5 **

(+ (+
605. 0 5
6 .

&'
)&'
, '( 를 계산하시오.
609. G-(.: 5(' , *일 때,
( )()* ( ,()* 0 9 .
6

* * * *
& ) ) ) ⋯ )&
G-*. &G-'. &G-+. G-5/.
의 값을 구하시오.

- 4 -
SSEN 수학 (하) 5. 유리식과 유리함수

유형 6 번분수식 유형 7 분수를 번분수로 나타내기


☯ ☯

* +! *
610. 1 0
6 . * , & 을 간단히 하시오.
*
614. 1 4 .
6

&
**
: 1 ) & 을 만족시키는
*
*, & 3) &
* *
*, & ?) &
* *
*, & H) &
( I
자연수 14 34 ?4 H4 I 에 대하여 1 ) 3 ) ? ) H ) I 의 값을 구하시오.

* **
611. 1 1
6 . G-(.: * ) & 에 대하여 G-;.: & 을 만족시키는
* 0 5+ *
*) &
*)(
615. 1 5 .
6

&
*/
: 1 ) & 을 만족시키는 자연수 14 34 ?4 H 에
*
3) &
상수 ;의 값은? *
?) &
① * ② ' ③ + H
④ 5 ⑤ " 대하여 13?H 의 값을 구하시오.

* *
& ,
F) ' &F) "
612. 1 2
6 .

&* *
이 자연수가 되도록 하는 정수 F 의 개수를
,& *
유형 8 유리식의 값 ; (± &의 값 이용

&
F) " F) %
(
구하시오.
' +
616. 1 6 .
6 (' , '( , * : ! 일 때, +(' ) '( , * , & ) &' 의 값을
( (
구하시오.

613. 1 3
6 .
자연수 F 에 대하여 1F 을
* * * 617. 1 7 .
6 ! 이 아닌 두 실수 1 4 3 에 대하여 1' , +13 ) 3' : ! 이
1* : & 4 1' : & 4 1+ : & 4 ⋯
5 * * 1+ 3+
*, & *, & 성립할 때, & ) &+ 의 값은?
5 * +
3 1
*,&
5
① 0 ② *' ③ *%
이라 할 때, 1'!*%1'!*/ 의 값을 구하시오.
④ '5 ⑤ +0

- 5 -
SSEN 수학 (하) 5. 유리식과 유리함수

유형 10 유리식의 활용

*
618. 1 8
6 .
' 5
( ) +( ) * : ! 일 때, ( , &5 의 값을 구하시오.
(
(단 , , * K ( K ! )
622. 2 2 .
6 수질 오염의 정도를 수치로 나타내는 한 방법으로
생물학적 지표가 이용된다. 이 지표는 유색 생물의 수가 L 이고
M
무색 생물의 수가 M 일 때, & × *!!-N.으로 정의된다. 지난달
L)M
수질 검사에서 어떤 호수의 생물학적 지표는 '!N 이었고, 이번 달에
이 호수의 수질을 검사한 결과 지난달에 비해 유색 생물의 수는 2배,
무색 생물의 수는 4배가 되었다. 이번 달 이호수의 생물학적 지표는
유형 09 유리식의 값; 1 ) 3 ) ? : ! 이용

약 몇 N 인지 소수점 아래 첫째 자리에서 반올림하여 구하시오.

619. 1 9
6 . 1 ) 3 ) ? : ! 일 때,
* *
3 ? - * *
? 1 . -
* *
1 &) & ) 3 &) & ) ? & ) &
1 3 . - .
의 값을 구하시오. (단, 13? ≠ ! )

623. 2 3 .
6 어떤 오렌지 주스 제조 공장에서 오렌지 원액 1N 가
포함되어 있는 오렌지 주스 *!! L와 오렌지 원액 3N 가 포함되어
있는 오렌지 주스 ( L 를 섞어 오렌지 원액 ?N 가 포함되어 있는
새로운 오렌지 주스를 만들려고 한다. 이때 ( 를 1 4 3 4 ? 에 대한
식으로 나타내면?
(단, 3 ≠ ? )
*!!? , 1 *!!-? , 1. *!!?
* * * ① & ② & ③ &
620. 2 0
6 . ! 이 아닌 세 실수 14 3 4 ? 에 대하여 & ) & ) & : ! 일
1 3 ?
3 3 ) *!!? *!!1 ) 3
*!!-? , 1. ?,1
1 3 ? ④ & ⑤ &
때, & ) & ) & 의 값은? 3,? *!! -3 , ? .
-1 ) 3 .-1 ) ?. -3 ) ?.-3 ) 1. -? ) 1.-? ) 3.
① ,' ② ,* ③ !
④ * ⑤ '

624. 2 4 .
6
A 회사의 휴대폰 단말기의 가격을 =N 인상한 후, >N 를
다시 인상하여 판매한다면 처음 가격의 (N 를 인상한 것과 같다.
이때, ( 를 = 4 > 에 대한 식으로 나타내면?
621. 2 1
6 . ! 이 아닌 세 실수 1 4 3 4 ? 에 대하여
=)>
' ① =)> ② & ③ =>
* * *
&' ) &' ) &' : &
1 3 ?
-
* * *
) )&
1 &3 ? . '
=>
④ = ) > ) => ⑤ =)>) &
1+ ) 3+ ) ?+ *!!
일 때, & 의 값을 구하시오.
13?

- 6 -
SSEN 수학 (하) 5. 유리식과 유리함수

유형 11 유리식의 값; 비례식이 주어질 때


* 5 5
629. 2 9 .
6
( ) & : * 4 '2 ) & : * 일 때, & ) 6 의 값은? (단,
'2 6 (
(26 ≠ ! )
()2 2)6 6)(
625. 2 5
6 . 세 실수 ( 4 2 4 6 가 & : & : & 를 만족시킬
+ 5 "
① * ② ' ③ +
④ 5 ⑤ "
(26
때, &
+ + + 의 값을 구하시오 . (단 , (26 ≠ ! )
( )2 )6

626. 2 6
6 . ! 이 아닌 세 실수 ( 4 2 4 6 에 대하여 ( 9 2 9 6 : + 9 5 9 " 일 630.
(,2
'( ) 2 , +6 : ! , ( , +2 ) 6 : ! 일 때, & 의 값을
()6
3 0 .
6

, '( ) +2 , 6
때, & 의 값은? 구하시오. (단 , (26 ≠ ! )
(,2)6
* * *
① & ② & ③ &
" 5 +
*
④ & ⑤ *
'

유형 13 비례식을 만족시키는 상수 구하기


627. 2 7
6 . ! 이 아닌 세 실수 (4 2 4 6 에 대하여 '( : +2 4 52 : "6 일
+( , 2 , +6 +1 , 3 '3 , ? 5? ) +1 01 ) +3 ) '?
때, & 의 값을 구하시오.
()2 )6 631. 3 1 .
6

& 5
: & : & : & 일 때, 상수
+ ' ;
; 의 값은? (단, 13? ≠ ! 4 ; ≠ ! )
① / ② ** ③ *'
④ *+ ⑤ *%

유형 12 유리식의 값; 방정식이 주어질 때


(2 ) 26 ) 6(
628. ( ) 2 , '6 : ! , +( , +2 ) '6 : !일 때, & 의
632. ( ) '2 +2 , '6 6 ) ( 5( ) "6
: & : & : & 일 때, 상수 ; 의
(' ) 2 ' ) 6 '
2 8
6 .

3 2 .
6

& + 5 " ;
값을 구하시오. (단, (26 ≠ ! ) 값을 구하시오. (단, (26 ≠ ! , ; ≠ ! )

- 7 -
SSEN 수학 (하) 5. 유리식과 유리함수

유형 15 유리함수의 정의역과 치역

633. 3 3
6 . 세 실수 1 4 3 4 ? 에 대하여
'3 ) +? +? ) 1 1 ) '3
& : &: &: ;
1 '3 +?
'( ) *
를 만족시키는 모든 ; 의 값의 합을 구하시오. (단, 13? ≠ !) 637. 3 7 .
6 함수 2 : & 의 치역이 C2 T ' K 2 ≤ "D일 때,
(,*
정의역은?
① C ( T ( ≤, ' D ② C(T ,'≤(K * D
③ C( T * K ( ≤ ' D ④ C(T (≥' D
⑤ C( T ,' ≤ ( K * 또는 *K(≤' D

유형 14 비례식의 활용

3( ) 5
638. 함수 2 : & 의 정의역이 C ( T( ≠ * 인 실수 D, 치역이
634. 1, (
3 8 .
6

3 4
6 .
신입생 정시 모집에서 A 4 B 두 학교의 지원자 수의 비는
* 9 ' , 합격자 수의 비는 + 9 5 , 불합격자 수의 비는 ' 9 " 일 때, C 2 T 2 ≠, 5 인 실수 D일 때 , 상수 1 4 3 에 대하여 13 의 값을
A 학교의 합격률은? 구하시오.
* ' +
① & ② & ③ &
# # #
5 "
④ & ⑤ &
# #

635. 넓이가 각각 @ 4 A 4 R 4 S 인
유형 16 유리함수의 그래프의 평행이동

3 5
6 .

정사각형을 오른쪽 그림과 같이 빈틈없이 붙여


놓았을 때, @ 9 S 는?
① %9 * ② /9* ③ *0 9 * 639. 3 9 .
6 보기의 함수 중 그 그래프가 평행이동에 의하여 함수
④ '" 9 * ⑤ +0 9 *
*
2 : & 의 그래프와 겹쳐지는 것만을 있는 대로 고른 것은?
"(

| 보 기 |
* *!( ) *
ㄱ. 2 : & ㄴ. 2 : &
"( , " "(
*!( ) + (,'
ㄷ. 2 : & ㄹ. 2 : &
"( ) " " , "(
636. 3 6
6 . * 학년과 '학년으로 구성된 어느 고등학교 방송반에서
① ㄱ, ㄴ ② ㄴ, ㄷ ③ ㄷ, ㄹ
*학년의 남학생과 여학생 수의 비는 * 9 '이고 ' 학년의 남학생과
④ ㄱ, ㄴ, ㄹ ⑤ ㄱ, ㄷ, ㄹ
여학생 수의 비는 * 9 " 이며 방송반 전체의 남학생과 여학생 수의
비는 5 9 ** 이라 한다. 이 고등학교 방송반의 전체 학생 수에 대한
*학년 학생 수의 비율을 구하시오.

- 8 -
SSEN 수학 (하) 5. 유리식과 유리함수

+( , ' 3( , *
643. 두 함수 2 : & 4 2 : & 의 그래프의 점근선이
640. ,()' '( ) 1
4 3 .
6

' '
4 0
6 .
원 -( ) '. ) -2 , +. : 5 가 점 - * 4 , ' . 를 중심으로
* 같을 때, 상수 14 3 에 대하여 13의 값을 구하시오.
하는 원으로 옮겨지는 평행이동에 의하여 함수 2 : & 의 그래프가
(
3( ) ?
함수 2 : & 의 그래프와 겹쳐질 때, 상수 1 4 3 4 ? 에 대하여
()1
1 ) 3 ) ? 의 값을 구하시오.

1( ) *
644. (
두 함수 2 : , & 4 2 : & 의 그래프의
()1 (,'
4 4 .
6

점근선으로 둘러싸인 도형의 넓이가 '! 일 때, 양수 1 의 값을


구하시오.

3( ) #
641. 4 1
6 .
함수 2 : & 의 그래프를 ( 축의 방향으로 + 만큼,
()1
*
2축의 방향으로 , ' 만큼 평행이동하면 함수 2 : & 의 그래프와
(
일치 할 때, 상수 1 4 3 에 대하여 1 ) 3 의 값은?
① * ② ' ③ +
④ 5 ⑤ "

유형 18 유리함수의 그래프의 대칭성


1( ) '3
645. 함수 2 : & 의 그래프가 2 축과 만나는 점의 2 좌표가
4 5 .
6

'( ) ?
* 이고

- *
.
점 , *4 , & 에 대하여 대칭일 때, 1 ) 3 ) ?의 값은?
'
(단 , 14 34 ? 는 상수이다.)
① * ② ' ③ +
④ 5 ⑤ "
유형 17 유리함수의 그래프의 점근선

1( ) 3
642. 4 2
6 .
함수 2 : & 의 그래프가 점 -'4 *. 을 지나고, 점근선의
()?
방정식이 ( : , *4 2 : ' 일 때 , 상수 14 34 ? 에 대하여
5( , +
1' ) 3' ) ?'의 값을 구하시오. 646. 함수 2 : & 의 그래프가 점 -14 3. 에 대하여 대칭이고,
4 6 .
6

'( ) *
직선 2 : ( ) ? 에 대하여 대칭일 때, 13? 의 값을 구하시오.
(단 , 14 34 ? 는 상수이다.)

- 9 -
SSEN 수학 (하) 5. 유리식과 유리함수

유형 04 그래프를 이용하여 유리함수의 식


3( ) '
647. 함수 2 : & 의 그래프가 두 직선
()1
구하기
4 7
6 .

2 : ( ) +4 2 : , ( , '에 대하여 대칭일 때, '3 , 1 의 값을


1( ) 3
구하시오.
651. 함수 2 : & 의 그래프가 오른쪽
()?
5 1 .
6

(단, 14 3는 상수이다.)
그림과 같을 때, 상수 14 34 ? 에 대하여 13? 의
값을 구하시오.

+( , "
648. 4 8
6 . 함수 2 : & 의 그래프와 중심의 좌표가 -'4 +. 인 원이
(,'
서로 다른 네 점에서 만날 때, 네 교점의 (좌표를 각각
(*4 ('4 (+4 (5 라 하자. 이때 (* ) (' ) (+ ) (5 의 값을 구하시오.

;
652. 함수 2 : & ) 3 의 그래프가
5 2 .
6

() 1
오른쪽 그림과 같을 때, 상수 14 34 ; 에
대하여 1 ) 3 ) ; 의 값은?
① * ② +
유형 19 유리함수의 그래프가 지나는 사분면

③ " ④ #
⑤ /
, '( , *
649. 함수 2 : & 의 그래프가 지나지 않는 사분면을 모두
()'
4 9
6 .

말하시오.

3
653. 함수 2 : & ) ? 의 그래프가
650. (, 1
5 3 .
6

5 0
6 . , *! 이상 *! 이하의 정수 14 34 ?4 H에 대하여 함수
오른쪽 그림과 같을 때, 옳은 것만을 보기에서
1( ) 3
2 : & 의 그래프의 점근선의 방정식이 ( : , '4 2 : 5 이다. 있는 대로 고른 것은?
?( ) H
(단, 14 34 ? 는 상수이다.)
이 그래프가 제 *4 '4 +4 5 사분면을 모두 지날 때, 1 ) 3 ) ? ) H 의
최솟값을 구하시오. (단, 1H , 3? ≠ !4 ? ≠ ! )
| 보 기 |
3 1
ㄱ. 1 , ? W ! ㄴ. 3 : 1? ㄷ. & ) & : !
1 ?

① ㄱ ② ㄴ ③ ㄱ, ㄴ
④ ㄱ, ㄷ ⑤ ㄱ, ㄴ, ㄷ

- 10 -
SSEN 수학 (하) 5. 유리식과 유리함수

유형 21 유리함수의 그래프의 성질 유형 22 유리함수의 최대∙ 최소


☯ ☯

* '( , *
654. , +( , "
함수 2 : & 에 대한 설명으로 옳지 않은 것은? 657. 5 7 .
6
, ' ≤ ( ≤ & 에서 함수 2 : & 의 최댓값을 1,
' (,*
()'
5 4
6 .

① 그래프는 점 -, '4 , +. 에 대하여 대칭이다. 최솟값을 3라 할 때, 1 ) 3 의 값을 구하시오.


② 정의역은 C( T ( ≠ , '인 실수D이다

③ 그래프와 ( 축의 교점의 좌표는 , & 4 ! 이다.


+ - "
.
④ 그래프는 제 *4 '4 + 사분면을 지난다.
*
⑤ 그래프는 함수 2 : & 의 그래프를 평행이동한 것이다.
(

+
+ 658. 정의역이 C( T , * ≤ ( ≤ 'D인 함수 2 : & ) 1 의
5 8 .
6

( )'
') &
(
655. 함수 G -(.: & 에 대하여 옳은 것만을 보기에서 있는
' +
최솟값이 , & 일 때, 실수 1 의 값은?
5 5
6 .

*, & 5
(
+ *
대로 고른 것은? ① ,& ② ,* ③ ,&
' '
| 보 기 | * +
④ & ⑤ &
ㄱ. 정의역은 C( T ( ≠ '인 실수D이다. ' '
ㄴ. 함수 2 : G-(.의 그래프는 모든 사분면을 지난다.
ㄷ. 함수 2 : G-(.의 그래프는 직선 2 : (에 대하여 대칭이다.

① ㄱ ② ㄴ ③ ㄷ
④ ㄱ, ㄴ ⑤ ㄴ, ㄷ

659. 유리함수 2 : G-(. 가 다음 조건을 모두 만족시킬 때,


5 9 .
6

, + ≤ ( ≤ ' 에서 2 : G-(. 의 최댓값과 최솟값의 합을 구하시오.

656. 오른쪽 그림은 네 함수


(가 ) 점근선의 방정식은 ( : +4 2 : *이다.
5 6
6 .

1 3 ? H (나 ) 그래프는 원점을 지난다.


2 : &4 2 : &4 2 : &4 2 : &
( ( ( (
의 그래프의 일부이다 . 이때 실수
14 34 ?4 H 의 대소를 비교하시오.

- 11 -
SSEN 수학 (하) 5. 유리식과 유리함수

유형 23 유리함수의 그래프와 직선의 유형 24 유리함수의 합성


☯ ☯

위치관계
(,' (,*
660. 6 0
. 함수 2 : & 의 그래프와 직선 2 : ;( ) * 이 한 점에서
()*
664. 함수 G -( . : & 에 대하여
6 4 .

(
만날 때, 양수 ;의 값은? G* : G, GF ) * -(. : -GF ∘ G.-(. ( F 은 자연수)
① / ② *! ③ ** 로 정의할 때, G
*!!
-% . 의 값을 구하시오.
④ *' ⑤ *+

(
661. 함수 2 : & 의 그래프와 직선 2 : Y( , 'Y 이 만나지
()* *
(,' 665.
6 1
.

함수 G -(.: & 에 대하여 -G ∘ G .-;.: & 을 만족시키는


(,* ;
6 5 .

않도록 하는 정수 Y 의 최댓값은?
① ," ② ,5 ③ ,+ 실수 ; 의 값은?
④ ,' ⑤ ,* ① ,5 ② ,' ③ ,*
④ ' ⑤ 5

(
666. 함수 G -( . : & 에 대하여
6 6 .

*,(
662. 6 2
.

C Z +( , *
두 집합 @ : -(4 2. 2 : & 4
( D G* : G, GF : G ∘ G ∘ G ∘⋯∘ G (F : '4 +4 54 ⋯ )
_^` ^a
A : C-(4 2. T 2 : 1( ) +D에 대하여 @ ∩A ≠ ∅ 일 때, 실수 1 의 F개

값의 범위를 구하시오. 1( ) 3
로 정의한다. G *! -(. : & 일 때 , 상수 14 34 ?에 대하여
?( ) *
1 ) 3 ) ? 의 값을 구하시오.

667. 오른쪽 그림은 유리함수


6 7 .

2 : G-(. 의 그래프이다.
'( ) 5 G* : G
663. * ≤ ( ≤ + 에서 함수 2 : & 의 그래프와 직선
GF : G ∘ GF , * ( F : '4 +4 54 ⋯ )
()*
6 3
.

'!*%
2 : 1( ) ' 가 한 점에서 만날 때, 실수 1 의 최댓값과 3 의 최솟값의 로 정의할 때, G -* . 의 값을 구하시오.
합을 구하시오.

- 12 -
SSEN 수학 (하) 5. 유리식과 유리함수

유형 25 유리함수의 역함수

()*
671. 함수 G -(.: & 에 대하여 함수 b-(.: G-( ) +., ' 가
7 1 .
6

1( ) 3
1(
668. 6 8
. 함수 G -( . : & 에 대하여 G : G
'( ) +
,*
가 성립할 때, 상수 b-'.: 5 , b : b, *를 만족시킬 때, 1 , 3의 값은?
(단, 14 3 는 상수이다.)
1의 값은? ① ,' ② ,* ③ !
① ,+ ② ,' ③ ,* ④ * ⑤ '
④ * ⑤ '

유형 26 유리함수의 합성함수와 역함수


1( ) 3
669. 6 9
. 함수 G -( . : & 의 그래프와 그 역함수의 그래프가
(,' 672. '( , +
함수 G-(. : & 에 대하여 -G
,*
∘ G ∘ G, *.-".의
(,+
7 2 .
6

모두 점 -+4 , '. 를 지날 때 , 상수 14 3 에 대하여 1 ) 3 의 값은? 값을 구하시오.


① , *! ② ,% ③ ,0
④ ,5 ⑤ ,'

'( , 5
673. 함수 G-(. : & 일 때, -G ∘ b.-(.: (를 만족시키는
7 3 .
6

(,*
함수 b -( . 에 대하여 -b ∘ b.-+.의 값을 구하시오.

670. 7 0
6 . 유리함수 2 : G -(. 의 그래프가 두 직선 2 : ( ) " ,
2 :, ( , + 에 대하여 대칭일 때, 그 역함수의 그래프는 두 직선
2 : 1( ) 3 , 2 : ?( ) H 에 대하여 대칭이다. 이때 상수 14 34 ?4 H 에
대하여 1 ) 3 ) ? ) H 의 값을 구하시오.

( '( , *
674. 함수 G-(. : & 와 b-(. : & 에 대하여
7 4 .
6

(,* (
-G, * ∘ b., *-+.의 값은?
+
① * ② & ③ '
'
"
④ & ⑤ +
'

- 13 -
SSEN 수학 (하) 5. 유리식과 유리함수


678. 서로 다른 네 양수 14 34 ?4 H 에 대하여 1H , 3? : ! 일 때,
7 8 .
6

C단계 실력 굳히기 옳은 것만을 보기에서 있는 대로 고른 것은?

| 보 기 |

675. 7 5
6 . 전기회로에서 크기가 c* -Ω .4
3 H
ㄱ. & : &
1 ?
c' -Ω. 인 두 저항을 연결하였을 때, 전체
1' ) ?' '1?
ㄴ. & : &
저항의 크기 c -Ω.에 대하여 직렬 13 ) ?H 1H ) 3?
연결이면 c : c* ) c'4 병렬 연결이면 3+ ) H+ -3 ) H.+
ㄷ. & :
* * * +
) + &
1 ? - ) .+ 1 ?
& : ) 이 성립한다 . 그림과 같이
c & c* &c'
① ㄱ ② ㄱ, ㄴ ③ ㄱ, ㄷ
크기가 각각 c -Ω.4 c ) *-Ω.4 'c -Ω. 인
④ ㄴ, ㄷ ⑤ ㄱ, ㄴ, ㄷ
세 저항을 연결한 전기회로의 전체 저항의 크기는?
'c ' ) c 'c ' ) +c
① &Ω ② &Ω
'c ) * 'c ) *
+c ' ) 5c "c ' ) +c
③ &Ω ④ &Ω
+c ) * +c ) *
"c ' ) 0c
⑤ &Ω
+c ) *

679. 7 9 .
6 !이 아닌 두 실수 14 3 에 대하여 1' , 513 ) +3' : ! 이
-1 ) 3.'
성립한다. &
' '
: ;일 때, 모든 ;의 값의 합을 구하시오.
1 )3
676. 7 6
6 . 다항식 G-(. 가 *!! 이하의 자연수 F 에 대하여
G-F.≠, *4 G-F. G-*!* , F.: *
을 만족시킬 때,
* * * *
& ) ) )⋯) &
* ) G-*. &* ) G-'. &* ) G-+. * ) G-*!!.
의 값을 구하시오.

677.
*
( ) & : * 일 때, 옳은 것만을 보기에서 있는 대로 고른
680. 키가 *%! cm 인 아버지가 가로등에서 0 m 떨어진 곳에
8 0 .
6

(
7 7
6 .

서고, 키가 *"! cm 인 연준이가 아버지보다 1 m 앞에 섰을 때,


것은? 아버지의 그림자 끝과 연준이의 그림자 끝이 일치하였다. 연준이의
| 보 기 | 3f
그림자의 길이가 3 m, 가로등의 높이가 f m일 때, & 의 값을
1) *
* *
ㄱ. * , & ) & :! 구하시오.
( '
(

- (
'
ㄴ. ( ) & , ( ) &
*
' .- *
) (+ ) &+ : !
( .- *
( .
+F ) ' * *
ㄷ. ( , (+F ) * ) & +F) '
,& +F) *
:, ' (단, F은 자연수)
( (

① ㄱ ② ㄷ ③ ㄱ, ㄴ
④ ㄴ, ㄷ ⑤ ㄱ, ㄴ, ㄷ

- 14 -
SSEN 수학 (하) 5. 유리식과 유리함수

+( ) ;
681. 유리함수 G-(. : & 의 그래프를 ( 축의 방향으로
( )5 684.
8 1
6 .

오른쪽 그림과 같이 함수
8 4 .
6

, ' 만큼, 2 축의 방향으로 + 만큼 평행이동한 곡선을 2 : b -( . 라 *


하자. 곡선 2 : b -( . 의 두 점근선의 교점이 곡선 2 : G -( . 위의 2 : & -( W ! . 의 그래프 위의 한 점을
(
점일 때, 상수 ; 의 값은? 꼭짓점으로 하는 정사각형 ABCD 가 있다.
① ,0 ② ,+ ③ ! 정사각형 ABCD의 한 변의 길이가 '일
④ + ⑤ 0 때, 점 D의 자취의 방정식은
1( , +
2 : & -( W ' .이다.
()3
이 때, 상수 14 3 에 대하여 1 ) 3 의 값을 구하시오.
(단, 정사각형의 모든 변은 ( 축 또는 2축과 평행하다.)

+
682. 8 2
6 .
좌표평면에서 함수 2 : & ) ; 의 그래프가 직선
(,"
685. 양의 실수 전체의 집합에서 정의된 함수
2 : ( 에 대하여 대칭일 때, 상수 ;의 값은?
8 5 .
6

',(
① * ② ' ③ +
G-(. : ∣&
( ∣
에 대하여 1 K 3 인 두 양수 14 3가

G -1 . : G -3 . 를 만족시킬 때, 옳은 것만을 보기에서 있는 대로 고른


④ 5 ⑤ "
것은?

| 보 기 |
ㄱ. ! K 1 K '
ㄴ. ! K G -3 . K *
' -1 ) 3 .
ㄷ. G -1 . ) G -3 . : &
13

① ㄱ ② ㄴ ③ ㄱ, ㄴ
④ ㄴ, ㄷ ⑤ ㄱ, ㄴ, ㄷ
;
683. 8 3
6 . 함수 2 : & ) % 의 그래프는 제 *4 '4 5 사분면만을
(,5
;
지나고 함수 2 : & , +의 그래프는 모든 사분면을 지나도록
()"
하는 정수 ;의 개수는?(단, ; ≠ ! )

① ** ② *+ ③ *"
④ *# ⑤ */
686. 오른쪽 그림과 같이
8 6 .
6

'
함수 2 : & -( W !. 의
(
그래프 위의 두 점
A -143. 4 B-?4 H.에 대하여
직선 AB 의 기울기가
*
, & 이다. 네 점 R -!4 H . , S -14 ! .4 i -14 H .4 j -?4 3 . 에 대하여
0
두 사각형 CODE 와 AEBF 의 넓이의 비가 * 9 5 일 때,
1 ) 3 ) ? ) H 의 값을 구하시오.(단, 1 K ? 이고 , n 는 원점이다.)

- 15 -
SSEN 수학 (하) 5. 유리식과 유리함수

()1 , ()*
687. 8 7
6 .
함수 2 :
∣&
(,5 ∣
의 그래프가 직선 2 : ( 와 서로 다른 691. 좌표평면 위에 점 P -, '4 , *. 과 곡선 2 : & 위를
9 1 .
6

()'
세 점에서 만나도록 하는 정수 1 의 개수를 구하시오.(단, 1 W, 5. '
움직이는 점 p 가 있다. 선분 PQ 의 길이의 최솟값이 Y 일 때, Y 의
값을 구하시오.

1( ) 0
688. 함수 G-(. : & 이 등식 -G ∘ G.-(. : ( 를 만족시킬
(,3
8 8
6 .

때, 옳은 것만을 보기에서 있는 대로 고른 것은?(단, 13 ≠ !)

'( ) ;
| 보 기 | 692. ! ≤ ( ≤ ' 에서 함수 2 : & 의 최댓값이 * 일 때, 상수
()*
9 2 .
6

ㄱ. 13 ≠, 0
; 의 값을 구하시오.
ㄴ. 1 :, 3
ㄷ. 함수 2 : G -( . 의 그래프는 제 *4 '4 +4 5 사분면을 모두
지난다.

① ㄱ ② ㄱ, ㄴ ③ ㄱ, ㄷ
④ ㄴ, ㄷ ⑤ ㄱ, ㄴ, ㄷ

*
693. 함수 2 : & 의 그래프와 직선 2 : '( ) ; 가 만나는 두 점을
(
9 3 .
6

각각 A4 B 라 하고, 선분 AB의 중점을 P-(4 2 .라 할 때, 점 P 의


자취의 방정식을 구하시오.(단, ; 는 상수이다.)

, 5( ) 02 ) 1
689. 8 9
6 .
두 실수 (4 2 에 대하여 & 의 값이 항상
+( , 32 ) '
일정하고 이 식의 분모를 ! 으로 만들지 않을 때, 1 ) 3 의 값을
구하시오.
(단, 1 4 3 는 상수이다.)

+( ) 1
694. 함수 G -( .=& 에 대하여 G
9 4 .
6

(,'
,*
-( . : -G ∘ G .-( . 를

만족시키는 상수 1 의 값을 구하시오.

*! *!
690. 두 함수 G-(. : & ) *4 b-(. : & , *!
( *! , (
9 0
6 .

에 대하여 2 : G -( .4 2 : b -( .의 그래프와 두 직선 ( : *4 ( : / 로
둘러싸인 도형의 넓이를 구하시오.

- 16 -
SSEN 수학 (하) 17. 무리식과 무리함수

[ !5!4 ~ !5!$] 다음 식을 간단히 하시오.


704.

A단계 기본 다잡기 0 4 .
7
0()
& + * ' ()
& 10()
& + * + ()
&1

705. 0()
& ' , + ()
& + , 10()
& ' , ' ()
&+,1
17-1 무리식의 뜻

0 5 .
7

[ !"#$ ∼ !"#% ] 다음 무리식의 값이 실수가 되도록 하는 실수 & 의


값의 범위를 구하시오.
17-3 분모의 유리화

695. 9 5
6 . & ' ()
&'*
[ !5!" ~ !5!5] 다음 수의 분모를 유리화하시오.
*
696. 9 6
6 .
()
& + , ' ()
& '-
706. 0 6 .
7

)
()-

,
697. ()
&+* + ) 707. )
,
() ()* + ,
9 7
6 .

-+&
0 7 .
7

()
& ', [ !5!% ~ !5,,] 다음 식의 분모를 유리화하시오.
698. 9 8
6 .

)
() ,
$+&
708. )
()
& , ' ()
0 8 .
7

' &

4
17-2 제곱근의 성질

709. )
()
& ' * + ()
0 9 .
7

&+*

699. 9 9
6 . . / ! 일 때, 그 값이 양수인 것만을 보기에서 있는 대로
고르시오. (). + ()
3
710. )
(). ' ()
1 0 .
7

| 보 기 |
3
)
ㄱ. (.*
)
ㄴ. + (.* ㄷ. ()
0+ .1*
)*
ㄹ. + (0+ . 1 ㅁ. 0+ ()
+ . 1* ()
& + , ' ()
&
711. )
()
& + , + ()
1 1 .
7

&

700. . 2 ! , 3 / ! 일 때, ()
.* ' ()
43* ' ()
0. + 3 1* 을 간단히
[ !5,* ~ !5,-] 다음 식을 간단히 하시오.
0 0
7 .

하시오.
, ,
712. ) ')
(). ' () ()
. + ()3
1 2 .
7

3
[ !5!, ~!5!- ] 다음 식을 간단히 하시오.
701. ()
0, + ()
* 1*
&+, &',
713.
0 1
7 .

) + )
&',', )
() (& ' , + ,
1 3 .
7

702. ()
0& + ,! 1* 0& 2 ,! 1
0 2
7 .

()* , ,
714. & 6 ) 일 때, ) ' 의 값을 구하여라.
* & )
, ' () , + ()
1 4 .
7

&
703. ()
0& + ,1* ' ()
0 3
7 . 0& + *1* 0, / & / *1

- 1 -
SSEN 수학 (하) 17. 무리식과 무리함수

[ !5*$ ~ !5*5] 함수 7 6 ()
+ -& 의 그래프를 다음과 같이 대칭이동한
17-4 무리함수의 뜻

그래프의 방정식을 구하시오.


725. & 축에 대하여 대칭이동
715.
2 5 .
7

1 5
7 .
무리함수인 것만을 보기에서 있는 대로 고르시오.

| 보 기 |

ㄱ. 7 6 ()
*& ㄴ. 7 6+ ()
-& )
ㄷ. 7 6 (, + &*
726. 2 6 .
7 7 축에 대하여 대칭이동

) &+,
ㄹ. 7 6 (0& ' *1* ㅁ. 7 6 )
() '
& ,
727. 2 7 .
7 원점에 대하여 대칭이동

[ !5," ~!5,# ] 다음 함수의 정의역을 구하시오.


716. 1 6
7 . 7 6 ()
&',

717. 1 7
7 . 7 6+ ()
+& '*
17-6 무리함수

7 6 ()
. 0& + > 1 ' ? 0. ≠ ! 1 의 그래프
718. 7 6 ()
*& + - ' ,
728.
1 8
7 .

2 8 .
7 함수 7 6 ()
$& 의 그래프를 & 축의 방향으로 + ,만큼,
7 축의 방향으로 , 만큼 평행이동한 그래프의 방정식을 구하시오.
719. 1 9
7 . 7 6 ()
4 + &*

17-5 무리함수 7 6± ()
.& 0. ≠ ! 1 의

그래프 729. 2 9 .
7
함수 7 6 ()
4 + *& ' *의 그래프는 함수 7 6 ()
+ *& 의

[ !5*! ~!5*- ] 다음 함수의 그래프를 그리고 , 정의역과 치역을 그래프를 &축의 방향으로 .만큼, 7 축의 방향으로 3만큼 평행이동한
구하시오. 것이다. 이때 .@ 3 의 값을 구하시오.

720. 2 0
7 . 7 6 ()
&

721. 2 1
7 . 7 6+ ()
&

[ !5-! ~ !5--] 다음 함수의 그래프를 그리고 , 정의역과 치역을


722. 7 6 ()
+&
구하시오.
2 2
7 .

730. 3 0 .
7 7 6 ()
-& + "

723. 2 3
7 . 7 6+ ()
+&

731. 3 1 .
7 7 6+ ()
*& ' - ' ,

724. 2 4
7 . 함수 7 6 ()
.& 0. ≠ !1 에 대하여 옳은 것만을 보기에서
있는 대로 골라라.
732. 3 2 .
7 7 6 ()
,+& +*
| 보 기 |
ㄱ. 정의역은 :& ; & ≥ !=이다.
ㄴ. 치역은 :7 ; 7 ≥ !=이다.
733. 3 3 .
7 7 6+ ()
4 + -& + ,
ㄷ. 그래프는 함수 7 6 ()
+ .& 의 그래프와 7축에 대하여 대칭
이다.
ㄹ. . / !이면 그래프는 원점과 제 , 사분면을 지난다.

- 2 -
SSEN 수학 (하) 17. 무리식과 무리함수

737.

+ , ≤ . ≤ , 이고 & 6 0. ' ,1* 일 때, ()


& ' ()
B단계 유형 뽀개기 간단히 하면?
3 7 .
7 & + 4. 를

① * ② *. ③ *0. ' ,1
* *
④ *. ⑤ * 0. ' , 1
유형 01 무리식의 값이 실수가 되기 위한 ☯

조건

734. ()
"&* ' $& + 4 의
3 4
7 . 값이 실수가 되도록 하는 실수 & 의
값의 범위를 구하시오.

738. 3 8 .
7
*
실수 . 에 대하여 & 6 . ' *@ 7 6 *. 일 때,

()
0& ' 71* + ()
0& + 71* 을 .에 대한 식으로 나타내시오.

,
735. 3 5
7 .
()
4 + -& ' )
()
의 값이 실수가 되도록 하는 정수
&'-
& 의 개수는?
① , ②* ③-
739. 3 9 .
7
()
& ' - + ()
- + & 의 값이 실수가 되도록 하는 실수 & 에
④ 4 ⑤ $
)
대하여 B*& + 5B ' (& + %& ' ," 을 간단히 하시오.
*

유형 03 음수의 제곱근의 성질

740. ()
. + - ()
, + . 6+ ()
0. + -10, + .1 를 만족시키는 실수
유형 02 제곱근의 성질
☯ 4 0 .
7

. 에 대하여 ()
0. ' * 1* ' ()
0. + 4 1* 을 간단히 하면?
① *. *
+ ② . ③ "
,
736. 3 6
7 .
+ ) / . / ,일 때, ()
*
.* + *. ' , ' ()
4.* ' 4. ' , 을 ④ +" ⑤ + *. ' *

간단히 하면?
① * ② + .+ * ③ .' *
④ -. ⑤ -. ' *

- 3 -
SSEN 수학 (하) 17. 무리식과 무리함수

() ) 745.
741. ! 이 아닌 두 실수 . , 3 에 대하여 )
4 1
7 .
.
()3
6+ ().3 일 때,
4 5 .
7 자연수 E 에 대하여 .E이
, , ,
()
$ + * 6 ) 6 ) 6 ) 6 ⋯을
()
.* + *.3 ' 3* ' *; . ; + ()
3* 을 간단히 하시오. 4'. , , ,
4' ) 4' )
4 ' .* ,
4' )
4 ' .-
만족시킬 때, .5 ' .% 의 값은?

① ! ② *()
$+4 ③ 4
④ *()
$ ⑤ *()
$ '4

유형 04 분모의 유리화

, ,
742. ) ' ) 을 간단히 하면?
() & )
& + , + () (& + , ' ()
4 2
7 .

&
, , &
① ) ② ) ③ )
()
+
* & , ()
+ & , ()
+
& ,
유형 05 무리식의 값 구하기

④ + *( )& ⑤ + *() &+,

()
- ' & + ()
-+&
746. & 6 ()
$ 일 때, ) 의 값을 구하시오.
()
- ' & ' ()
4 6 .
7

-+&

743. & 2 7 2 ! 인 & , 7 에 대하여 & ' 7 6 *()


4 3
7 . - , &7 6 * 일
) )
(& ' (7
때, ) 의 값을 구하시오.
()
& + ()
7

()- ()
,+& ()
, '&
747. 4 7 .
7 & 6 ) 일 때, )
* )
(, ' &
' ) 의 값은?
()
,+&
① , ② * ③-
, ④ 4 ⑤ $
744. 자연수 & 에 대하여 C0&16 ) 일 때,
()
& ' ()
4 4
7 .

&' ,
C0,1' C0*1' C0-1' ⋯ ' C0,!!1 의 값을 구하시오.

- 4 -
SSEN 수학 (하) 17. 무리식과 무리함수

748. , 유형 07 무리식의 값 구하기


()
& ' * 6 * 일 때, ) 의 값을
,
4 8
7 .

()
&+) ; & 6 . ± ()3 꼴
) ,
(& + )
()&+,
구하시오. 752. 5 2 .
7 & 6 - ' *()
- 일 때, &- + "&* + *& + -의 값을 구하시오.

유형 06 무리식의 값 구하기

753. & 6 ()
5 + , 일 때, 0& ' -10& + ,10&* ' *& ' 41 의 값은?
: & 6 ()
. ' ()
3 @ 7 6 ()
. + ()
5 3 .
7

3꼴 ① ,! ② ,$ ③ *!
④ *$ ⑤ -!
()
7 ()
&
749. 4 9
7 . & 6 ()
- ' ()
* , 7 6 ()
- + ()
* 일 때, ) ' ) 의
(& ()
) 7
값은?
-()
-
① ()
- ②) ③ *()
-
*
$( )
-
④ ) ⑤ -()
-
*

&4 ' -&- + %&* + #& ' 4


754. & 6 ()
$ + * 일 때, ) 의 값을
&* ' 4& ' *
5 4 .
7

구하시오.

()
* ', ()* + ,
750. &6))
@ 7 6 ) 일 때, ()
*& + ()
*7 의 값을
(* + , ()* ' ,
5 0
7 .

구하시오.

()
- ', ()- + ,
751. 유형 08 무리함수의 정의역과 치역

&6) @ 7 6 ) 일 때,
(*) ()
5 1
7 .

*
) ) )
(& + (7 (& ' (7 )
) ' 의 값은? 755. 함수 7 6 ()
+ *& ' * ' 3 의 정의역이 :& ; & ≤ .=이고,
()
& ' ()7 ) ()
& + ()
5 5 .
7

7
치역이 :7 ; 7 ≥ -=일 때, 상수 .@ 3 에 대하여 .3의 값은?
① ()
- + ()
* ② ()
"+* ③ *()
-
① +" ② +- ③ ,
④ (- ' ()
) * ⑤ ()
" '* ④ - ⑤ "

- 5 -
SSEN 수학 (하) 17. 무리식과 무리함수

유형 09

756. 5 6
7 . 함수 7 6+ ()
.& ' # ' 3 의 정의역이 :& ; & ≥+ -=이고, 무리함수의 그래프의 평행이동과 대칭이동
그래프가 점 0#@ + 41를 지날 때, 이 함수의 치역을 구하시오.
(단, .@ 3는 상수이다.) 759. 함수 7 6 ()
5 9 .
7 .0& + ,1 ' - 의 그래프를 & 축의 방향으로
3 만큼, 7축의 방향으로 F 만큼 평행이동하면 함수 7 6 ()" + -& 의
그래프와 일치할 때, 상수 .@ 3@ F에 대하여 .3F 의 값을 구하시오.

760. 보기의 함수 중 그 그래프가 평행이동 또는 대칭이동에


6 0 .
7

4& ' ,-
757. 함수 7 6 ) 의 그래프의 점근선의 방정식이 & 6 . , 의하여 함수 7 6 ()
+ & 의 그래프와 겹쳐지는 함수만을 있는 대로
&'$
5 7
7 .

고른 것은?
7 6 3 이고, 함수 C0&1 6 ()
.& ' 3 ' F에 대하여 C0!1 6 , 일 때,
함수 7 6 C0&1 의 정의역과 치역을 구하시오 . (단, .@ 3@ F 는 | 보 기 |
상수이다.) ㄱ. 7 6+ ()
& ㄴ. 7 6 ()
+ *&
,
ㄷ. 7 6+ ()
-+& ㄹ. 7 6 ) ()
4& + - ' ,
*

① ㄱ, ㄷ ② ㄷ, ㄹ ③ ㄱ, ㄴ, ㄷ
④ ㄱ, ㄷ, ㄹ ⑤ ㄴ, ㄷ, ㄹ

.& ' -
758. 5 8
7 . 함수 7 6 ) 의 그래프의 점근선의 방정식이 & 6 4 ,
&'3
7 6+ , 일 때, 함수 7 6 ()
3& ' . 의 정의역에 속하는 정수의 761. 함수 7 6 ()
6 1 .
7 + & ' , 의 그래프를 & 축의 방향으로 * 만큼,
최댓값은? (단, .@ 3 는 상수이다.) 7 축의 방향으로 + , 만큼 평행이동한 후 7 축에 대하여 대칭이동하면
① +* ② +, ③ ! )
7 6 (.& ' 3 ' F의 그래프와 일치한다. 이때 상수 .@ 3@ F 에 대하여
④ , ⑤ * . ' 3 ' F 의 값을 구하시오.

- 6 -
SSEN 수학 (하) 17. 무리식과 무리함수

유형 10 무리함수의 그래프가 지나는 사분면


765. 함수 7 6 ()
6 5 .
7 .& ( . / ! )의 그래프를 &축의 방향으로 ,만큼,
*& + -
762. 6 2
7 .
다음 중 함수 7 6 ()
*& ' 4 + - 의 그래프가 지나는 7 축의 방향으로 , 만큼 평행이동한 그래프가 함수 7 6 ) 의
&+,
사분면만을 모두 고른 것은? 그래프와 제, 사분면에서 만날 때, 상수 . 의 값의 범위는?
① 제 , , * 사분면 ② 제 , , - 사분면 ① ./ +4 ② +4/ . / ! ③ ./ +*
③ 제 , , 4 사분면 ④ 제 , , * , - 사분면 ④ +*/ ./ ! ⑤ ./ +,
⑤ 제 , , - , 4 사분면

유형 11

763. 6 3
7 . 다음 함수 중 그 그래프가 제 4사분면을 지나지 않는 것은? 그래프를 이용하여 무리함수의 식 구하기
① 76 ()
&+, ② 76 ()+&',
③ 7 6 ()
+&',+, ④ 7 6+ ()
+ 0& + ,1
766. 함수 7 6 ()
6 6 .
7 .& ' 3 ' F 의 그래프가
오른쪽 그림과 같을 때, 상수 .@ 3@ F 에
⑤ 7 6+ ()&',
대하여 . ' 3 ' F 의 값을 구하시오.

764. 6 4
7 . 함수 7 6+ ()
+ *& ' * ' . 의 그래프가 제,@ *@ - 사분면을
지나도록 하는 정수 . 의 최솟값을 구하시오.
767. 함수 7 6 ()
6 7 .
7 .0& ' 31 ' F 의 그래프가
오른쪽 그림과 같을 때 상수 .@ 3@ F 의
부호를 구하시오.

- 7 -
SSEN 수학 (하) 17. 무리식과 무리함수

768. 3
함수 7 6 ) ' F 의 그래프가 770. 함수 7 6 .()
7 0 . 3& 에 대하여 옳은 것만을 보기에서 있는 대로
&'.
6 8
7 .

고른 것은? (단, . ≠ !@ 3 ≠ ! )
오른쪽 그림과 같을 때, 함수
7 6 ()
.& ' 3 ' F의 그래프의 개형은? | 보 기 |

(단, .@ 3@ F 는 상수이다.) ㄱ. . 2 !@ 3 / !이면 그래프는 제* 사분면을 지난다.


ㄴ. 3 2 ! 이면 정의역은 :& ; & ≥ !=이다.
ㄷ. 그래프는 7 6+ .()
3& 의 그래프와 7 축에 대하여 대칭이다.
① ②
① ㄱ ② ㄴ ③ ㄱ, ㄴ
④ ㄱ, ㄷ ⑤ ㄱ, ㄴ, ㄷ

③ ④

유형 13 무리함수의 최대 ․ 최소

771. 7 1 . + , ≤ & ≤ - 에서 함수 7 6 - + ()
*& ' - 의 최댓값을 . ,
최솟값을 3라 할 때, . ' 3 의 값을 구하시오.

유형 12 무리함수의 그래프의 성질

769. 함수 7 6 ()
" + *& + , 에 대한 설명 중 옳은 것은?
772. 함수 7 6 ()
-& + " ' . 의 최솟값이 4 이고, 이 함수의
6 9
7 .

7 2 .

① 정의역은 :& ; & ≤ "=이다.


그래프가 점 03@ 51을 지날 때, 상수 .@ 3 에 대하여 . ' 3의 값은?
② 치역은 :7 ; 7 ≥ ,=이다.
① $ ② 5 ③ #
③ 그래프는 점 0-@ ,1 을 지난다.
④ ,, ⑤ ,-
④ 그래프는 7 6 ()
+ *& 의 그래프를 평행이동한 것이다.
⑤ 그래프는 제 - 사분면을 지난다.

- 8 -
SSEN 수학 (하) 17. 무리식과 무리함수

773. 7 3
. * ≤ & ≤ ,5 에서 함수 7 6 ()
-& ' . + $ 의 최댓값이 * 일 776. 두 집합 H 6 :0&@ 71 ; 7 6 ()
7 6 . & + * =,
때, 최솟값은? (단, . 는 상수이다.) I 6 :0&@ 7 1 ; 7 6+ -& ' G=에 대하여 H ∩I ≠ ∅일 때, 실수 G의
① +, ② +* ③ +- 최솟값은?
④ +4 ⑤ +$ ① - ② 4 ③ $
④ " ⑤ 5

777. 두 함수 7 6 ()
7 7 . & ' , , 7 6 & ' G 에 대하여 C0G1 를 두
함수의 그래프의 교점의 개수라 할 때,
774. + * ≤ & ≤ . 에서 함수 7 6 ()
, + 4& ' $ 의 최댓값이 ,
0 1 $
01
7 4
.

C ) ' C0,1' C ) ' C0*1 의 값을 구하시오. (단, G 는 실수이다.)


3@ 최솟값이 "일 때, . ' 3 의 값을 구하시오. 4 4

778. 직선 7 6 & ' , 이 & 축과 만나는 점을 A @ 이 직선과 함수


7 8 .

7 6 ()
.& + - 의 그래프가 만나는 서로 다른 두 점을 각각 B@ C라
하자. 점 B 가 선분 AC 를 * O - 으로 내분할 때 , 상수 . 의 값은?
유형 14 무리함수의 그래프와 직선의

① $ ② " ③ 5
④ % ⑤ #
위치관계
775. 7 5
. 함수 7 6 ()
4 + *& 의 그래프와 직선 7 6+ & ' G 가 서로
다른 두 점에서 만날 때, 실수 G의 값의 범위는?
$ $
① G≥ * ② G≤ ) ③ !/G/ )
* *
$ $
④ */ G≤ ) ⑤ *≤G/ )
* *

779. )
두 함수 7 6 (, + &* @ 7 6
7 9 .
()
+ &* ' ,!& + ," 의 그래프에
동시에 접하는 직선을 P 이라 하고 두 접점을 각각 A@ B라 할 때,
선분 AB 의 길이를 구하시오.

- 9 -
SSEN 수학 (하) 17. 무리식과 무리함수

유형 15 무리함수의 역함수와 그 성질 유형 16 무리함수의 합성함수와 역함수


☯ ☯

780. 8 0
7 .
함수 7 6 ()
.& ' 3 의 그래프와 그 역함수의 그래프가 모두 784. 정의역이 :& B & 2 ,=인 두 함수
8 4 .
7

점 0*@ ,1 을 지날 때, 상수 .@ 3 에 대하여 .3의 값은? &',


C0&16 ) @ Q0&16 ()
-& + *
① + *, ② +5 ③ +- &+,
에 대하여 0C ∘ 0Q ∘ C1 ∘ C10-1 의 값은?
+,
④ * ⑤ *,
① , ② * ③ -
④ 4 ⑤ $

781. 8 1
7 . 함수 7 6 *()
& + - ' , 의 그래프를 & 축의 방향으로 . 만큼
평행이동한 그래프의 식을 7 6 C 0& 1라 하자. 함수 7 6 C 0& 1 의
그래프와 그 역함수의 그래프가 접할 때, . 의 값을 구하시오. 785. 함수 C 0& 1 6 ()
8 5 .
7 *& + 4 에 대하여 함수 Q 0& 1 가
0C ∘ Q10& 1 6 & 를 만족시킬 때, 0Q ∘ Q 104 1 의 값은?
① 4% ② $! ③ $*
④ $4 ⑤ $"

782. 8 2
7 .
함수 C0&1 6 ()
& + . ' , 의 그래프와 그 역함수
7 6 C 0& 1 의 그래프의 두 교점 사이의 거리가 ()
+,
* 일 때, 상수
.의 값을 구하시오. 786. 두 함수 C 0& 1 6 ()
8 6 .
7 & + , ' 4@ Q 0& 1 6 ()
*& ' , 에 대하여
0C+ , ∘ Q1+ , 0*1 의 값을 구하시오.

783. 8 3
7 . 함수 C0&1 6 ()
.& ' 3 ' F 의 역함수
7 6 C+ , 0&1 의 그래프가 오른쪽 그림과 같을 787. 함수
8 7 .
7

때, 상수 .@ 3@ F 에 대하여 . ' 3 ' F 의 값은? ) 0& ≥ !1


① " ② 5 ③ %
C0&1 6 :,(+)(&
,+& 0& / ! 1
④ # ⑤ ,! 에 대하여 0C
+,
∘ C+ , 10.1 6 ," 을 만족시키는 상수 . 의 값을
구하시오.

- 10 -
SSEN 수학 (하) 17. 무리식과 무리함수

.& ' 4

791. ,! 이하의 자연수 .@ 3 에 대하여 두 함수 C0&1 6 ) @
&',
C단계 실력 굳히기
9 1 .
7

)
Q0&1 6 3& ' " 의 그래프가 &축 위에서는 만나지 않고 서로 다른
(
두 점에서 만나도록 하는 .@ 3 의 순서쌍 0.@ 31의 개수를 구하시오.
788. 8 8
7 . ! 이 아닌 실수 .@ 3@ F@ S에 대하여 (단 , . ≠ 4 )
.& ' 3
함수 7 6 ) 의 그래프가 오른쪽 그림과
F& ' S
같을 때,
) )
()
0.S + 3F1* ' ( )F3 ( )3F
를 간단히 하시오.

792. 다음 그림과 같이 함수 7 6 ()
9 2 .
7 *& 의 그래프와 직선 7 6 4 의
교점을 A @ 직선 7 6 4 위의 한 점을 B라 하자. 직선 AO 와 & 축의
양의 부분이 이루는 각을 직선 BO 가 이등분할 때, 삼각형 AOB의
넓이를 구하시오. (단, O는 원점이다.)

789. 8 9
7 .
오른쪽 그림과 같이 반지름의 길이가
,인
원 T 에 내접하는 사분원을 T ′이라 할 때,
사분원 T ′ 에 내접하는 원의 반지름의 길이는?
*( )
*+,
① ) ② *()
*+*
*
③ * + ()
* ④ ()
*+,
-( )
*+,
⑤ )
* 793. 어느 발효 식품 회사에서 생산한 제품 A 의 ,개당 가격을
9 3 .
7

& 만 원이라 하고, 숙성연도가 W년인 제품 A 의 월 판매량을


,!!!7 개라 할 때, &@ 7@ W사이의 관계식은
.()W
7 6 ) , & 6 3()W ' * ( . , 3 는 상수)
&
이고 그래프는 다음 그림과 같다.

+ *& ' 4
790. - ≤ & ≤ $ 에서 정의된 두 함수 7 6 ) 와
9 0
7 .

&+,
)
7 6 (-& ' G 의 그래프가 한 점에서 만나도록 하는 실수 G 의 숙성연도가 , 년에서 4년까지인 제품 A 의 월 판매량의 최솟값은

최댓값을 U 이라 할 때, U *의 값을 구하시오. %!!! 개, 최댓값은 #!!! 개이다. 숙성연도가 ," 년인 제품 A 의 월


X
판매량을 X개라 할 때, ) 의 값을 구하시오.
,!!

- 11 -
SSEN 수학 (하) 17. 무리식과 무리함수

794. 함수 C 0& 1 6 ()
& + . + 3 의 그래프와 그 역함수
797.
9 4
7 .

)
함수 C 0& 1 6 (& + *& + - 은 & 6 Z 또는 & 6 [ 에서
*
7 6 C+ , 0&1 의 그래프가 한 점에서 만날 때, 양수 .@ 3 에 대하여
9 7 .
7

)
최소이고, 함수 Q 0& 1 6+ (+ & ' *& ' % 은 & 6 \ 에서 최소이다.
*
.3의 최댓값은?
, , , 이때 세 점 A0Z@ C 0Z 11 , B0[@ C 0[ 11 , C0\ @ Q 0\ 11 에 대하여
① ) ② ) ③ )
"4 4% -* 삼각형 ABC 의 넓이를 구하시오.
, ,
④ ) ⑤ )
*4 ,"

795. , ,
두 함수 C 0& 1 6 ) & ' ) G 0& ≥ ! 1, Q 0&1 6 ()
*
$& + G 에 798. 오른쪽 그림과 같이 함수
9 8 .
7

$ $
9 5
7 .

7 6 ()
-& + - 의 그래프 위의 점 P 가
대하여 7 6 C 0&1 , 7 6 Q 0&1 의 그래프가 서로 다른 두 점에서
두 점 A0,@ !1, B04@ -1 사이를 움직일
만나도록 하는 모든 정수 G 의 개수는?
때, 삼각형 ABP 의 넓이의 최댓값을
① $ ② 5 ③ #
구하시오.
④ ,, ⑤ ,-

,
796. ! 이 아닌 세 실수 & , 7 , Y 가 ()
9 6
7 . & ()
7 6 ()
&7 , 799. 함수 C 0&1 6 ) ()
9 9 .
7

*
& + - ' , 의 역함수를 Q 0&1 라 할 때,

) ) 함수 7 6 Q 0&1 의 그래프를 & 축의 방향으로 . 만큼, 7 축의


(Y
)
()
7
Y
7 (
6+ ) 를 만족시킬 때,
방향으로 3 만큼 평행이동하면 함수 7 6 G& 의 그래프의 일부와
*

B& + 7B' ()
07 + Y1* + ()
0& + 7 ' Y1* 을 간단히 하시오. 겹쳐진다. 이때 상수 . , 3 , G 에 대하여 .3G 의 값을 구하시오.

- 12 -
SSEN 수학 (하) 18. 순열과 조합

18-2 순열

A단계 기본 다잡기
[0807~0810] 다음 값을 구하시오.
807. 0 7 .
8

*P&
18-1 경우의 수

800. 0 0
8 . 서로 다른 두 개의 주사위를 동시에 던질 때 나오는 눈의 808. 0 8 .
8

*P%
수의 합이 ! 또는 "이 되는 경우의 수를 구하시오.

809. 0 9 .
8
P
! !

[0801~0802] # 부터 $% 까지의 자연수가 각각 하나씩 적힌 $% 개의


공이 들어있는 주머니에서 한 개의 공을 꺼낼 때 , 다음을 구하시오 .
810. $P#⋅&-
801. $ 의 배수 또는 ## 의 배수가 적힌 공이 나오는 경우의 수
1 0 .
8

0 1
8 .

[0811~0814] 다음을 만족시키는 . 또는 / 의 값을 구하시오.


811. 1 1 .
8

.P ) 0 &%
802. & 의 배수 또는 " 의 배수가 적힌 공이 나오는 경우의 수
812.
0 2
8 .

1 2 .
8

$P/ 0 *%

"-
813. 1 3 .
8

"P/ 01
!-
[0803~0805] 두 지점 A , B 사이에는 & 개의 버스 노선과 ) 개의
지하철 노선이 있다. A 지점에서 출발하여 B 지점으로 갔다가 다시
814. 1 4 .
8
P 0 #)%
. .
A 지점으로 돌아올 때, 다음을 구하시오.
803. 0 3
8 . 갈 때는 버스를, 올 때는 지하철을 이용하는 방법의 수

[0815~0816] #, ), & , ! , $ 의 숫자가 각각 하나씩 적힌 $ 장의 카드


중에서 서로 다른 & 장을 뽑아 세 자리 자연수를 만들려고 한다.
다음을 구하시오.
804. 0 4
8 . 갈 때와 올 때 모두 버스를 이용하는 방법의 수 815. 세 자리 자연수의 개수
1 5 .
8

816. 짝수의 개수
1 6 .
8

805. 0 5
8 .
갈 때와 올 때 모두 지하철을 이용하는 방법의 수
[0817~0819] ! 개의 문자 A , B , C , D 를 일렬로 나열할 때, 다음을
구하시오.
817. 모든 방법의 수
1 7 .
8

806. * 종류의 샌드위치, ! 종류의 탄산음료, & 종류의 샐러드


818.
0 6
8 .

A 가 맨 마지막에 오도록 나열하는 방법의 수


중에서 샌드위치 , 탄산음료 , 샐러드를 각각 # 종류씩 고르는 방법의
1 8 .
8

수를 구하시오.

819. 1 9 .
8
B 와 C 가 이웃하도록 나열하는 방법의 수

- 1 -
SSEN 수학 (하) 18. 순열과 조합

[0832~0834] " 가지 무지개 색 중에서 ! 가지 색을 택하려고 한다.


18-3 조합

다음을 구하시오.
832. 빨간색을 포함하여 택하는 방법의 수
3 2 .
8

[0820~0823] 다음 값을 구하시오.
820. 2 0
8 .

4C)

833. 보라색을 제외하고 택하는 방법의 수


3 3 .
8

821. 2 1
8 .

#%C5

834. 노란색은 포함하고 초록색은 제외하여 택하는 방법의 수


3 4 .
8

822. 2 2
8 .
C
$ %

835. 남자 $ 명과 여자 ! 명 중에서 & 명의 대표를 뽑을 때,


823.
3 5 .
8

2 3
8 .

!C! 적어도 남자 # 명이 포함되도록 뽑는 방법의 수를 구하시오.

[0824~0825] 다음을 만족시키는 . 의 값을 구하시오. 836. 오른쪽 그림과 같이 원 위에 * 개의


824.
3 6 .
8

2 4
8 .

.C& 0 $* 점이 있을 때, 주어진 점을 이어서 만들 수


있는 서로 다른 직선의 개수를 구하시오.

825. 2 5
8 .

). 6 #C) 0 "5

[0826~0827] 다음을 만족시키는 . 또는 / 의 값을 구하시오.


826. 2 6
8 .

.C! 0 .C*

18-4 분할과 분배

827. 2 7
8 .
C C
" / 0 " /7 &

[0837~0839] 서로 다른 사탕 4 개가 있을 때, 다음을 구하시오 .

828. 2 8
8 . 서로 다른 4 개의 과자 중에서 * 개를 고르는 방법의 수를
837. 사탕을 ) 개, & 개, ! 개의 세 묶음으로 나누는 방법의 수
3 7 .
8

구하시오.

838. 사탕을 ) 개, ) 개, $ 개의 세 묶음으로 나누는 방법의 수


3 8 .
8

829. 2 9
8 . 동호회 회원 5 명이 다른 회원과 모두 한 번씩 악수를 할
때, 악수한 총 횟수를 구하시오.
839. 사탕을 & 개씩 세 묶음으로 나누는 방법의 수
3 9 .
8

[0830~0831] ! 명의 남학생과 & 명의 여학생에 대하여 다음을


구하시오.
830. & 명의 학생을 뽑는 방법의 수
840.
3 0
8 .

서로 다른 종류의 꽃 #% 송이를 & 송이, & 송이, ! 송이의 세


4 0 .
8

묶음으로 나누어 & 명에게 나누어 주는 방법의 수를 구하시오.


831. 3 1
8 . 남학생 ) 명과 여학생 # 명을 뽑는 방법의 수

- 2 -
SSEN 수학 (하) 18. 순열과 조합

845.

B단계 유형 뽀개기 부등식 8 6 )9 ≤ * 을 만족시키는 자연수 8 , 9 의 순서쌍


4 5 .
8

;8< 9= 의 개수는?
① $ ② * ③ "
④ 5 ⑤ 4
유형 01 합의 법칙

841. 4 1
8 . 서로 다른 두 개의 주사위를 동시에 던질 때 나오는 눈의
수의 합이 $ 의 배수가 되는 경우의 수를 구하시오.

846. 한 개의 가격이 각각 #%% 원, &%% 원, $%% 원인 & 종류의


4 6 .
8

사탕을 #%%% 원어치 사는 방법의 수를 구하시오.

842. 4 2
8 . # 부터 ! 까지의 자연수가 각각 하나씩 적힌 ! 개의 공이
들어 있는 주머니에서 한 개씩 세 번 공을 꺼낼 때, 꺼낸 공에 적힌
세 수의 곱이 & 또는 ! 가 되는 경우의 수를 구하시오.
(단 , 꺼낸 공은 다시 넣는다.)

847. 서로 다른 두 개의 주사위 A , B 를 동시에 던져서 나오는


4 7 .
8

눈의 수를 각각 ? , @ 라 할 때, 이차함수
9 0 8) 6 ;? 6 @=8 6 ?@ 6 # 의 그래프가 8 축과 만나지 않도록 하는
? , @ 의 순서쌍 ;?< @= 의 개수는?
① #) ② #! ③ #*
④ #5 ⑤ )%
843. 4 3
8 . # 부터 #%% 까지의 자연수 중에서 ) 와 $ 로 모두
나누어떨어지지 않는 자연수의 개수는?
① )% ② &% ③ !%
④ $% ⑤ *%

유형 02 방정식과 부등식의 해의 개수

유형 03 곱의 법칙

844. 4 4
8 .
방정식 8 6 )9 6 &: 0 )% 을 만족시키는 자연수 8 , 9, : 의
순서쌍 ;8< 9< := 의 개수를 구하시오. 848. 십의 자리의 숫자는 홀수이고 일의 자리의 숫자는 소수인
4 8 .
8

두 자리 자연수의 개수는?
① #$ ② )% ③ )$
④ &% ⑤ &$

- 3 -
SSEN 수학 (하) 18. 순열과 조합

849. 4 9
8 . 두 집합 A 0 B#< )< &< !C, D 0 B)< !< *C 에 대하여 853. 5 3 .
8 $!% 의 양의 약수 중 짝수의 개수를 H , & 의 배수의 개수를
E 0 B;?< @= F ?∈A < @∈D C 일 때, .;E = 를 구하시오. I 라 할 때, H 6 I 의 값을 구하시오.

유형 05 수형도를 이용하는 경우의 수


850. ;? 6 @ 6 G=;8 6 9 6 := 를 전개할 때, 항의 개수는?


854.
5 0
8 .

① & ② * ③ 4
5 4 .
8 #, ) , & , ! 를 일렬로 나열하여 네 자리 자연수 ?#?)?&?! 를
④ #) ⑤ #$ 만들 때, ?J ≠ J 를 만족시키는 자연수의 개수는?
(단, J 0 # , ) , & , ! )
① 4 ② #% ③ ##
④ #) ⑤ #&

유형 04 약수의 개수

855. 5 5 .
8 #, ) , & , ! , $ 가 각각 하나씩 적힌 $ 개의 농구공을 A # ,
A ) , A & , A ! , A $ 라 쓰여진 가방에 각각 # 개씩 넣을 때, ) 가 적힌
851. 5 1
8 . )5% 과 !)% 의 양의 공약수의 개수를 구하시오.
공은 A # 에 넣고, L 가 적힌 공은 A L 에 넣지 않는 방법의 수를
구하시오.
(단 , L 0 & , ! , $ )

852. ") 의 양의 약수의 개수를 ? , 양의 약수의 총합을 @ 라 할


856.
5 2
8 .

오른쪽 그림과 같은 팔면체의 꼭짓점


때, @ 7 ? 의 값은?
5 6 .
8

A 에서 출발하여 모서리를 따라 움직여


① #"# ② #"" ③ #5&
꼭짓점 F 에 도착하는 방법의 수를 구하시오.
④ #54 ⑤ #4$
(단, 한 번 지나간 꼭짓점은 다시 지나지
않는다.)

- 4 -
SSEN 수학 (하) 18. 순열과 조합

유형 06 지불 방법의 수와 지불 금액의 수 유형 07 도로망에서의 방법의 수


☯ ☯

857. 5 7
8 . #%% 원짜리 동전 # 개, $% 원짜리 동전 & 개, #% 원짜리 동전 860. 오른쪽 그림과 같이 ! 개의 도시
6 0 .
8

) 개의 일부 또는 전부를 사용하여 지불할 수 있는 방법의 수를 ? , A , B , C , D 를 연결하는 도로가 있다.


지불할 수 있는 금액의 수를 @ 라 할 때, ? 7 @ 의 값은? A 도시에서 출발하여 C 도시로 가는
(단, % 원을 지불하는 경우는 제외한다.) 방법의 수를 구하시오.
① ! ② $ ③ * (단, 한 번 지나간 도시는 다시 지나지
④ " ⑤ 5 않는다.)

858. $%% 원짜리 동전 ) 개, #%% 원짜리 동전 & 개, $% 원짜리


861.
5 8
8 .

오른쪽 그림과 같이 네 지점을


동전 ! 개의 일부 또는 전부를 사용하여 지불할 수 있는 방법의 수를
6 1 .
8

연결하는 도로가 있다. 집에서 출발하여


구하시오. (단, % 원을 지불하는 경우는 제외한다.)
도서관으로 가는 방법의 수를 구하시오. (단,
한 번 지나간 지점은 다시 지나지 않는다.)

859. #%%%% 원짜리 지폐 ) 장, $%%% 원짜리 지폐 ) 장,


862. 오른쪽 그림과 같은 도로망에서
5 9
8 .

6 2 .
8

#%%% 원짜리 지폐 & 장의 일부 또는 전부를 사용하여 지불할 수


B 지점과 D 지점을 연결하는 도로를 추가하여
있는 금액의 수는?
A 지점에서 출발하여 C 지점으로 가는 방법의
(단, % 원을 지불하는 경우는 제외한다.)
수가 4% 이 되도록 하려고 한다. 추가해야 하는
① )* ② )" ③ )5
도로의 개수는?
④ )4 ⑤ &%
(단, 한 번 지나간 지점은 다시 지나지 않고,
도로끼리는 서로 만나지 않는다.)
① & ② ! ③ $
④ * ⑤ "

- 5 -
SSEN 수학 (하) 18. 순열과 조합

유형 08 색칠하는 방법의 수

866. 오른쪽 그림의 $ 개의 영역을 서로 다른


6 6 .
8

$ 가지 색으로 칠하려고 한다. A , B , C , D , E 의


863. 6 3
8 .
오른쪽 그림의 $ 개의 영역을 빨간색, 영역에 같은 색을 중복하여 사용해도 좋으나 인접한
주황색, 노란색 , 파란색, 검정색 색연필을 사용하여 영역은 서로 다른 색으로 칠할 때, 칠하는 방법의
칠하려고 한다. A , B , C , D , E 의 영역에 같은 수는?
색을 중복하여 사용해도 좋으나 인접한 영역은 (단, 각 영역에는 한 가지 색만 칠한다.)
서로 다른 색으로 칠할 때 , 칠하는 방법의 수를 ① #*% ② )"% ③ &*%
구하시오. (단, 각 영역에는 한 가지 색만 칠한다.) ④ !)% ⑤ $!%

864. 6 4
8 . 오른쪽 그림의 & 개의 영역을
빨간색, 노란색 , 초록색, 보라색의 ! 가지
유형 09 이웃하는 순열의 수

색으로 칠하려고 한다. A , B , C 의 영역에


같은 색을 중복하여 사용해도 좋으나 인접한
영역은 서로 다른 색으로 칠할 때, 칠하는 867. 6 7 .
8 # 학년 학생 & 명과 ) 학년 학생 ! 명을 일렬로 세울 때,
방법의 수는? # 학년 학생끼리 이웃하게 세우는 방법의 수는?
(단 , 각 영역에는 한 가지 색만 칠한다.) ① *4% ② "%% ③ "#%
① )! ② )5 ③ &) ④ ")% ⑤ "&%
④ &* ⑤ !%

865. 6 5
8 . 오른쪽 그림과 같이 어느 도시를
! 개의 영역으로 나누어 놓은 지도를 서로 868. 6 8 .
8
POWER 에 있는 $ 개의 문자를 일렬로 나열할 때, P 와
다른 ! 가지 색으로 칠하려고 한다. A , B, R 를 이웃하게 나열하는 방법의 수를 구하시오.
C , D 의 영역에 같은 색을 중복하여
사용해도 좋으나 인접한 영역은 서로 다른
색으로 칠할 때 , 칠하는 방법의 수를
구하시오.
(단 , 각 영역에는 한 가지 색만 칠한다.)

- 6 -
SSEN 수학 (하) 18. 순열과 조합

869. 6 9
8 . 초등학생 ! 명, 중학생 & 명 , 고등학생 ) 명을 일렬로 세울 873. 7 3 .
8 & 명의 학생이 일렬로 놓인 5 개의 똑같은 의자에 앉을 때,
때, 초등학생은 초등학생끼리, 중학생은 중학생끼리 이웃하게 세우는 어느 두 명도 이웃하지 않게 앉는 방법의 수를 구하시오.
방법의 수는?
① &!%% ② &!$* ③ &$%%
④ &$!* ⑤ &*$%

유형 11 자리에 대한 조건이 있는 순열의 ☯

870. 7 0
8 .
남학생 . 명과 여학생 & 명을 일렬로 세울 때 , 여학생끼리 수
이웃하게 세우는 방법의 수는 &* 이다. . 의 값을 구하시오.
874. 남학생 & 명과 여학생 ! 명이 놀이공원에서 놀이기구를 타기
7 4 .
8

위해 일렬로 서서 기다리려고 한다. 이때 양 끝에 여학생이 오도록


세우는 방법의 수는?
① $!% ② *%% ③ ")%
④ #%5% ⑤ #!!%

유형 10 이웃하지 않는 순열의 수

871. 7 1
8 . 어느 산악 동호회에서 남자 ) 명, 여자 & 명이 일렬로 서서
지리산을 등반하려고 한다. 이때 남자끼리 이웃하지 않게 세우는 875. 7 5 .
8
promise 에 있는 " 개의 문자를 일렬로 나열할 때, 자음과
방법의 수를 구하시오. 모음을 번갈아 나열하는 방법의 수는?
① ") ② 4* ③ #!!
④ #4) ⑤ )55

872. 7 2
8 .
climate 에 있는 " 개의 문자를 일렬로 나열할 때, 모음끼리
이웃하지 않게 나열하는 방법의 수는?
① #!! ② &*% ③ ")% 876. 빨간색 꽃 & 송이와 노란색 꽃 ) 송이를 일렬로 심을 때,
7 6 .
8

④ #%5% ⑤ #!!% 노란색 꽃이 홀수 번째에 오도록 심는 방법의 수를 구하시오.


(단 , 꽃의 종류는 모두 다르다.)

- 7 -
SSEN 수학 (하) 18. 순열과 조합

877. 7 7
8 .
winter 에 있는 *개의 문자를 일렬로 나열할 때, _ 와 880. 8 0 .
a< b< c< d< e 의 $ 개의 문자를 일렬로 나열할 때, ?< @< G
`사이에 )개의 문자가 들어가도록 나열하는 방법의 수를 구하시오. 중에서 적어도 )개가 이웃하도록 나열하는 방법의 수를 구하시오.

유형 13 자연수의 개수

유형 12 ‘적어도’의 조건이 있는 순열의 수


881. 8 1 . %< #< )< &< ! 의 숫자가 각각 하나씩 적힌 $ 장의 카드에서


서로 다른 & 장을 뽑아 만들 수 있는 세 자리 자연수 중 &의 배수의
878. 7 8
8 .
silent 에 있는 *개의 문자를 일렬로 나열할 때, 적어도 개수는?
한쪽 끝에 모음이 오도록 나열하는 방법의 수는? ① #% ② #$ ③ )%
① &* ② ") ③ #!! ④ )$ ⑤ &%
④ )55 ⑤ !&)

882. 여섯 개의 숫자 #< )< &< !< $< * 에서 서로 다른 네 개를


8 2 .

사용하여 만들 수 있는 네 자리 자연수 중 백의 자리와 일의 자리의


숫자가 짝수인 자연수의 개수를 구하시오.

879. 7 9
8 .
남학생 ! 명, 여학생 *명 중에서 반장 # 명, 부반장 # 명을
뽑을 때, 다음을 구하시오 .
(1) 모든 방법의 수
(2) 반장, 부반장 모두 남학생을 뽑는 방법의 수
(3) 반장, 부반장 중에서 적어도 한 명은 여학생을 뽑는 방법의 수

883. 여섯 개의 숫자 %< #< &< $< "< 4 에서 서로 다른 네 개를


8 3 .

사용하여 만들 수 있는 네 자리 자연수 중 $ 의 배수의 개수를


구하시오.

- 8 -
SSEN 수학 (하) 18. 순열과 조합

유형 14 사전식으로 배열하는 방법의 수 유형 15 .Pr 와 .Cr 의 계산


☯ ☯

884. 8 4
. $ 개의 문자 A < B< C< D< E 를 모두 한 번씩 사용하여 888. 8 8 .

.P/ 0 &*%< .C/ 0 #$ 일 때, . 6 / 의 값은?


사전식으로 배열할 때, CBEDA 는 몇 번째에 오는지 구하시오. ① * ② " ③ 5
④ 4 ⑤ #%

885. 8 5
. 여섯 개의 숫자 #< )< &< !< $< * 에서 서로 다른 네 개를 889. 등식
8 9 .

.P ! 0 )%⋅.P) 를 만족시키는 자연수 . 의 값을


사용하여 만든 네 자리 자연수 중 !&%% 보다 큰 자연수의 개수를 구하시오.
구하시오.

886. FRIEND 에 있는 *개의 문자를 모두 한 번씩 사용하여


890.
8 6
.

사전식으로 배열할 때, &%# 번째에 오는 것은? 등식


9 0 .
8
C 0 #!C/ 6 )를 만족시키는 모든 자연수 / 의 값의
#! / )

① FIEDNR ② FINDER ③ FINDRE 합은?


④ FNDEIR ⑤ IFREND ① $ ② * ③ "
④ 5 ⑤ 4

887. 여섯 개의 숫자 %< #< )< &< !< $ 를 모두 한 번씩


891.
8 7
.

등식 .P) 6 *⋅.C) 0 #)⋅. 7 #C&을 만족시키는 . 의 값을


사용하여 여섯 자리 자연수를 만들 때, ##% 번째로 큰 수는?
9 1 .
8

① $%)#&! ② $%)&#! ③ $%)&!# 구하시오.


④ $%)!#& ⑤ $%)!&#

- 9 -
SSEN 수학 (하) 18. 순열과 조합

892. 9 2
8 . 8 에 대한 이차방정식 .C)8) 7 .C&8 6 .C$ 0 % 의 두 근을 895. 다음은 # ≤ / ≤ . 일 때, /⋅.C/ 0 .⋅.7 #C/ 7 # 이
9 5 .
8

e< f 라 할 때, ef 0 # 이다. e 6 f 의 값을 구하시오. 성립함을 증명하는 과정이다.

| 증 명 |
;. 7 # =-
.⋅.7 #C/ 7 # 0 .⋅ 1
;/ 7 #=- ㈎

01
;/ 7 #=-;. 7 /=-
/⋅.-
0 1 0 /⋅.C/
㈐ ;. 7 /=-
∴ /⋅.C/ 0 .⋅. 7 #C/ 7 #
유형 16 .Pr 와 .Cr 를 이용한 증명

위의 과정에서 ㈎, ㈏, ㈐에 알맞은 것은?

893. 9 3
8 .
다음 등식 중 옳지 않은 것은?
㈎ ㈏ ㈐
① ;. 7 / 7 #=- ;. 7 #=- ;/ 7 #=-
① .C% 0 .C. ② .C# 0.
② ;. 7 / 7 # =- .- /-
③ .P / 0 .C/ ⋅/- ④ . 6 #C/ 0 . 6 #C. 7 /
③ ;. 7 /=- ;. 7 #=- ;/ 7 #=-
.-
⑤ .C/ 01 ④ ;. 7 / =- .- ;/ 7 # =-
/- ;. 7 / =-
⑤ ;. 7 /=- .- /-

896. 9 6 .
8 # ≤ / g . 일 때, .C/ 0 . 7 #C/ 6 . 7 #C/ 7 # 이 성립함을
증명하시오.

894. 9 4
8 . 다음은 # ≤ / g . 일 때, . 7 #P / 6 /⋅. 7 #P/ 7 # 0 .P/ 가
성립함을 증명하는 과정이다.

| 증 명 |
;. 7 # =- ;. 7 # =-
P P
. 7 # / 6 /⋅. 7 # / 7 # 0 1 6 /⋅ 1
;. 7 # 7 /=- ;. 7 /=-
;. 7 #=-
0 1⋅ ㈎
;. 7 / =-
유형 17 조합의 수


0 1 0 .P /
;. 7 / =-
∴ . 7 # / /⋅. 7 # / 7 # .P/
P 6 P 0 897. 의사 "명과 간호사 * 명 중에서 & 명을 뽑을 때, & 명의
9 7 .
8

직업이 모두 같은 경우의 수를 구하시오.


위의 과정에서 ㈎, ㈏에 알맞은 것을 차례대로 나열한 것은?
① .< .- ② .< ;. 7 # =-
③ . 7 #< ;. 7 # =- ④ . 6 #< .-
⑤ . 6 #< ;. 6 # =-

- 10 -
SSEN 수학 (하) 18. 순열과 조합

898. 9 8
8 . 서로 다른 색연필 .자루와 서로 다른 공책 $ 권 중에서 유형 18 특정한 것을 포함하거나 ☯

색연필 & 자루와 공책 ) 권을 택하는 방법의 수가 )%% 일 때, . 의 포함하지 않는 조합의 수


값을 구하시오.
902. 혜원이와 민준이를 포함한 #) 명의 아카펠라 동호회 회원
0 2 .
9

중에서 축제 공연 무대에 설 회원 $ 명을 뽑을 때, 혜원이와


민준이를 모두 뽑는 방법의 수는?
① #)% ② #5% ③ )!%
④ &%% ⑤ &*%

899. 9 9
8 . 서로 다른 $ 개의 동아리에 회원이 각각 ! 명씩 있다. 이
)%명의 회원 중에서 &명을 뽑을 때, & 명 모두 다른 동아리의
회원을 뽑는 방법의 수는?
① !5% ② $)% ③ $*%
④ *%% ⑤ *!%

903. 0 3 .
9 #부터 #% 까지의 자연수가 각각 하나씩 적힌 #% 개의 공이
들어있는 상자에서 동시에 & 개의 공을 꺼낼 때, ) 가 적힌 공은
꺼내고 " 이 적힌 공은 꺼내지 않는 방법의 수를 구하시오.

900. 0 0
9 . # 부터 4 까지의 자연수가 각각 하나씩 적힌 4장의 카드가
들어 있는 주머니에서 동시에 & 장의 카드를 꺼낼 때, 카드에 적힌
수의 총합이 짝수가 되는 경우의 수를 구하시오.

904. 0 4 .
9 A < D 를 포함한 4 명의 학생 중에서 ! 명을 뽑을 때, A < D
중에서 한 명만 뽑는 방법의 수는?
① &$ ② $* ③ *&
901. 0 1
9 . # 부터 )% 까지의 홀수 중에서 서로 다른 두 수를 택할 때, ④ "% ⑤ #)*
택한 두 수의 합이 ! 의 배수가 되는 경우의 수를 구하시오.

- 11 -
SSEN 수학 (하) 18. 순열과 조합

905. 0 5
9 . 집합 A 0 B8 F 8 는 #) 이하의 자연수 C 에 대하여 다음에 908. 빨간색 꽃 !송이, 노란색 꽃 & 송이, 보라색 꽃 & 송이
0 8 .
9

답하시오. 중에서 !송이를 고를 때 , 다음을 구하시오. (단, 꽃의 종류는 모두


(1) 집합 A 의 부분집합 중에서 $를 원소로 갖고 원소의 개수가 &인 다르다.)
집합의 개수를 구하시오. (1) 빨간색 꽃이 적어도 #송이 포함되도록 고르는 방법의 수
(2) 집합 A 의 부분집합 중에서 &의 배수를 원소로 갖지 않고 (2) 노란색 꽃이 적어도 )송이 포함되도록 고르는 방법의 수
원소의 개수가 !이하인 집합의 개수를 구하시오.

909. 어느 카페에서 직원을 모집하는데 #) 명이 지원하였고, 이


0 9 .
9

중에서 & 명을 뽑으려고 한다. 남자 지원자가 적어도 한 명


포함되도록 뽑는 방법의 수가 #*! 일 때, 남자 지원자 수를 구하시오.
906. 0 6
9 . 크기가 서로 다른 $켤레의 구두 #%짝 중에서 !짝을 택할
때, 한 켤레만 짝이 맞도록 택하는 방법의 수는?
① #%% ② #)% ③ #$%
④ #5% ⑤ )#%

910. 1 0 .
9 #%미만의 자연수 중에서 서로 다른 ! 개의 자연수를 뽑을
때, 홀수와 짝수가 각각 적어도 # 개씩 포함되도록 뽑는 방법의 수를
구하시오.

유형 19 ‘적어도’의 조건이 있는 조합의 수


907. 0 7
9 . 남자 *명 , 여자 *명으로 구성된 모임에서 !명의 대표를
뽑을 때, 남자와 여자가 적어도 #명씩 포함되도록 뽑는 방법의
유형 20 뽑아서 나열하는 방법의 수

수는?
① !$$ ② !*$ ③ !"$
④ !5$ ⑤ !4$
911. 어른 $명과 어린이 * 명 중에서 어른 # 명과 어린이 )명을
1 1 .
9

뽑아 일렬로 세우는 방법의 수는?

① &5% ② !%% ③ !)%


④ !$% ⑤ !5%

- 12 -
SSEN 수학 (하) 18. 순열과 조합

912. 1 2
9 . # 부터 4 까지의 자연수 중에서 서로 다른 홀수 & 개, 서로 915. 1 5 .
9 두 집합 A 0 B#< )< &< ! C, D 0 B&< !< $C에 대하여
다른 짝수 )개를 택하여 만들 수 있는 다섯 자리 자연수의 개수는? 치역과 공역이 일치하는 A 에서 D로의 함수의 개수는?

① *%%% ② *!%% ③ *5%% ① )! ② )5 ③ &)


④ ")%% ⑤ "*%% ④ &* ⑤ !%

916. 집합 A 0 B#< )< &< !< $< *< "C에 대하여 다음 조건을
913.
1 6 .
9

1 3
9 . 지원이와 수현이를 포함한 5 명 중에서 $ 명을 뽑아 일렬로 모두 만족시키는 A 에서 A 로의 함수 k 의 개수를 구하시오
세울 때, 지원이와 수현이가 모두 포함되고 이들이 서로 이웃하도록
세우는 방법의 수를 구하시오
(가 )함수 k 는 일대일 대응이다.
(나 ) k ;# = 0 "
(다 ) L ≥ ) 이면 k ;L = ≤ L

유형 21 함수의 개수

유형 22 직선의 개수

914. 두 집합 i 0 B#< &< $ C, j 0 B)< !< *< 5< #%C에


917.
1 4
9 .

한 평면 위에 있는 서로 다른 $ 개의 점 중에서 어느 세
대하여 함수 k l i →j 가 다음 조건을 만족시킬 때 , 함수 k 의
1 7 .
9

개수는? 점도 한 직선 위에 있지 않을 때, 주어진 점을 이어서 만들 수 있는


서로 다른 직선의 개수는?
( 단, ?∈i < @∈i )

① 4 ② #% ③ ##
④ #) ⑤ #&
? g @ 일 때, k;?= g k;@=

① * ② 5 ③ #%
④ #) ⑤ #!

- 13 -
SSEN 수학 (하) 18. 순열과 조합

918. 1 8
9 . 오른쪽 그림과 같이 두 922. 대각선의 개수가 !!인 다각형의 꼭짓점의 개수를 구하시오.
2 2 .
9

평행한 직선 위에 5 개의 점이 있을
때, 주어진 점을 이어서 만들 수
있는 서로 다른 직선의 개수는?

① #$ ② #* ③ #"
④ #5 ⑤ #4

유형 24 다각형의 개수

919. 오른쪽 그림과 같이 정오각형의 각


923.
1 9
9 .

오른쪽 그림과 같이 정삼각형 위에


변을 연장하여 만든 별 모양의 도형 위에
2 3 .
9

#%개의 점이 있을 때, 주어진 점을 이어서 만들 같은 간격으로 놓인 4개의 점 중에서 & 개의


수 있는 서로 다른 직선의 개수를 구하시오. 점을 꼭짓점으로 하는 삼각형의 개수를
구하시오.

920. 2 0
9 .
오른쪽 그림과 같이 같은
간격으로 놓인 #) 개의 점이 있을 때, 두
점 이상을 지나는 서로 다른 직선의
개수를 구하시오.
924. 2 4 .
9 정칠각형의 꼭짓점 중에서 &개의 점을 꼭짓점으로 하는
삼각형의 개수는?

① )% ② )$ ③ &%
④ &$ ⑤ !%

유형 23 다각형의 대각선의 개수

921. 2 1
9 . 오른쪽 그림과 같은 팔각형에서
대각선의 개수는?
925. 오른쪽 그림과 같이 평행한 두
2 5 .
9

① #4 ② )% 직선 o< p 위에 각각 !개, $ 개의 점이
③ )# ④ )) 있다. 이 중에서 ! 개의 점을
⑤ )& 꼭짓점으로 하는 사각형의 개수를
구하시오.

- 14 -
SSEN 수학 (하) 18. 순열과 조합

929. 오른쪽 그림과 같이 가로 방향의


2 9 .
9

평행한 직선 ! 개와 세로 방향의 평행한 직선


926. 2 6
9 . 오른쪽 그림과 같이 반원 위에 있는
! 개가 각각 수직으로 만나고 있다. 직선
"개의 점 중에서 & 개의 점을 꼭짓점으로 하는 사이의 간격이 #로 일정할 때, 다음에
삼각형의 개수는? 답하시오.

① &% ② &# ③ &)


④ && ⑤ &!

(1) 이 평행한 직선으로 만들어지는 정사각형의 개수를 구하시오.

(2) 이 평행한 직선으로 만들어지는 정사각형이 아닌 직사각형의


개수를 구하시오.

927. 2 7
9 . 오른쪽 그림과 같이 같은 간격으로
놓인 #*개의 점 중에서 & 개의 점을
꼭짓점으로 하는 삼각형의 개수를 구하시오.

930. 평면 위에 .개의 평행한 직선과 만나는 . 6 ) 개의 평행한


3 0 .
9

직선이 있다. 이 평행한 직선으로 만들어지는 평행사변형의 개수가


)#% 일 때, . 의 값을 구하시오.

유형 26 분할하는 방법의 수

유형 25 평행사변형의 개수

931. 배, 감, 귤, 사과, 참외, 키위가 각각 한 개씩 있다. 이 * 개의


928.
3 1 .
9

2 8
9 . 오른쪽 그림과 같이 $ 개의 평행한 과일을 똑같이 바구니 & 개에 빈 바구니가 없도록 나누어 담는
직선과 ! 개의 평행한 직선이 서로 만날 때 , 이 방법의 수를 구하시오.
평행한 직선으로 만들어지는 평행사변형의
개수는?

① &% ② !% ③ $%
④ *% ⑤ "%
932. 3 2 .
9 남자 5 명, 여자 ! 명을 * 명씩 두 개의 조로 나눌 때 , 여자
! 명이 같은 조에 속하는 방법의 수를 구하시오.

- 15 -
SSEN 수학 (하) 18. 순열과 조합

933. 3 3
9 . 경찰관 " 명 , 소방관 & 명을 $ 명씩 두 개의 조로 나눌 때, 937. 3 7 .
9 운전석을 포함한 ! 인용 승용차 &대에 #% 명이 나누어
각 조에 적어도 한 명의 소방관이 포함되도록 나누는 방법의 수를 타려고 한다. 이 중에서 운전면허가 있는 사람이 & 명이고 이들은
구하시오. 각각 지정된 승용차를 운전한다고 할 때, #% 명이 차에 나누어 타는
방법의 수를 구하시오.

유형 27 분할한 후 분배하는 방법의 수


유형 28 대진표 작성하기

934. 3 4
9 . " 명의 학생을 &명, & 명, #명의 & 개의 조로 나누어 과학실,
음악실, 미술실을 청소하도록 하는 방법의 수는? 938. 씨름 대회에 참가한 * 명이 오른쪽
3 8 .
9

그림과 같은 토너먼트 방식으로 시합을 할


① #*% ② )#% ③ &*% 때, 대진표를 작성하는 방법의 수는?
④ !)% ⑤ 55%
① $% ② *%
③ "$ ④ 4%
⑤ #%%

935. 3 5
9 . 5 명의 학생이 ) 명씩 짝을 이루어 A< B< C< D ! 종류의
컴퓨터 게임을 하는 방법의 수를 구하시오. 939. 줄다리기 대회에 참가한 * 개의
3 9 .
9

학급이 오른쪽 그림과 같은 토너먼트


방식으로 시합을 할 때, 대진표를 작성하는
방법의 수를 구하시오.

936. 3 6
9 . " 층짜리 건물의 # 층에서 "명이 승강기를 함께 탄 후
"층까지 올라가는 동안 & 개의 층에서 각각 ) 명, ) 명, & 명이 내리는
방법의 수는? (단, 새로 타는 사람은 없다) 940. 4 0 .
9 4 개의 팀이 다음 그림과 같은 토너먼트 방식으로 시합을 할
때, 대진표를 작성하는 방법의 수를 구하시오.
① &#$% ② *&%% ③ 4!$%
④ #)*%% ⑤ #$"$%

- 16 -
SSEN 수학 (하) 18. 순열과 조합


943. 오른쪽 그림과 같이 #* 개의
4 3 .
9

C단계 실력 굳히기 정사각형으로 이루어진 판 위에 모양과


크기가 같은 흰 공 !개와 검은 공
4개를 놓으려고 한다. 이때 다음 조건을
941. 4 1
9 . 그림과 같이 중심이 같고 반지름의 길이가 각각 모두 만족시키도록 공을 놓는 방법의
수는?
#< )< &< !< $ 인 다섯 개의 원이 있다. 이 다섯 개의 원을 경계로
하여 안에서부터 다섯 개의 영역 A < D< E< q< r 로 나누고, 서로
다른 & 가지 색의 물감을 칠하여 색칠된 문양을 만들려고 한다. 각
영역은 # 가지 색으로만 칠하고, 이웃한 영역은 서로 다른 색을
칠한다. & 가지 색의 물감은 각각 #% 통 이하만 사용할 수 있고 물감
(가 ) 흰 공은 각 행과 각 열에 #개씩 놓는다.
# 통으로는 영역 A 의 넓이만큼만 칠할 수 있을 때, 만들 수 있는
(나 ) 검은 공은 각 열에 # 개씩 놓는다.
서로 다르게 색칠된 문양의 개수는?
(다 ) 각 열에서 흰 공과 검은 공을 이웃하지 않게 놓는다.

① ") ② 4* ③ #)%
④ #!! ⑤ #*5

① 4 ② #) ③ #$
④ #5 ⑤ )#

944. 길동이는 어느 인터넷 사이트에 가입하면서 비밀번호를


4 4 .
9

자신의 주민등록번호 앞자리인 %)%"#& 과 영어 이름인 sJotu.s을


이용하여 다음과 같은 규칙으로 만들려고 한다.

(가 ) 앞의 !자리는 숫자, 뒤의 !자리는 알파벳을 나열하여 총

942. 4 2
9 .
그림과 같이 경계가 구분된 * 개
5 자리의 비밀번호를 만든다.
(나 ) 주민등록번호 앞자리의 * 개의 숫자 중 !개를 사용하여
지역의 인구조사를 조사원 $ 명이 담당하려고
나열하며, % 이 )개 사용될 경우에는 %끼리 서로
한다. $ 명 중에서 #명은 서로 이웃한 ) 개
이웃한다.
지역을, 나머지 !명은 남은 ! 개 지역을 각각
(다 ) 영어 이름의 " 개의 알파벳 중 ! 개를 사용하여 나열하며,
#개씩 담당한다. 이 조사원 $ 명의 담당 지역을
s 가 )개 사용되고, s와 s 사이에 적어도 한 개의 모음을
정하는 경우의 수는?
나열한다.
(단, 경계가 일부라도 닿은 두 지역은 서로 이웃한 지역으로 본다.)
① ")% ② 5!% ③ 4*%
길동이를 만들 수 있는 비밀번호의 개수는?
④ #%5% ⑤ #)%%
① !*)% ② !*!% ③ !**%
④ !*5% ⑤ !"%%

- 17 -
SSEN 수학 (하) 18. 순열과 조합

945. 4 5
9 . # 부터 연속된 . 개의 홀수의 곱을 k;.= 이라 하자. 예를 947. 오른쪽 그림과 같이 4 개의
4 7 .
9

들어 칸으로 나누어진 정사각형의 각 칸에


k;#%=0 #⋅&⋅$⋅ ⋯ ⋅#4 4 개의 자연수 #< )< &< ⋯< 4 를 다음
조건을 모두 만족시키도록 적는 방법의
이다. 등식 $% C&5 0 ;가= ⋅k;)$= 가 성립할 때 , (가)에 알맞은 식은?
)$ )$ &" 수를 구하시오.
① ) ⋅k;#)= ② ) ⋅k;#&= ③ ) ⋅k;#)=
&" &5
④ ) ⋅k ;#&= ⑤ ) ⋅k ;#)=

(가 ) #행, ) 행, &행에 있는 세 수의 합이 서로 같다.


(나 ) )행의 가운데에는 $ 를 적는다.
(다 ) 색칠한 칸에 적힌 네 수의 합은 짝수이다.

948. 4 8 .
9 .이 ! 이상의 자연수일 때, 홀수와 짝수를 각각 적어도 두
개 이상 포함한 . 개의 자연수 ?#< ?)< ?&< ⋯< ?. 중에서 서로
946. 4 6
9 . 다음은 자연수 . 에 대하여
다른 두 수를 택하여 더하면 그 합이 짝수가 되는 경우의 수의
. 6 &C. 7 . 6 &C.7 # 0 ;다= ⋅ . 6 !C! 가 성립함을 증명하는 과정이다. 최솟값을 k;.=이라 하자. 옳은 것만을 보기에서 있는 대로 고른
것은?
| 증 명 |
| 보 기 |
. 6 &C. 7 . 6 &C.7 #
ㄱ. k ;!=0 )
#
0 ; ;가= = - 1 7 1B #
.- &- ;. 7 #=- !- C ㄴ. .이 짝수일 때, k;.=0 1
.) 7 ).
!
;나=
0 ; ;가= = -⋅ 1 .) 7 .
.- !- ㄷ. .이 홀수일 때, k;.=0 1
)
;. 6 !=-
0 ;다= ⋅ 1
.- !- ① ㄱ ② ㄴ ③ ㄱ, ㄴ
0 ;다= ⋅ . 6 !C! ④ ㄴ, ㄷ ⑤ ㄱ, ㄴ, ㄷ

위의 과정에서 (가), (나 ), (다 )에 알맞은 식을 각각 k ;.= , s;.= ,


k;#=6 s;)=
w;.=이라 할 때, 1 의 값은?
w;&=
① !% ② !) ③ !!
④ !* ⑤ !5

- 18 -
SSEN 수학 (하) 18. 순열과 조합

949. 4 9
9 . 다음 그림과 같이 크기가 같은 정육면체 모양의 투명한 951. 아래 그림과 같은 서로 다른 두 대의 !인승 레일바이크가
5 1 .
9

유리 상자 )!개로 직육면체를 만들었다. 있다. 남학생 &명과 민아를 포함한 여학생 ! 명이 다음 조건에 따라
모두 두 레일바이크에 탑승하려고 한다.

(가 ) 여학생은 ) 명씩 짝을 이루어 탑승하고 , 서로 옆자리에


앉는다.
(나 ) 민아가 탑승한 레일바이크에는 빈자리가 없도록 앉는다.

이때 레일바이크에 탑승하는 경우의 수를 구하시오.

이 중에서 *개의 유리 상자를 같은 크기의 정육면체 모양의 검은색


유리 상자로 바꿀 때 , 직육면체를 위에서 내려다 본 모양이 [그림
#], 정면을 기준으로 오른쪽 옆에서 본 모양이 [그림 ) ]와 같이
되도록 바꾸는 방법의 수를 구하시오.

952. 좌표평면 위에 4 개의 점 ;J < x=


5 2 .
9

( J 0 %< !< 5< x 0 %< !< 5 )이 있다. 이 4개의 점 중 네 점을


꼭짓점으로 하는 사각형 중에서 내부에 세 점 ;#< #= , ;&< #= ,
950. 5 0
9 . A , D 두 사람이 서로 다른 ! 개의 동아리 중에서 ) 개씩 ;#< &= 을 꼭짓점으로 하는 삼각형을 포함하는 사각형의 개수는?
가입하려고 한다. A 와 D 가 공통으로 가입하는 동아리가 # 개
이하가 되도록 하는 경우의 수를 구하시오.
(단, 가입 순서는 고려하지 않는다.)

① #& ② #$ ③ #"
④ #4 ⑤ )#

- 19 -
SSEN 수학 (하) 18. 순열과 조합

953. 5 3
9 . * 명의 학생이 탁구 시합을 하기 위해 다음과 같은 규칙으로 956. 오른쪽 그림과 같이 # 부터 4까지의
5 6 .
9

대진표를 작성하기로 하였다. 숫자가 각각 하나씩 적힌 4개의 정사각형을


빨강, 파랑, 노랑의 & 가지 색의 물감을
(가) * 명의 학생을 세 팀 A < D< E 로 나누는 데 각 팀은 # 명 사용하여 번호 순서대로 칠하려고 한다. 이때
이상 & 명 이하로 구성되어 있다. 같은 색을 중복하여 사용해도 좋으나 다음
조건을 모두 만족시키도록 칠하는 방법의
(나) 모든 학생은 서로 다른 팀의 학생 중 # 명과 # 번 시합을
수를 구하시오.
한다.

이때 시합을 하는 두 선수가 같아도 두 학생이 소속된 팀이 다르면 (가 ) 각 정사각형에는 #가지 색만 칠하고, 4개의 정사각형을

서로 다른 경우로 구분한다 . 예를 들어 y 학생과 z 학생이 시합을 모두 칠하는 동안 )회 이하로 색을 바꿀 수 있다.


하는 경우 오른쪽과 같은 ) 가지는 서로 다른 (나 ) )와 $ 가 적힌 정사각형은 같은 색을 칠한다.
경우라 할 때, 대진표를 작성하는 방법의 수를
구하시오.
(단, 대진표에 이름을 적는 순서는 고려하지
않는다.)

954. 5 4
9 .
오른쪽 그림과 같이 거리가 # 인 두 957. 오른쪽 그림과 같이 원 위에 같은
5 7 .
9

평행선 위에 # 의 간격으로 점이 각각 $ 개씩 간격으로 놓인 5개의 점 중에서 & 개의 점을


있을 때, 네 점을 꼭짓점으로 하여 만들 수 있는 사각형 중에서 꼭짓점으로 하는 직각삼각형의 개수를 ? , !개의
넓이가 ) 인 것의 개수를 구하시오. 점을 꼭짓점으로 하는 직사각형의 개수를 @ 라 할
때, ? 6 @ 의 값을 구하시오.

955. 5 5
9 . 주머니 속에 #< )< &< !< $ 가 하나씩 적혀 있는 구슬이 958. 승객 * 명이 타고 있는 버스가 세 정류장 A < B< C 에
5 8 .
9

각각 # 개, # 개, # 개, ) 개, & 개가 들어 있다. 5 개의 구슬 중에서 정차한다. & 개의 정류장 A< B< C 중에서 ) 개의 정류장에 모든
임의로 ! 개의 구슬을 동시에 꺼내어 네 자리 자연수를 만들 때, 승객이 내리는 방법의 수를 구하여라. (단, 새로 타는 승객은
같은 숫자끼리는 이웃하지 않는 자연수의 개수를 구하시오.

- 20 -

You might also like